You are on page 1of 331

國中數學

第一冊

部編版 評量
主題一 數與數線

1-1 負數與數線

壹、 基本題

一、 選擇題

1. ( ) 小明某次段考成績,和前次段考成績相比較,進步記

為 + ,退步記為-,下表是小明各科成績進退情形:

科目 國文 英語 數學 社會 自然

進退分 +5 -10 +5 +5 -10

若小明上次段考國文考了 60 分,請問本次段考小明國文分數為

多少?(A)55 (B)50 (C)65 (D)70

2. ( ) 以中午 12 時整為基準,下午 3 時記作+3,那麼上午

9 時可以記作多少?(A)+9 (B)-9 (C)+3 (D)-3

3. ( ) 如果 +20° 表示北緯 20° ,則南緯 20° 該如何表示?

(A)  +20° (B)  -20° (C)  +80° (D)  -80°

4. ( ) 以中午 12 時整為基準,下午 3 時記作+3,那麼上午 9

時可若要在數線上標示出 1.1 的坐標,應該怎麼做比較好?

(A)將 0 與 1 之間的長度等分為 10 格,從坐標 1 往左數 1 格即是 1.1

(B)將 1 與 2 之間的長度等分為 10 格,從坐標 1 往右數 1 格即是 1.1

(C)將 0 與 1 之間的長度等分為 10 格,從坐標 1 往右數 1 格即是 1.1


(D)將 1 與 2 之間的長度等分為 10 格,從坐標 1 往左數 1 格即是 1.1

5. ( )下列那個數在數線上的位置,是介於-7 與-5 之間?

(A)-4 (B)-5 (C)-6 (D)-7

6. ( )下列關於 0 的敘述何者正確?

(A)0 是正整數(B)0 是自然數

(C)0 是整數(D)0 是負整數

7. ( )數線上表示 2 與 9 兩點之間的整數點共有多少個?

(A)9ˉ(B)8ˉ(C)7ˉ(D)6

8. ( )數線上表示-2 與 9 兩點之間的正整數點共有多少個?

(A)11ˉ(B)9ˉ(C)8ˉ(D)7

9. ( ) 在數線上,數字 a 的位置記為 A 點,我們以 A (a ) 的符號

表示。現在數線上有 P (3 ) 、 Q (- 4) ,請問下列何者錯誤?

(A)P、Q 兩點的距離為 7 (B)P 點在原點的右邊

(C)Q 點在原點的左邊 (D)Q 點到原點的距離為 - 4

10. ( ) 若今天教室裡的溫度,以中午 12 點的溫度 25℃為基準

點,早上八點的溫度 20℃記為+5,那麼下午四點的溫度 23℃應

記為多少?

(A) + 2 (B) - 2 (C) + 3 (D) - 3 。

11. ( )如果賺錢以正號表示,賠錢以負號表示,若賺 100 元記


為+5,符號記為-2,代表什麼?

(A) 賠 2 元ˉ(B) 賠 60 元

(C) 賠 40 元ˉ(D) 賺 40 元

12. ( )如果賺錢以正號表示,賠錢以負號表示,若賺 100 元記

為+5,符號記為-2,代表什麼?

(A) 賠 2 元ˉ(B) 賠 60 元

(C) 賠 40 元ˉ(D) 賺 40 元

13. ( )下列那一個正整數,在數線上比較靠近-1.4?

(A)-1 (B)1 (C)0 (D) -2

14. ( )0 與 1 之間分成 10 等份,共有 9 個等份點,從左邊開始

的第 3 個等分點所代表的數字為

(A) 0.7 (B) 0.2 (C)  10  (D) 0.3
15. ( )下列敘述何者正確?

(A)數線上的單位長可以任意選取

(B)數線無法標明 0.99999

(C)數線上與原點距離 4 個單位長的點只有一個

(D)數線上的點,一定都是正數,

16. ( )在一數線上有五點 P、Q、R、S、T,所表示的數分別是

-4、-12、15、-8、5,則原點必在哪兩點之間?
(A)P 與 Q (B)Q 與 S (C)R 與 T (D)P 與 T
2  3 
17. ( )數線上介於- 3 與- 8 之間共有多少個分數的點?
(A)0 個ˉ(B)6 個ˉ(C)8 個ˉ(D)無限多個
49 
18. ( )數線上與- 12  最接近的整數為多少?
(A)-4 (B)-5 (C)4 (D)5

19. ( )下列敘述何者錯誤?

(A)數線上原點所代表的數最小

(B)任何數都可在數線上找到對應的點

(C)在 1 與 2 之間有無限多個點

(D)數線上代表 3.3333 的點只有一點


11 
20. ( )數線上表示-4、-1.25、3.7、-  2  的點和原點距離最
11 
遠的是哪一個?(A)-  2  (B)-4 (C)-1.25 (D)3.7
21. ( )下列哪一個數既不是正數,也不是負數?

(A) 5  (B) 2 (C) –0.1 (D) 0
3  -2 6 6  81 
3.3 = 3  1 = -1  1.81 = 1 
22. ( )  10 、– 0.12=  25  、  25 25 、  100 、 
211 
-2.789 = -2 
1000 ,以上正確的式子有多少個?

(A)1 (B)2 (C)3 (D)4

23. ( )毛利公司最近三個月來,第一個月淨賺 24.8 萬,第二個

月淨賠 13.2 萬,第三個月又淨賠了 10.7 萬,則毛利公司這三


個月合起來的盈虧如何?

(A) 淨賺 0.9 萬 (B) 淨賠 0.9 萬 (C) 淨賺 1.1 萬

(D) 淨賠 1.1 萬

24. ( )以海平面為基準,海平面以 0 公尺表示,海平面下方 200

公尺以 -200 公尺表示,有一艘潛艇原本的位置在海平面下方

1250 公尺處,現在它上升了 295 公尺,則新的位置要如何表示?

(A) 295 公尺 (B)  -1250 公尺 (C)  -955 公尺 (D) -1545 公尺

25. ( )以正午 12 時為準,正午 12 時用 0 表示,下午 8 時用+8

表示,則上午 8 時該如何表示?

(A) +8 (B) –8 (C) +4 (D) –4

26. ( )以上午 10 時為基準,上午 10 時用 0 表示,下午 2 時記

為 +8 ,則上午 2 時,以什麼表示?

(A)  -8 (B)  +8 (C)  -16 (D)  +16



B (3 )
27. ( )數線上有 A(-5.4) 、  4  兩點,請問下列哪一個敘述『不
正確』?


(A)  A 點所代表的數為 -5.4 (B)  B 點所代表的數為  4 

-5.4 > 3 
(C)  A 點到原點的距離較 B 點到原點的距離遠 (D)  4

28. ( )下圖中有 A 、 B 、 C 、 D 四個點。下列有關這四點坐標的

描述中,何者是『錯誤』的?
A  B  O  C  D 

- 4 - 3 0  1  2  4  5
3  2 
A( -4 )  D (4 )
(A)  5  (B)  B(-2)  (C)  C (1.5)  (D)  3 

29. ( )數線上 A、B 兩點,分別表示的數為 a、b,若 A 在 B 的

左邊,且 B 在原點 O 與 A 之間,則下列何者正確?

(A) 0<a<b (B) 0<b<a (C) a<b<0 (D) b<a<0

30. ( )將 -4 與 -5 這兩點所連成的線段 6 等分,則最左邊的等分


1  5  1  5 
-5  -4  -4  -5 
點為何? (A)  6 (B)  6 (C)  6 (D)  6
31. ( )在一數線上表示—1000 的點有幾個? (A)0(B)1(C)—

1000(D)1000 個。

32. ( )在一數線上規定原點右方為正向,則愈左方的點,所代

表的數(A)愈大(B)愈小(C)相等(D)不能比較。

33. ( )在數線上 A 點表示的數是-4,則 A 點在原點的 (A)左邊

(B)右邊 (C)原點上 (D)無法確定

34. ( )0 是 (A)正數(B)負數(C)整數(D)自然數。

35. ( )數線上 3 與 4 之間共有多少個點?(A)0(B)1(C)2(D)無限

多點
1  1
-
36. ( )若-x=2,則 x= (A)  2 (B)  2 (C) 2 (D) -2 。
37. ( )下列何者不正確?

(A) -8>-11 (B) │-8│>│-6│ 


3  2 
(C) 6> │-4│(D) - 4 >- 3 。
1  8 
- 3. 5 
38. ( )-0.1、- 2 、0、0.09、  、-4、-|  5  | ,左
列數中,正數有 a 個、負整數有 b 個,則 2 × a+b=?

(A) 6 (B) 7(C)8 (D) 9。


1  1  1 
39. ( )-( 3 + 4 - 5 )=?
23  23  1  1 
(A)  60  (B) - 60  (C)  60  (D) - 60  。

40. ( )1 4  的相反數是
2  3  7 

(A)  3  (B) -1 4  (C)  4  (D) 1.75
41. ( )0 的相反數為 (A) -1(B) 1(C) 0(D)不存在。

42. ( )0 屬於下列哪一類?

(A)正數(B)負數(C)整數(D)以上皆非。

43. ( )下列何者在數線上的位置最接近 0?
1  1  1  1 
(A) - 96  (B) - 97  (C)  98  (D)  99 。
44. ( )a 小於它的相反數,則 a 是

(A)正數(B) 0 (C)負數(D)正分數。

45. ( )若│甲數│=3,則 甲數=?

(A) 3 (B)-3 (C) 3 或-3 (D)不存在。

46. ( )下列那一個數的絕對值會小於 4?

(A) 1 (B) -2 (C) 3(D) -5。


47. ( )│A│=│B│,若 A>B,且 A、B 均不為 0,則

(A) A>0,B<0 (B) A<0,B<0 (C) A>0,B>0

(D) A<0,B<0。

48. ( )絕對值小於 9 的數(包含整數、分數、小數)有幾個?

(A) 8 (B) 9 (C) 17 (D)無限多。

49. ( )甲是整數,且 6<|甲|<10,則甲有多少個?

(A) 3 (B) 4 (C) 5 (D) 6。

50. ( )在數線上 A(-6),何者錯誤?

(A) A 點在原點的左邊(B) A 點在負向上

(C) A 點的對稱點是 6 (D)A 點與原點相距-6 單位長。

51. ( )數線上位於原點右側的點是(數線以原點右側為正向)

(A) 負整數 (B) 負數 (C) 正整數(D) 正數。

52. ( )在設定數線上的單位長的時候,可以取下列何種長度

(A) 1 公寸 (B) 1 英吋 (C) 1 cm (D) 以上皆可。



-2
53. ( )數線上有三點 A(  2  )
、B(-2)
、C(-3.12)
,哪一
點在另外兩點之間?

(A) A (B) B (C) C (D) 以上皆可。

54. ( )數線上一點 B(b)到原點的距離是

(A) b (B) | b | (C) -b (D) 不一定。


55. ( )a<0,b<0,c<0, | c | > | b | > | a | ,哪一

個數位置在最左邊?

(A) a (B) b (C) c (D)無法判別。

56. ( )數線上單位長 3 cm,A(-2)


,則 A 點與原點的距離=?

(A) 6 cm (B) -6 cm (C) 5 cm (D) 負數沒有距離。



57. ( )數線上 A(9)
,若改變原數線的單位長為原來的 3 倍,則
A 點坐標會改成 (A) 27 (B)-27 (C) 3 (D)-3。

58. ( )如圖,哪一點所表示的數最大?

(A) A (B) B(C) C(D) D

59. ( )在數線上欲畫出 2.125 這個數至少要在 2、3 兩整數間分

成幾等分? (A) 1000(B) 100(C) 10(D) 8

60. ( )A(4)
、B(7),若以 A 點為數線上新的原點,且單位長
7  4 
不變,則 B 點是 (A)-3 (B) 3 (C)  4  (D)  7 
61. ( )|甲|>|乙|哪個數距離原點較遠?

(A)甲(B)乙(C)不一定(D)相同。

62. ( )在數線上 10、11 之間有多少個分數的點?

(A) 10 個(B) 9 個(C) 1 個(D) 無限多個。

63. ( )數線上一個點 A(a),而 B(-a)是其對稱點,則 a 是


(A) 正數 (B) 負數 (C) 0 (D) 不能確定正、負數。

64. ( )在數線上-1、-2 兩個數之間,分成 10 等分得 9 個等

分點,則由左往右起第 4 個等分點所表示的數是

(A)-1.4 (B)-1.6 (C)-2.4 (D)-2.6。

65. ( )數線上有一隻螞蟻在 12 所代表的點,先向左走 18 個單

位,再向右走 5 個單位,這隻螞蟻最後停留在哪個數的位置?

(A) 25 (B) 35 (C) -1 (D) 1。

66. ( )A(6)
,在數線上的對稱點是 B 點,單位長 10 公分,則

A、B 兩點的距離

(A) 1.2 公尺(B) 1.2 公分(C) 12 公分(D) 60 公分。


3  2 
-
67. ( )
『 -8 、 5 、  3 、 -17 、 -0.7 、99.9、0.25』
上面各數中,共有多少個負數?

(A)2 個 (B)3 個 (C)4 個 (D) 5 個


解答:

1.C 2.D 3.B 4.B 5.C 6.C 7.D 8.C 9.D 10.A

11.C 12.C 13.B 14.D 15.A 16.D 17.D 18.A 19.A 20.A

21.D 22.B 23.A 24.C 25.D 26.C 27.D 28.A 29.C 30.B

31.B 32.B 33.D 34.C 35.D 36.D 37.D 38.C 39.A 40.B

41.C 42.C 43.D 44.C 45.C 46.D 47.A 48.D 49.D 50.D

51.D 52.D 53.A 54.B 55.C 56.A 57.A 58.C 59.D 60.B

61.A 62.D 63.D 64.B 65.C 66.A 67.C


二、 填充題

1. 以海平面為基準點,比海平面高 200 公尺的高度,可記為+200

公尺;潛入海平面以下 100 公尺的深度,可記為_________公

尺;小天爬山離平面高 1200 公尺,可記為 公尺。

2. 在數線上有 A、B 兩點,距離為 6,若 A 點所表示的數是 8,則

B 點所表示的數是 。

3. 最大的負整數是 ,最小的正整數是 。

4. 以學校為準,學校的東方 5 公里處,記為+5 公里時,則學校的

西方 12 公里處,應記為 公里。

5. 一年六班以顏老師的身高 175 公分為基準,記錄全班同學的身

高。顏老師身高 175 公分,記成 0 公分,小琪身高 160 公分,

記成  -15 公分,如果小安身高 182 公分,應該記成_________

公分。

-6 
6. 在數線上, A 點表示  3 ,如果 B 是表示整數的點,而且最接
近 A 點,則 B 點可以表示為 。

7. 數線上 A、B、C 三點,所代表的數分別是 2、6、12,若 AB = CD ,

則 D 點所代表的數是 。

8. 畫一條數線需要注意的要素為 、 、 。

9. 如右圖,數上 A 點表示的數是 ,B 點所表示的數是 。


­3 A  B  ­2 

10. 在一數線上,與代表—2 的點距離為 10 個單位的點代表哪些

數 。

11. 若以 155 公分為標準,甲生身高 165 公分,表示+10,則乙生身

高 151 公分可表示成 。

12. 以正午 12 點為準,若下午四點以「+4」表示,則上午 7 點

用 表示。

13. 東方和西方是相對的,如果由某地向東走2公尺,以+2表示,

則由同一地點往西走 2.5 公里,就以 表示之。

14. 阿里山上,早上溫度為8℃,到夜晚溫度降低了 12℃,則此時

山上的溫度為 (用正負數表示)

15. 甲、乙二人現年都是 12 歲,問 5 年後誰年齡較大

16. 在數線上找出最大的負整數為 ,最小的正整數為 。

17. 如果|a|=5,則 a 的值為 。

18. 已知 a 小於 6,在-3、-4、-5、-6、-7 中,依對稱點

的觀念,判斷哪些數可能是 a 的相反數。答: 。

19. 在數線上,某數的絕對值(與原點的距離)等於 12 的點共有

個 
解答:

1.-100、+1200 2.2 或 14 3.-1;1 4.-12 5.7 6.-7

7.8 或 16 8.原點、單位長、方向(箭頭) 9.(1)-2.5 (2)-2.25

10. —12,8 11.-4 12.-5 13.-2.5 14.-4℃ 15.一樣大

16.-1;1 17.5 或-5 18.-7 19.2


三、 綜合、題組題

12 3 
1. 畫出一條數線,並在數線上標示 5、 - 、-1 及  。
5  4 

2. 請在【 】中填入「>」、「=」或「<」來表示各組數的大小

關係:
4 2
- -
(1)-5【 】-2 (2)  5 【 】  3 
4 2 
- -5
(3)-0.2【 】  5  (4)|5.4|【 】  5 

7  1 
-2
(5)| 3 |【 】|  5 |

3. 求出下列絕對值的值:

4 3 
- - - (- )
(1)  - (-3 )  (2)  5  (3)  7 

4. 請在空格中填入「>」「=」「<」來表示各組數的大小關係:
1  2
-
(1) -9 -5 (2) - 2  3 

1
-
(3) -0.3  4  (4)  ­ 5  3 

7  2 
(5)  ­ 1 
5  5 

5. 如果以正號來表示體重增加,以負號來表示體重減少。請回答

下列問題:

(1)大寶胖了 5 公斤,應怎麼記錄?

(2)大妹瘦了 6 公斤,應怎麼記錄?
(3)大平原本體重 54 公斤,現在變成 62 公斤,應怎麼記錄?

6. 小傑以生日 5/20 當天的凌晨 0 點為基準點,過了 5/20 當天凌

晨 0 點以正號表示,5/20 凌晨 0 點以前使用負號表示,例如 5/19

晚上十點記為-2,5/20 凌晨三點記為+3,試回答下列問題:

(1)5/20 早上八點應怎麼記錄?

(2)5/19 晚上六點應怎麼記錄?

(3)如果記錄為-4,應該是幾月幾日幾點?

7. 若水庫的水位上升 1 公分以+1 表示,下降 1 公分以-1 表示,請

回答下列問題:

(1)水庫水位上升 10 公分該如何記錄?

(2)水庫水位下降 4 公分該如何記錄?

(3)水庫水位上升 3 公寸,該如何記錄(1 公寸=10 公分)?

8. 請依據下列於數線上標示坐標點的動作寫出坐標。

(1)在數線的-1 與-2 兩個坐標間取中點,該點所代表的坐標為多少?

(2)將數線的-2 與-3 兩個坐標間四等分,自坐標-3 向右的第一個四

等分點,該點所代表的坐標為多少?

(3)在數線的 3 與 4 兩個坐標間作十等分,自坐標 4 向左的第二個十

等分點,該點所代表的坐標為多少?

9. 若箭號表示正的方向,下圖是小誠繪製的數線,請問是否正確?
請說明理由。

2  1  23 
2  - -
10. 『0.3、19、–1、0、  3 、  7 、  5 、100、–0.01、9.09、–1001』
(1)上面各數中,共有幾個正數?

(2)上面各數中,那些數是正整數?

11. 如果以+號表示存款增加,-號表示存款減少,下表是小裕從七

月以來的存款數字,請回答下列問題:

月份 存款 (1)8 月與 7 月存款相比較,是增加還是減少?該
7 1200
如何記錄?
8 1600
9 1300 (2)9 月與 8 月存款相比較,是增加還是減少?該

如何記錄?

12. 試比較下列各組數的大小(請填>或<):


(1)3  -5 (2)6.25 6 (3)  4  0
2  1 
- 1  -2 
(4) -1 0 (5)  3 3 (6) - 2.125 -2

13. 將下列各數按由大至小順序排列,且用「>」分隔開來:
7  1 
- 5 
3、  2 、 -1.2 、  4  、 -0.5
14. 在下列各空格中填入適當的數:

(1)賺與賠是相對的,賠 3000 元記為 -3000 元,那麼賺 2500 元可

以記為 元。
(2)上升與下降是相對的,如果上升 5 公尺,以+5 公尺表示,

那麼下降 3 公尺可以用 公尺表示。

(3)東和西是相對的,如果由甲地向東走 2 公里,以+2 公里表

示,那麼由甲地向西走 2.5 公里,就用 公里表示。


4  5 
-3  5 
15. 請寫出比  7 大,且比  6 小的所有整數。

16. 在下列空格中填入適當的數:

(1) 0 的相反數為 ; -2 的相反數為 。

(2) – ( – 5 ) = ;– ( + 3 ) = 。

17. 在下列空格中填入適當的數:

•比 0 小 13 的數記為 ‚比 0 小 7.5 的數記為


4  7 
2  -
ƒ比 0 小  5 的數記為 „  8 是比 0 小 的數
18. 在下列空格中填入適當的數:

•以海平面為基準點,若海平面上方 300 公尺記為+300 公尺,

則海平面下方 500 公尺記為

‚以目前的氣溫為基準點,如果氣溫下降 30 °C 記為 -30 °C ,則氣

溫上升 10 °C 記為

19. 比較下列各數的大小關係,在空格中填入「=、<、>」

-4 
•3 -5 ‚-4 0 ƒ-3 -9 „-4.5  2
…-3.7 -4 †-3.7 -3

20. 在一條水平的數線上,若以向右為正向,則

•愈右邊的點表示的數愈 ,

‚愈左邊的點表示的數愈 。

21. 在數線上標出 A(-5)、B(7)、C(0)、D(-2)

­10­9 ­8 ­7 ­6 ­5 ­4 ­3 ­2 ­1 0  1  2  3  4  5  6  7  8  9 10

4  13 
-1 
22. 分別求出 7 、  5 、  4  、 3.1 的相反數
23. ( ) 數線上有 A、B、C、D 四點,已知 A 點在 B 點左邊,C

點在 B 點的右邊,D 點在 A 點的左邊,則四個點中,最大的點

是 C。
1  2  1  3 
3  -3  -1 
24. 在數線上標出 A( 2  )、B(  3 )、C(  3 )、D(  4 ),並比較大小。

­4  ­3  ­2  ­1  0  1  2  3  4 

1  1  1  1 
25. 在數線上標出 A(  )、B( 3  )、C( -3  )、D( - )分別寫下各點與
2  3  3 2

­4  ­3  ­2  ­1  0  1  2  3  4 


原點的距離 。
5  1 
26. 畫出一條數線,並在數線上由大到小標出- 3 ,-1 4 ,1.5,0,
-4 的點。

27. 一年一班六位同學的某次數學小考成績與全班平均成績的差距 
統計如附表,個人成績比全班平均成績

高 1 分,以「+1」表示,如果全班平均分數為 72 分。請問:

學生 功佳 程文 韋杰 育翔 中佑 宗翰
成績與全班平均 +11 -3 -8 +9 0 +2
成績的差距

(1) 這六位同學中,最高分與最低分相差幾分?
(2)這六位同學的平均分數為幾分?
2  1 1 
-3 - 1 
28. 畫出一條數線,並在數線上標出  3 , 2.5 ,  4 ,  - 2 ,  5 各

數的點。

2  æ 5 ö
- ç - ÷
29. 寫出下列各數的相反數:4、-3、 7  、 - [- (- 9)] 、 è 6 ø

30. 求下列各式的值:

(1) 34+|-(-0.6)|= __________。



(2) -(-2)-|- 3 |= __________。
31. 請用正、負數表示下列各相反或相對的量:

(1)用+20 公噸表示進貨 20 公噸,則出貨 30 公噸可用 公

噸表示。

(2)如果用-50 表示在海平面下方 50 公尺處,則在海平面上方 200

公尺,可用 表示。

(3)如果以+3 公里表示由火車站向東走 3 公里,那麼從火車站向西

邊走 5 公里,可以用 公里表示。
(4)如果以-3 表示從水桶中取出 3 公升的水,則將 5 公升的水加入

水桶中,應記為 公升。

(5)若以+3 表示比班上平均分數多 3 分,則比班上平均分數少 15

分可以用 表示。

32. 在下列【 】中填入適當的數:

(1) |-37|=【 】
。(2) |-(-0.6)|=【 】

7  8 
(3) -|- 8  |=【 】
。(4) -|-(- 5 )|=【 】

33. 在下列【 】中填入適當的數:

(1) -(-24)=【 】
。(2) -(-1)+1=【 】。

(3) 3-(-(-3)
)=【 】。

(4) -(-(-(-5.63)
))=【 】。

34. 求下列各數:

(1) -(21-19) =【 】
。(2) -(1-0.32)=【 】

2  3 
(3) -(  -  )=【 】。
3  2 

35. 將下列各數依大小順序排列:
4 4
--
4、-6.2、0.3、  3 、  5 
36. 請用正、負數表示出下列各相反或相對的量:

(1)若用+30 公克表示進貨 30 公克,則出貨 50 公克可

用 __________ 表示。
(2)如果用-240 表示在海平面下方 240 公尺處,則在海平面

上方 60 公尺,可用 __________ 表示。

(3)如果以+2 公里表示由家門口向東走 1 公里,那麼從家門

口向西走 2.5 公里,可以用 __________ 公里表示。

(4)如果以-1 表示從水缸中取出 2 公升的自來水,則將 6 公

升的自來水加入水缸中,應記為 __________ 。

(5)若以+3 表示水庫水位升高 1 公尺,則水位下降 2 公尺可

以用__________ 表示。

37. 求下列各數的絕對值:
4 2 
- 4 
2 、  5  、 -(-119) 、  5 

38. 請判斷下列敘述是否正確。(正確者打○,錯誤者打╳)

(1)【 】 當 a<0 時,|a|= -a。

(2)【 】 當 a=-b,則|a|= |b|。

(3)【 】 當 a<b,則|a|<|b|。

(4)【 】 a+|a|一定是正數。

(5)【 】 -|a| £ -a。

(6)【 】 若 a+b=0,則|a|= |b|。

39. 求下列各數的相反數:

2  11 
-4.321、911、 - 6 、  、-(-1)、-(-(-27))
5  9 
40. 下列各數中,哪些是小於 0 的數?答【 】
。 
2  1 
3 、-2.8、0、-7、-2 9 、6

41. 請把下列各點在數線上的位置由左到右依序排列出來。

1 1 
A( - )、B( - 3 )、C(2.6)、D(-3)
2  2 

42. 在如圖中,哪些點坐標的絕對值大於 2?答:


【 】。

43. 在如圖中,A 點的坐標是 8, B 點的坐標是 6。請在下面的數線

上標示出 C(11)和 D(2.5)來。

B    A 

6    88 

44. 請寫出數線上 A、B、C、D、E 點的位置,並求出各點到原點的

距離。 
解答:


1.略 2. (1)<;(2)<;(3)>;(4)=;(5)> 3.(1) 3 (2) 

3  2  2  4 
(3)  (4)  (5) 2  (6) 3  4.(1) < (2) > (3) < (4) >
7  5  3  5 

(5) = 5.(1)+5 公斤(2)-6 公斤(3)+8 公斤 6.(1)+8 (2)-6

(3)5/19 晚上八點 7.(1)+10 (2)-4 (3)+30 8.(1)-1.5

(2)-2.75 (3)3.8 9.不正確,數線應有原點、單位長及箭號表示

正的方向,小誠的數線正向位置填的是負數,應該填入正數。

10.5 個、19 及 100 11.(1)+400 元 (2)-300 元 12. (1)> (2)> (3)>

1  7 
(4)< (5)> (6)< 13. 5  >3> -0.5 > -1.2 > - 14.(1) +2500 (2) -3
4  2

(3) -2.5 15.  -3 、 -2 、 -1 、0、1、2、3、4、5 16.0,2;5,-3

4  7 
17. •-13 ‚-7.5 ƒ  -2  „  18. •-500 公尺 ‚ -10 °C
5 8 

19. •>‚<ƒ>„=…>†< 20. •大‚小

­10­9 ­8 ­7 ­6 ­5 ­4 ­3 ­2 ­1 0  1  2  3  4  5  6  7  8  9 10 

A  D  C  B
21.
4  13 
22.  -7 、 1  、 - 、 -3.1 23.(×)∵數線上的點無大小之分。
5  4

­4  ­3  ­2  ­1  0  1  2  3  4 


24. B>A>D>C 
­4  ­3  ­2  ­1  0  1  2  3  4
1  1 
25. A、D:  ,B、C: 3 
2  3 
2  5
26.略 27.19、74 28.略 29.-4、3、-  、9、 -
7  6 

30.34.6、  31.(1)-30 (2)+200 (3)+5 (4)+5 (5)-15

7  8
32.(1)37 (2)0.6 (3)  (4)- 33.(1) 24 (2) 2
8  5


(3)0 (4)5.63 34.(1) -2 (2) -0.68 (3) 

4 4
35.4>0.3>、 - > - >-6.2 36.(1)-50 公克 (2) 60 或+60
5  3 
4  2 
(3)-5 (4)3 或+3 (5)-2 37. 2 、  、 119 、 4 
5  5 

38.(1) ○ (2) ○ (3) ╳ (4) ╳ (5) ╳ (6) ○

2  11 1 
39.4.321、-911、 6  、 - 、-1、27 40.-2.8、-7、-2 
5  9  9 

41. B→D→A→C 42.C、、D 43.



44. A(-3), OA =3 B(4), OB =4 C(3.2), OC =3.2 D( - 2 ), 


OD = 2  E( 0 ), OE =0 

貳、 進階題

一、 填充題

1. 九月的某一天,臺南的氣溫是 35 度,臺北的氣溫是 37 度,阿

里山的氣溫比臺北低 18 度,則阿里山的氣溫比臺南的氣溫高

___________度(如果答案是高 2 度記成+2,低 2 度記成-2)』


2. 某商人以每公斤 60 元的價格買進花生 30 公斤,又以每公斤 200

元的價格買進腰果 8 公斤。如果將這些花生與腰果混合後,再

以每公斤 110 元的價格全部賣出,試問此商人賺或賠多少

元? 。

3. 如下圖,若將 AE 等分成四分,三個等分點分別為 B、C、D,且

數線上的 A 點表示 9,D 點表示 21,則 E 點表示的數是______。

A  B  C  D  E

9  21 

4. 如果以上午 12 時為準,上午 10 時用-2 時表示,則下午 5 時

可用 時表示

5. 數線上 A、B 兩點,已知 A 點所表示的數為-6,B 點所表示的

數為 29,將 A、B 分為 5 等分,問由左起第二個等分點所表示

的數為

6. 以下午 3 時為準,若下午 6 時以-9 時表示,則上午 10 時可表

示為 
7. 如圖,AB=BC=CD=DE,則原點較靠近哪一點?

解答:

1.16 2.賺 780 元 3.24 4.+5 5.8 6.+15 7.C


二、 綜合、題組題

4  5  3 
1. 0.5、 -2 、26、0、 1  、 - 、 - 、52、 -0.02 、3.02、 -501 中,
5  3 13

(1) 其中為正整數的有哪些?正小數的有哪些?正分數的有哪些?

(2)其中為負整數的有哪些?負小數的有哪些?負分數的有哪些?

2. 若以體重 50 公斤為基準,伯仁的體重 58 公斤,記為 +8,則:

(1) 仲仁的體重 47 公斤,記為___________。

(2) 如果伯仁、仲仁和季仁三人平均體重記為 +2,則這三個人平均

體重___________公斤。

(3) 季仁的體重為___________公斤。

3. 某次段考全班同學的平均分數是 62 分,甲、乙、丙、丁、戊五

位同學所得的分數與平均分數的比較如下表:

甲 乙 丙 丁 戊

+4 -8 +12 +3 +1

甲生代表比平均分數多 4 分。求:

(1) 丁生的實際分數是___________分。

(2) 這五位同學中,最高分與最低分相差___________分。

(3) 這五位同學的平均分數是___________。

4. 完成以下表格:

(1) -7 與 25 兩個數同時減去___________後成為相反數。
(2) 2 與 -10 兩個數同時加上___________後成為相反數。

(3) 19 與 -7 兩個數同時加上___________後成為相反數。

5. 如圖,C、D、E 將 AB 四等分,則

C=【 】、D=【 】,E=【 】。

6. 如圖,A、B 互為相反數,若 B(x-3),則 x=【 】。

7. 如圖,各點間的距離相等,則 D 點是【 】。

8. A(-4)在數線上與 A 點相距 4 個單位長的數是【 】。

9. 某整數的絕對值不大於 8,某整數的所有可能和為多少?

10. 若 0.01-0.1=x,則 x 的相反數=?

11. 甲、乙兩人分別在數線上表示 -25 ,95 的位置上,同時相向而

行。若甲的速率是乙速率的 2 倍,則:

(1)最初甲、乙兩人相距___________個單位長。

(2)相遇時,乙走了___________個單位長。

(3)甲、乙兩人在數線上表示___________的位置上相遇。

12. 在數線上一單位代表 4 公分,A 點與原點相距 124 公分,則 A

點所表示的數是【 】。
解答:


1. (1)正數:0.5、26、 1  、52、3.02

正整數:26、52

正小數:0.5、3.02


正分數: 1 

5  3 
(2)負數: -2 、 - 、 - 、 -0.02 、 -501
3 13

負整數: -2 、 -501

負小數: -0.02

5  3 
負分數: - 、 -
3 13

2.(1) -3 (2)52(3)51 3.(1)65(2)20(3)64.4

4. (1) 9(2) 4(3)  -6 5. -6.175;-6.15;-6.125 6.8



7. D( 7  ) 8.0 或-8 9.0 10.0.09 11.(1) 120 (2)40(3)55

12. ±31
參、 挑戰題

一、 填充題

1. 以原數線上表示-8 的點為新原點,原單位長的 2 倍為新單位

長,畫出新數線。求原數線上表示 16 的點,在新數線上表示那

一個數 ?

二、 綜合、題組題

1. 數線上有 A 、 B 、 C 三點,其所表示的數分別是 3、8、10.5,

(1) 若以 A 點為新原點,則 C 點所表示的數為___________。

(2) 若以 A 點為新原點,原單位長之一半為新單位長,則 B 點所表示

的數為___________, C 點所表示的數為___________。

(3) 若以 B 點為新原點, B 、 C 兩點間的距離為新單位長,若 D 點所

表示的數是 4,則 D 點在原數線上所表示的數為___________。

2. 已知數線上兩點 P(p)和 Q(q)且 p 比 q 小 6。若 R、S 兩點

分別是 P 點和 Q 點的對稱點,而且 R、S 兩點中左邊的點坐標

是 7,求 P、Q、R、S 四點的坐標。

3. 所有的整數中,倒數等於它自己本身的數有哪些?

4. 已知| x | = 3,| y | = 5,試求 x+y 的所有可能值為哪些?

5. 法律上規定年滿 17 歲且體重達 45 公斤以上(含 45 公斤)即可捐

血,若小豪、小儒 、小銓、小庭都滿 17 歲且小銓的體重恰好


為 45 公斤,而小豪的體重比小銓輕,但小銓的體重比小庭輕,

小庭的體重又比小儒輕,請問(1)何者的體重最重 。

(2)小豪能不能去捐血 。

解答:

填充題

1.16

題組題

1.(1) 7.5 (2) 10;15(3) 18 2. P(-13)、Q(-7)、R(13)、S(7)

3. 1、-1,共 2 個 4. 2、-2、8、-8 (共四個)

5. (1)小儒(2)不能
主題一 數與數線

1-2 數的大小與絕對值

壹、 基本題

一、 選擇題

1. ( )已知數線上有相異兩點 A 與 B,其中 A 在原點的左邊,B

在原點的右邊,請問下列敘述何者正確?

(A)A 與 B 兩點所表示的數均為正數

(B)A 與 B 兩點所表示的數均為負數

(C)B 點所表示的數大於 A 點所表示的數

(D)A 點所表示的數大於 B 點所表示的數


1  5 9 
­ 
2. ( )已知數線上有 A、B、C、D 四點,分別表示 2 、  3 、 4 、 
1
-
2 ,請問這四點在數線上由左而右排列的順序為?

(A)A B C D (B)B D A C

(C)A C D B (D)D B A C

3. ( )下列何數的絕對值比 5 小?(A)-3 (B)-6 (C)-5 (D)5

4. ( )若 a £ 10 ,則 a 不可能是下列哪一個數字?

(A) - 4 (B) - 11 (C) 10  (D) 0  。

5. ( )數線上有 A (- 1) 、 B (3 ) 、 C (- 5) 、 A (4 ) 四點,請問:哪一個

點離原點最近? (A)A (B)B (C)C (D)D 。


6. ( )若數線上 A 點所表示的數為甲,且甲數的絕對值是乙,

求甲+乙=?

(A)0 (B)-2 (C)-4 (D)-1.5

7. ( )請問│-3│、-│5│、-│-7│、-(-4)的相反數

中,共有幾個負數?

(A)1 (B)2 (C)3 (D)4

8. ( )-(-a)代表什麼意思?

(A)a 的絕對值 (B)a 的倒數

(C)-a 的相反數 (D)a 的相反數

9. ( )下面關於數的大小何者錯誤?
1  1 
(A)8>3 (B)-5 3 >-3 5 
(C)7.4>2.8 (D)│-5│>-5

10. ( )設甲數為整數,且 0<│甲│<3,請問甲數有幾個?

(A)2 個 (B)3 個 (C)4 個 (D)無限多個

11. ( )假設絕對值小於 10 的整數有 a 個,則 a 是多少?

(A)9 (B)19 (C)29 (D)39



12. ( )若 a 為整數,且∣a∣> 5 ,則下列何者不可能為 a 的值?
(A)-2 (B)1 (C)3 (D)4

13. ( )下圖中 O 是原點。請問下列哪一個點所表示的數,其「絕


A  B  O C  D 
對值」最大?

(A) A (B) B (C) C (D) D

14. ( )在一數線上有甲、乙、丙、丁四點,分別表示—50、2、—

1、30,何者到原點的距離最遠?(A)甲(B)乙 C)丙(D)丁。

15. ( )下列何者可以表示數線上-5 與 8 兩點的距離?

(A) 3 (B) - 5 + 8 

(C) - 5 - 8  (D) 8 - 5 

解答:

1.C 2.B 3.A 4.B 5.A 6.A 7.B 8.C 9.B 10.C

11.B 12.B 13.A 14.A 15.C 


二、 填充題

1. 計算 9 - 12 - (- 4 ) =

2  1 
2. 計算 2  - 1  =
3  3 

3. 計算 - 2 - 7  =

4. 若 a = 7 ,則 a = ___________。

5. |-16 |+|-9|-|-6|= 。

6. 8 ﹣(﹣5) +  - 7 =

6 - 13  + ( -16 ) - - 3 - 12 


7.  =

8. │(-8)-(-12)│-│34-(-6)│=

9. -(-5)的相反數是

解答:


1.1 2.  3.9 4. ±7 5.19 6.20 7.-24 8.-36 9.-5

三、 綜合、題組題

1. 請寫出下列各數的相反數。

(A)5 (B)-2

2  12
(C)  (D) -
5  7 

2. 請畫一條數線,並在數線上標示出和原點相距為 3 的點。

3. 請寫出絕對值小於 6 的整數。

4. 求下列各算式的數值:

(1) - 6  =

(2) -  9 - 4  =

(3) 5 - 8  + 4 - 10  =

(4) 2 . 3 - 5 . 8  - 5 . 8 - 2 . 3  =

5. (1)若∣甲數∣=4,則甲數=?
(2)若乙數與丙數互為相反數,則∣乙數∣–∣丙數∣=?
(3)若∣丁數∣=-5,則丁數=?
4  2 
6.  -2.8 、0.2、  -1  、 -0.8 、  - ,以上哪些數大於 -1 呢?
7 5

7. 在下列的空格中填入「>」
、「=」或「<」

(1)  -6 ______ -5 。

(2) 2______ -2.5 。

(3)  -6 ______  -5 。

(4)  2  ______  -2.5 。


8. 試比較下列各數的大小關係: 

- 411 、 360 、 - 212 、 - - 435 、 251 


4  13 
-1 
9. 分別求出 7 、  5 、  4  、 -3.1 的絕對值
甲 =4.5 
10. 若  ,則甲數是多少?

11. 在數線上標出絕對值小於 5 的所有整數點。

­10­9 ­8 ­7 ­6 ­5 ­4 ­3 ­2 ­1 0  1  2  3  4  5  6  7  8  9 10

12. 在數線上標出絕對值等於 5 的所有整數點。

­10­9 ­8 ­7 ­6 ­5 ­4 ­3 ­2 ­1 0  1  2  3  4  5  6  7  8  9 10 

1  1
-1 
13. 在數線上標出 2  、 -3 、  3 、 - -2 、 -3 ,並比較大小。

­4  ­3  ­2  ­1  0  1  2  3  4 

14. 試比較下列各數的大小關係: 

- 411 、 360 、 - 212 、 - - 435 、 251 

15. 如圖所示,將 A(-4)、B(-2)之間等分成 6 分,再把 C(4)、D(6)

之間等分成 4 分,P 點、Q 點均在等分點上,則

• P、C 兩點的距離為 (1) 單位長。


‚ P 點所表示的數為 (2) 。
ƒ Q 點所表示的數的絕對值為 (3) 。 
„ R 點在此數線上,且與 P 點的距離為 19 單位長,則 R 點所表示

的數為 (4) 。

A  B  C  P  D 
Q
-4  -2  0  2  4  6 

16. 在下列的空格中填入「>」
、「=」或「<」

• |951-753| -9。

‚ -|8-11| 2.7。

17. 請列出絕對值小於 6 的負整數

18. 比 - 5. 6 大而比 - 2. 6 小的整數有哪些?

19. 寫出絕對值小於 3 的整數 


解答:

2 12 
1. (A)-5 (B)2 (C) - (D)  2. 3 與-3
5  7 

3.  ± 5 、 ± 4 、 ± 3 、 ± 2 、 ± 1 、0 4.(1)6 (2)-5 (3)9 (4)0


4  2 
5.(1) ± 4 (2)0(3)無解 6. 0.2、  -1  、 -0.8 、 -
7 5

7. (1) <(2) >(3) >(4) < 8.  360 > 251 > - 212  > -411 > - - 435 
4 4  13 13  甲=4.5 或
9.  7 = 7 、  -1 = 1  、  = 、  -3.1 = 3.1 10. 
5 5 4 4 甲=-4.5 

­10­9 ­8 ­7 ­6 ­5 ­4 ­3 ­2 ­1 0  1  2  3  4  5  6  7  8  9 10 

11.

­10­9 ­8 ­7 ­6 ­5 ­4 ­3 ­2 ­1 0  1  2  3  4  5  6  7  8  9 10 

12.

­4  ­3  ­2  ­1 


1  0 1 1  2  3  4 
- 3 - - 2 - 1  - 3 1 1 
13.  2 3 -3 >  -1  >  > - -2 > -3
3 2 

14,  360 > 251 > - 212  > -411 > - - 435 

15.(1)1 (2)5 (3)3 (4)24 或-14

16.>、< 17. -5,-4,-3,-2,-1 18. -3,-4,-5

19. 2,-2,1,-1,0 
貳、 進階題

一、 選擇題

1. ( ) 若|a|+|b|=5,則 b 不可能是下列何數?

(A) 0 (B) 5 (C)  -5 (D) 6

a >b
2. ( )若  ,則下列敘述何者正確?

(A)  a > b (B) 若 a、b 為負數,則 a > b

(C) 若 a、b 為正數,則 a > b

(D) 若 a>0、b<0,則 a + b < 0 

3. ( )甲<0<乙且|甲|>|乙|,甲+乙=a,下列敘述何

者正確?

(A) a 必為正數 (B)a 必為負數

(C)a=0

(D)無法判定 a 是正數或是負數。

4. ( )設|甲+6|+|乙-5|=0,則甲+乙=?

(A)11 (B)1 (C)-11 (D)-1 。

5. ( )下列算式何者正確?

(A) 0> - 100

(B)-5>4

(C)-10.7<  - 1. 07 
(D) - 10 < 10 

6. ( )若甲數是正數,乙數是負數,且|甲數|=|乙數|,則甲

數+乙數的和

(A)正數 (B)0 (C)負數 (D)不一定

A  = 3
7. ( )已知 A 與 B 為數線上相異的兩點,  ,A 和 B 相距 1

單位,請問 B 所代表的數不可能為下列何值?

(A)-4 (B)4 (C)2 (D)1

8. ( )已知數線上有相異兩點 A 及 B,其與原點距離相同,請

問下列敘述何者錯誤?

(A) A 點一定位於 B 點的左邊。

(B) A、B 兩點所表示的數之絕對值相等

(C) A、B 兩點所表示的數互為相反數

(D) 若 A 和原點的距離是 a,A、B 兩點的距離為 2a。

9. ( )數線上兩點 A、B,已知 A 點所表示的數為 a,B 點所表

示的數為 b,a 在 b 的左邊,則下列敘述何者正確?

(A)∣a∣>∣b∣ˉ(B)∣b∣>∣a∣ˉ

(C) a>bˉ (D) b>a

解答:

1.D 2.C 3.B 4.D 5.C 6.B 7.D 8.A 9.D


二、 填充題

1. 若 A - 4  + B + 5  = 0 ,請求出 A+B=

a - 2 + 4 ´ b - 3  = 0 


2. 設 a,b 都是整數,且  ,求 a= b=

p  + 3 ´ q + 2  = 5 
3. 設 p,q 均為整數,且  ,求 p= q=

4. 數線上 A、B 兩點分別表示-2,10,C 點與 A、B 的距離相等,

求 C 點表示的數是

5. 在數線上A、B兩點,A表示的數是6,A、B相距 10 單位長,

則B所表示的數是

6. 在數線上 A、B、C 三點表示的數為-1、3、7,若以B為新原

點,單位長不變,則 A 點代表

7. 數線上 A、B 兩點,分別表示-1、5,若將 AB 六等分,則最左

邊的等分點所代表的數為

8. 設│a+5│+│b-4│=0,則 a= ,b=

9. 若│甲數│= 5,則甲數 =

10. 數線上 A、B 兩點的距離 9,已知 A 點表示 6,則 B 點表示的數

- -3
11.  - (-3 )  (填上 < 或 = 或 > 符號)。

12. |甲數|-2=5,則甲數=
解答:

1.-1 2.2、3 3.p= ± 5或 ± 2 、q=-1、-2、-3 4.4

5. 16,-4 6.-4 7.0 8.a= -5 ,b= 4 9.5 or -5

10. 15 or -3 11.> 12.7 or -7


三、 綜合、題組題

1. 請依據下圖回答問題。

jA 點的坐標為__。B 點的坐標為__。C 點的坐標為__。D 點的

坐標為__。

‚請問 A 點到 B 點的距離為______;D 點到 C 點的距離為_______。

ƒ若 A 點到 B 點的距離記作  A - B  ,請問  A - B  =__;  A - D  =__; 

B - A =__。

„  A - 0 =__;此符號是表示 A 點到__的距離。

…  A  =__;  B  =__; C  =__;  D  =__

2. 整數中有多少個數的絕對值比 5 小?

3. (1) -5 3 的相反數為 (2)-(-4.8) 的相反數為
4. 如果 a 的相反數為 0,b 的絕對值為 9,則 a-b=______

5. 若 x 為正整數,

(1) 絕對值小於 x 的整數有 15 個,則 x = ___________。

(2) 絕對值小於 x 的正整數有 15 個,則 x = ___________。

(3) 絕對值小於或等於 x 的整數有 15 個,則 x = ___________。

(4) 絕對值不大於 x 的正整數有 15 個,則 x = ___________。

6. 如圖,請將 a、b、c、d 四個數的「絕對值」按大小排出順序。


7. 如果哈特利從健一家往東走到十八尖山,共走了 3 公里,同時,

獅子丸從健一家往西走到南寮漁港,共走了 2 公里。若以 500

公尺為一個單位長的長度畫出數線,請問:

(1)十八尖山與健一家距離 單位長。

(2)若以健一家為基準點,向東的方向為正向,則十八尖山在數線

上位置應記為 ,南寮漁港在數線上位置應記為 。

(3)十八尖山與南寮漁港相距 個單位長。

(4)若哈特利與獅子丸分別自十八尖山、南寮漁港同時在數線上以

相同速度往回走,請問他們會在數線上那個位置相遇? 。

8. 若 a > 0, b < 0 

(1) a  =  (答案選: a 、 - a )


(2) b  =  (答案選: b 、 - b )

(3) a - b  0 (填入 > 或 = 或 <)

(4) b - a  0 (填入 > 或 = 或 <)

(5) a - b  b - a  (填入 > 或 = 或 <)


解答:

1.jA(-5);B(-1);C(3);D(5)‚4;2ƒ4;10;4„5;原點

…5;1;3;5 2.9 3. (1) 5  (2) -4.8

4. 如果 a 的相反數為 0,b 的絕對值為 9,則 a-b=__ ± _9___

5.(1) 8 (2)16 (3)7 (4)15 6.d>a>c>b

7.(1)6 (2)+6,-4 (3)10 (4)1(健一家東方 500 公尺處)

8. (1) a (2) - b (3) a - b >0(4) b - a <0(5) a - b  = b - a 


參、 挑戰題

一、 選擇題

1. ( ) 數線上 A、B 兩點表示的數互為相反數,已知 A、B 兩點

的距離是 8 個單位長,這兩個數相乘的絕對值為多少?

(A)8 (B) 16 (C) 0 (D)-16

2. ( )滿足 0 |x|<10 的整數 x 共有幾個?

(A)10 (B)11 (C)19 (D)21 。

3. ( )設 a、b 均表為負數,若 a>b,則下列敘述者正確?

(A) a>|b|

(B)|a|>|b|

(C)|a|=|b|

(D)|a|<|b|

解答:

1.B 2.C 3.D


二、 填充題

1. 小敏在數線上原點的位置玩丟骰子遊戲(1 顆),如果出現奇

數,小敏向左移動 2 單位,如果出現偶數,則小敏向右移動 3

單位,共丟了 10 次,出現偶數 3 次,求小敏最後位置在數線上

代表 的地方。

2. 甲為整數,絕對值小於|甲|的整數有 199 個,甲=

A + 5 + 3 - B  = 0 
3.  , 則 A+B =

A - B 
4. A、B互為相反數且  =15 , 若A>B, 則A=

5. 若甲數為整數,且甲數的絕對值小於 4 而大於|-2|,則甲數

解答:

1. -5 2. ±99 3.-2 4. 7.5 5.3,-3


三、 綜合、題組題

1.  (    )任何整數的絕對值必為正數。

2. 在□中填入>、<或=。

(1)(-3) □ 2

(2)0 □ (-1)

(3)-(-5) □ 3

(4)︱-6︱□ (-6)

(5)3 □︱-4︱

(6)(4 的相反數) □ 4

(7)︱-8︱□ 8

(8)(-3) □ (-5)

解答:

1.(×)  0 =0 2. (1) < (2) > (3) > (4) > (5) <

(6) < (7) = (8) >


主題一 數與數線
1-3 整數的加減運算

壹、 基本題

一、 選擇題

1.( )計算 3 + (- 2 ) ´ 4 - 3 = ? 

(A)1 (B)-8 (C)5 (D)27 。

2.( )某人從司令台的西方 65 公尺,走直線到司令台的東方 35 公

尺處,則他一共走了多少公尺?(A)3 公尺ˉ(B)6 公尺 (C)30 公尺

ˉ(D)100 公尺

3.( )(-12)+(-3)的值加上哪一數後會變成-(-9)?

(A)6 (B)18 (C)24 (D)6

4.( )計算(26-│(-24)-(-13)│)-│7-10│+│(-5)-12

│之值為多少?

(A)29 (B)15 (C)5 (D)-5

5.( )小新計算 13+(-5)+(-16)+27 的式子如下:

步驟一:原式=(13-5)+(-16)+27=8+(-16)+27

步驟二:原式=-(16-8)+27=(-8)+27

步驟三:原式=-(8+27)=-35

則下列敘述何者正確?
(A)步驟一就錯了 (B)步驟二發生錯誤

(C)步驟三不對 (D)小新的算式完全正確

6.( )請判斷下列四個算式中,哪一個算式左右兩邊等號是不成立

的?

(A)(-57)+(-93)=(-93)+(-57)

(B)[(-12)+(-7)]+10=(-12)+[(-7)+10]

(C)(42-93)-85=85-(42-93)

(D)(-14)+[39+(-45)]=[(-14)+39]+(-45)=[(-14)+39]

-45

7.( )比-9 小-5 的數是多少?(A)-14 (B)14 (C)-4 (D)4

8.( )若-5 與(甲數+2)互為相反數,則甲數的相反數為何?(A)

-3 (B)3 (C)-4 (D)4

9.( )關於整數的加減法計算,下列哪一個選項是正確的?

(A)-5-3=-(5-3)

(B)-4+1=-(4+1)

(C)(-2)-(-5)=-(2+5)

(D)(-1)+(-4)=-(1+4)

10.( )5+(-3)=2,下列有關算式中「+」
、「-」號的描述,何

者正確?
(A) 「+」和「-」都是運算符號

(B) 「+」和「-」都是性質符號

(C) 「+」是運算符號,「-」是性質符號

(D) 「+」是性質符號,「-」是運算符號

11.( )下列哪一選項的答案與「 (-11)-4」相同?

(A) 11-4 (B) 4-11 (C) (-11)+(-4) (D) 4-(-11 )

12.( )若零下 2℃以-2 表示,則下列哪一選項的算式可用來表示

「零下 30℃比零下 5℃低 25℃」?

(A) (-30)-5=-25 (B) (-30)-(-5 )=-25

(C) (-30)+(-5)=-25 (D) 5-30=-25

13.( )下列何者可以表示「-(-3)」?

(A) 3 的相反數 (B)-3 的倒數 (C) -3 的相反數 (D) 3 的倒數

14.( )若 a,b 兩數為相反數(a≠0),下列敘述何者「不正確」?

(A)a×b=-1 (B)a+b=0 (C)a×b<0 (D)|a|=|b|

15.( )(-8)+15+(-12) 的運算結果與下列哪一選項的運算結

果不同?

(A)[(-8)+(-12)]+15 (B) 15+[(-8)+(-12 )]

(C)(15-8)+(-12 ) (D)-(8-12)+15

16.( )若甲數=(-5)+(-8)、乙數= (-5)-(-8 )、


丙數= 5+(-8 ),則甲、乙、丙三數的大小關係為何?

(A)甲數>乙數>丙數 (B)甲數>丙數>乙數

(C)乙數>丙數>甲數 (D)乙數>甲數>丙數

17.( )-(-7+22)-8=?

(A) 21 (B) 7 (C) – 7 (D) – 23

18.( )比 (-8) 小 (-2) 的數等於多少?

(A) -10 (B) -6 (C) 6 (D) 10

19.( )75-63-[(- 42)+(- 3)]=?

(A) 57 (B) – 67 (C) – 33 (D) 51

20.( )下面哪一個數減去 4 之後,其絕對值小於 6?

(A) -3 (B) 10 (C) 5 (D) -4

21.( )在數線上,A 點代表 (-11),B 點代表 9,則 AB =?

(A) -2 (B) 2 (C) -20 (D) 20

22.( )設 A、B 兩點分別代表 -4 及 -2,則 AB =?

(A) 6 (B) 2 (C) -2 (D) -6

23.( )∣(-7)-(-2 )∣在數線上代表的意義為何?

(A) -7 到 -2 的距離 (B) 7 到 2 的距離

(C) -7 到 2 的距離 (D) 7 到 -2 的距離

24.( )若∣a∣+∣b∣=10,則 b 不可能是下列何數?


(A) 0 (B) 10 (C) -10 (D) 11

25.( )從 (-8)、(-6)、(-4)、(-2)、3、5、7 中任取三個數

相乘,則最小的乘積為何?

(A) - 336 (B) - 280 (C) - 210 (D) -30

26.( )阿明的爸爸做生意,一至三月平均每月賺了 6 萬 5 仟元,

四至六月平均每月賠了 5 萬 9 仟元,下半年平均每月賠了 7 仟元,請

問:阿明的爸爸全年共賺或賠多少元?

(A)賺 2 萬 4 仟元 (B)賠 2 萬 4 仟元 (C)賺 1 仟元 (D)賠 1 仟元

27.( )如果 a+1 的相反數是 1,那麼 a =?

(A) -2 (B) -1 (C) +1 (D) 0

28.( )若|甲-2|+|乙 +3|=0,則甲-乙 =?

(A) 5 (B) -1 (C) -5 (D) 1

29.( )若甲+乙=0,則甲的相反數+乙的相反數 =?

(A) 2 倍甲 (B) 2 倍乙 (C) 0 (D) 無法決定


30.( )數線上 A(-9)
、B(0)、C(-4) 、D(3)四點,則下列
何者最長? (A) AB (B) AC (C) AD (D) CD 。
31.( )下列何者的值最大

(A)2000+(-394) (B)—(2000+394)

(C)(-2000)+(—394) (D)394-(—2000)。
32.( )石門水庫季節性洩洪,若每天上午八時正水位調降 6 公寸,

則前天上午八時正水位比今天上午八時正水位

(A)低 12 公寸(B)低 18 公寸(C)高 12 公寸(D)高 18 公寸。

解答:

1.B 2.D 3.C 4.A 5.C 6.C 7.C 8.B 9.D 10.C

11.C 12.B 13.C 14.A 15.D 16.C 17.D 18.B 19.A 20.C

21.D 22.B 23.A 24.D 25.A 26.B 27.A 28.A 29.C 30.C

31.D 32.C
二、 填充題

1. 計算 (-2) + (-4) + (-6) + 8 + 9 =

2. 計算 (-2) + (-4) + (-6) + (6) + (4) + (2) =

3. 某日全球最低氣溫是-12℃,而當天新竹市的氣溫是 16℃,則當

天新竹市的氣溫比全球最低氣溫高出 度?

4. 數線上有隻金龜蟲在某一點上,先向左移 4 單位,再向右移 6 單

位,又向左移 3 單位,最後停在表示 13 的點上,則此隻金龜蟲最初

位置表示的數是

5. 計算 3 - (-19) + (2 - 4) + (5 - 3) =

6. 計算(–41)+(–29)–(–70)=?

7.計算 37–12+15–(–10)=?

8.計算 7-{
[(–9)–8]+10}+18–∣(–13)–(–6)∣=?

9. 計算 1+(-2)+3+(-4)+ … +99+(-100)=?

10. 2-3+4-5+6-7+8-9+…+18-19+20=?

11. 計算 (- 14 ) - (- 3 + 9 ) =

12. (-989994)+[(-5897898)+989992]=ˉˉˉˉ。

13. 0 的相反數是 ;–  的相反數是 。

14. 11+(-111)+1111+(-11111)=

15. 9+99+999+10000-1=
16. (-5)-7=(-5)+甲數,則甲數=

17. 某公司營運第一年賠 600 萬,第二年賠 50 萬,第三年賺 560 萬,

則三年來共(賺或賠) 元。

18. 有一隻螞蟻停在數線上點 A(-5) ,它先向右走 7 個單位,再向

左走 8 個單位,再向左走 5 個單位,最後停在 B 點,則 B 點所代表的

數是 。

19. 在數線上,A 點所代表的數為 13,B 點與 A 點的距離為 4,則 B

點所代表的數為 。

20. 數線上一點 A 所表示的數為 a,a=5,在 A 點左邊 14 個單位長

的一點為 B 點,B 代表的數為 b,若 C 點是在 B 點的右邊 3 個單位長,

C 代表的數為 c,

則 a+b-c = 。
21. 假如有一個裝了水的量筒,倒水進去或倒水出來,水位都會改
變,水位升高的數量用正數來表示,水位下降的數量用負數來表示。
如果第一次水位下降 8 公分,第二次水位上升 3 公升,則兩次水位的
總改變是 公分。
22. 從 20.5 °C 的平地到零下 5 °C 的高山,氣溫上升或下降了多少度?
答: 。

23. 甲數是 – 38 的相反數,乙數是∣– 75∣的相反數,則甲數+

乙數是
24. 小明玩打靶遊戲,打中 1 隻小鳥得到 1 分,打中一隻老鷹則倒扣
1 分。若小男今打中 17 隻老鷹,22 隻小鳥,則他的總得分為
分。
25. 7 +{[( – 5 ) – 8]+ ( – 3 )}+ 17 –∣( – 15 ) – ( –

6 )∣=( )

26. 如下圖,數線上的 A 點表示 5,E 點表示 33,若將 A 點與 E 點之

間等分成四

份,三個等分點為 B、C、D,則 B 點表示的數是多少? 。

27. 數線上有 A、B 兩點, AB = 7,A 點所代表的數為 a,若∣a ∣= 6,

且 A 點在原點的左邊,則 B 點所代表的數為 。

28. 數線上 A、B 兩點所代表的數為 2.1 與 6.3,將 A、B 兩點之間等

分為七份,則得到六個等分點,其中左邊數來第二個等分點所代表的

數為 。

29. 若 ( 甲數 + 5 ) 為負數,則 ( –5 – 甲數 ) 0。( 填 >,

<或= )

30. 小猪的蛋糕店第一個月賺了 25000 元,第二個月賠了 43000 元,

第三個月賺了 10000 元,請問這三個總共賺或賠 元?

31. 若|甲數|=7,乙數=-5,則甲數-乙數=【 】。

32. 請求出下列算式的值:
80 - 70 + ( -60 ) =

33. 請求出下列算式的值:
( - 40 ) - 16 - ( -55 ) =

34. 計算 .-(-16 )-[ (-8) -|-36|] 。

35.  5  + ( - 18 ) =

36.  - 27 + ( - 9 )  - 3 =

37. (﹣11 ) ﹣15 + 24 + (﹣33) =

38. (﹣13 + 21) ﹣[ 27 ﹣(15﹣24)] =

39. (﹣7) + (﹣11 的相反數)=

40. (-3410)+1982+(-1989)=
41. 85+(-53)+(-89)=
42. (-39)-(-50) =

43. 20-99+1=

44. 最大的負整數與最小的正整數之和為

45. (-63)-[(-3)-(-5)]=

46. 計算|2-7|-|5-(-5)|+|2-9|=
解答:

1.5 2.0 3.28 4.14 5.20 6.0 7.50 8.3 9.-50 10.11

11.-20 12.-5897898 13. 0,  14.-10100 15.11106 16.-7

17.賠 900,000 元 18.-11 19.9 or17 20.2 21.下降 1

22.下降 25.5 °C 23.-37 24.5 25.-1 26.12 27.1or-13

28.3.3 29.> 30.賠 8000 元 31.12 或-2 32.-50 33.-1

34.60 35.-13 36.-39 37.-35 38.-27 39. 4 40.-3417

41.-57 42.-11 43.-78 44.0 45.-65 46.2


三、 綜合、題組題

1. 小豪負責記錄冰櫃溫度變化,溫度上升時以+號表示,溫度下降

時以-號表示,例如早上溫度比清晨溫度上升 4 度,記錄為+4,下

午溫度比早上高 2 度記成+2。下表為星期一到星期四溫度調整記錄

表,試回答以下問題:

早上 下午

星期一 +3 -1

星期二 -2 -4

星期三 +1 +2

星期四 -3 +4

(1)請問星期一冰櫃的溫度變化為多少度?

(2)請問星期一冰櫃的溫度變化為多少度?

(3)請問星期一冰櫃的溫度變化為多少度?

(4)請問星期一冰櫃的溫度變化為多少度?

2. 請用數線圖解下列運算結果:

(1) (-1)+(6)

(2) (1)+(2)

(3) (4)+(-3)

(4) (-1)+(-3)
3. 小安六月份零用錢為 400 元,買了文具用品花費 200 元,七月份

同樣有零用錢 400 元,領了獎學金 1000 元之後,請客花了 300 元,

請問小安六、七月份合計剩或欠多少元?

4. 數線上有一輛小汽車,放在坐標-5 上,先向右行駛 6 單位,再

向左行駛 7 單位。若向右記為+,向左記為-,請回答下列問題:

(1)請用數線表示小汽車的位置變化。

(2)請用數學式子列出小汽車的位置變化。

(3)請問小汽車最後在那一個坐標位置?

5. 試回答下列問題:

(1)比(-2)大 8 的數為多少?

(2)比 10 小 8 的數為多少?

(3)比(-5)大(-2)的數為多少?

(4)比(-5)小(-2)的數為多少?

6. 小明在某次段考中,英語考了 80 分,若其他科分數與英語相差分

數如下表,試回答下列問題:

國文 數學 社會 生物 健教

與英語科相 +10 -4 +12 -2 -1

差之分數
(1)小明的國文考了多少分?

(2)小明的數學考了多少分?

(3)最高分和最低分相差多少分?


7. 求所有絕對值小於 4  的所有整數和。

8. 一數線上有一點 A,其坐標為 A(4),請回答下列問題:

(1) 位於 A 點右方 5 單位的坐標為何?

(2) 位於 A 點左方 6 單位的坐標為何?

9. a=(-5)-2、 b=-∣-3∣+2、

c=4-(-1)、 d=(-2)+6 -5。

A、B、C、D 四個點分別代表 a、b、c、d。請問那一個點離原點最遠?

10. 某人作生意,第一個月賠了 7850 元,第二個月賠了 9870 元,第

三個月又賺了 4320 元,請問這三個月他總共賺或賠多少元?(如果

賺錢,答案請用正數表示,如果賠錢,答案請用負數表示。)

11.(1)計算 [(-5)-7]-[(-8)-(-11)]= 。

(2)計算 7-77-777-7777= 。

(3)計算(-1)+11+(-111)+1111+(-11111)= 。

(4)計算 1+(-2)+3+(-4)+…+99+(-100)= 。
12. (1)求 [(-867)+962]+(-961)= 。
(2)求 [(-12345)+23456]-(-12345)= 。
13.
(1)(-3)和 37 同時加上 時,則二數會變成相反數。
(2)設(-12)與 6 同時減去一數 b 後,恰好為相反數,則 b= 。

3  5 
14. (1) 甲為整數,滿足 | -2  | <甲< | 9  | 的甲數有哪些?
8  6 
(2) 承上題,其中最大整數和最小整數的差為多少?
15. 某工廠訂定一固定目標生產量為標準,但實際生產量有時增有時

減,下表為一週的生產情形:

星期 一 二 三 四 五 六 日

實際生產

量比目標

生產量的 +4 +15 -8 -4 +2 -9 -5

增減情形

(單位:台)

(1) 這一週的總產量比目標總產量多或少幾台?

(2) 若這週的總產量是 415 台,求每天的目標生產量為幾台?

(3) 星期四的產量是幾台?

(4) 本週內每日實際生產量最高和最低者相差幾台?

16. 求下列各式的值:

(1)(-7)+(-93)=【 】。

(2)(-19)+(-71)=【 】。

17. 求下列各式的值:

(1)(-42)+(-31)=【 】。
(2)(-69)+(-31)=【 】。

18. 求下列各式的值:

(1)(-22)+(-15)=【 】。

(2)(-93)+(-21)=【 】。

19. 求下列各式的值:

(1)(-17)+25=【 】。

(2)(-30)+48=【 】。

20. 求下列各式的值:

(1) 11+(-43)=【 】

(2) 8+(-70)=【 】。

21. 求下列各式的值:

(1)(-7)+97=【 】。

(2) 48+(-48)=【 】

22. 計算(-119)+50+19 的值

23. 計算 30+(-8)+(-22)的值

24. 計算(-87)+10+87 的值

25. 計算(-141)+567+(-259)的值

26. 計算 1234+(-8)+(-1234)的值

27. 計算 9931+10+(-31)+(-11)的值
28. 計算 65+(-83)+35+83 的值

29. 計算(-87)+(-8)+(-13)的值

30. 某日合歡山上早晨的氣溫為-3℃,中午時氣溫上升到 2℃,請

問中午的氣溫比早晨的氣溫高多少度?

31. 某日阿里山上早晨的氣溫為-3℃,下午時氣溫上升到 5℃,請

問下午的氣溫比早晨的氣溫高多少度?

32. 請填入適當的數:

(1) 15-21=15+【 】=【 】


(2) 40-(-40)=40+【 】=【 】


33. 請填入適當的數:

(1)(-28)-12=(-28)+【 】=【 】。

(2)(-71)-(-31)=(-71)+【 】=【 】。

34. 請填入適當的數:

(1)(-52)-(-62)=(-52)+【 】=【 】

(2)(-79)-2=(-9)+【 】=【 】。

35. 請填入適當的數:

(1)(-32)-(-12)=(-32)+【 】=【 】。

(2)(-2)-9=(-2)+【 】=【 】。

36. 求出下列各式的值:
(1) 25-48=【 】。

(2) 15-(-35)=【 】。

37. 求出下列各式的值:

(1)(-9)-9=【 】。

(2)(-3)-1=【 】。

38. 求出下列各式的值:

(1)(-4)-(-10)=【 】。

(2) 5-(-5)=【 】

39. 求出下列各式的值:

(1)(-11)-11+30=【 】。

(2) 63-(-5)+(-5)=【 】

40. 求出下列各式的值:

(1)(-21)+32-11=【 】。

(2) (-4)-42+(-4)=【 】

41. 求出下列各式的值:

(1)(-16)-(-34)-5=【 】。

(2)|(-6)-7|+(-13)=【 】。

42. 求出下列各式的值:

(1) 5-20+(-100)
(2)(-30)+50+300

43. 求出下列各式的值:

(1) 36+(-4)+(-22)

(2)(-7)-2-(-43)

44. 求出下列各式的值:

(1)(-219)+50+19

(2) 35-(-5)+(-19)

45. 數線上 A(5)、B(-8)、C(-12)、D(20)四點,

則 AB =【 】, AC =【 】, 

BC =【 】, CD =【 】

46. 數線上 A (- 10 ) 、 B ( 9 ) 、 C (- 3 ) 三點,試求 AB 、 AC 、 BC 的長度

分別是多少?

47. 在下列空格填上適當的數:
• (-16)+7=-( -7)= 。
‚ (-42)-77=(-42)+ =

48. ( ) 不為 0 的整數相加,所得的和一定是正數或負數。

49. a = -(6 + 7 ) , b = (- 6) + 7 , c = 6 + (- 7 ) 。比較 a 、 b 、 c 三數的大小

50. (1)求 [( - 9 ) - 11 ] + [ 7 - ( -13 )] =

(2) 求 - 36 - - 12  + - 16  =


解答:

1.(1)+2 度 (2)-6 度 (3)+3 度 (4)+1 度 2.略 3.剩 1300 元

4. (1)

(2) (-5) + (+6) + (-7) (3)-6 5.(1)6 (2)2 (3)-7 (4)-3

6.(1)90 分(2)76 分(3)16 分 7.0 8.(1)9 (2)-2 9.A

10. -13400 元 11.(1)-15 (2)-8624 (3)-10101 (4)-50

12. (1)-866 (2)23456 13. -17;-3 14.(1)3,4,5,6,7,8,9

(2)6 15.(1)少 5 台 (2)60 台 (3)60-4=56 台 (4)15-(-9)=24 台

16. (1)-100 (2)-90 17. (1)-73 (2)-100

18. (1)-37 (2)-114 19. (1)8(2)18 20. (1)-32 (2)-62

21. (1)90 (2)0 22.-50 23.0 24.10 25.967 26.-8

27.9899 28.100 29.-108 30.5 31.8 32.-6、80 33.-40、

-40 34.10、-81 35.-20、-11 36.(1) -17 (2) 50


37.(1)-18 (2)-4 38.(1)6 (2)10 39.(1)8 (2)63

40. (1)0 (2)-42 41. (1)13 (2)0 42.(1)-115 (2)320


43. (1)10 (2)34 44. (1)-150 (2)21 

45.  AB =【13】, AC =【-17】
, BC =【  4】, CD =【32】。

46.  AB =  (-10 ) - 9  = - ( 10 + 9 )  = - 19  = 19 

AC =  (-10 ) - ( -3 )  = ( -10 ) + 3  = - 7  = 7 

BC =  9 - ( -3 )  = 9 + 3  = 12  = 12 

47.(1)16、-9(2)-77、-119 48.× 49. b>c>a 50.(1)0(2)40


貳、 進階題

一、 選擇題

1.( )假設甲、乙均為整數,且│甲+1│+│乙-1│=0,則│

甲-乙│-│乙-甲│=?(A)4 (B)2 (C) -2 (D) 0

2.( )若|甲數|+|乙數|=7,那麼甲數可能是下列哪一個數?

(A) 5 (B)-10 (C) 11 (D)-30。

3.( )設甲數是正數,乙數是負數,則下列何者恆為負數?

(A)甲數+乙數 (B)甲數-乙數

(C)乙數-甲數 (D)-(甲數+乙數)

4.( )有一隻蛤蟆在數線上的 A 點,牠先右跳了 5 個單位後,再向

左跳了 7 個單位,後再向右跳了 9 個單位停在 B 點,B 點所表示的數

為 2,則 A 點所表示的數為(A)-5 (B)-2 (C)3 (D)6 。

5.( )下列敘述何者正確?

(A)(甲數-乙數)-丙數=甲數-(乙數-丙數)

(B)絕對值最小的正數是 1

(C)甲數-乙數=甲數+(乙數的相反數)

(D)0 是自然數

解答:

1.D 2.A 3.C 4.A 5.C


二、 填充題

1.  - 89 + ( -99 ) + 97 + ( -98 ) + 89 - ( -199 )  =

2. (-989994)+[(-5897898)+989992]=
3. 以學校為準,在學校東方 9 公里處以+3 表示時,在學校西方 12

公里處應以 表示

4. 設甲數比-10 大 4,乙數比-6 小-2,則甲數+乙數=

5. 3939889+(-12345)+(-3939889)=

6. [( - 34987695 ) + 899931 ] + ( -899930 ) =

7. 7 - {[( -4 ) - 5 ] + ( -6 ) } + 10 - ( -12 ) - ( -4 )  =

解答:

1.99 2.-5897900 3.-18 4.-14 5.-12345

6. -34987694 7.24
三、 綜合、題組題

1. 小狸和三個好友一起平分 52 個巧克力球,分完後發現其中一人多

拿 3 顆,另一人少拿 1 顆,另一人多拿 1 顆,請問小狸拿了幾顆巧克

力球?

2. 求 1+(-2)+3+4+(-5)+6+7+(-8)+9+10+(-11)+12

之值。

3. 若 甲+4=0,乙=-(-3),丙+(-5)=0,把甲、乙、丙按大小順

序排列為 。

4. ( ) 數線上相異兩點 A(a)、B(b),其中 a、b 為整數且 a<b,


那麼 A、B 兩點的距離等於(-a)+b。

解答:

1.10 2.43 3. 丙、乙、甲 4.○


參、 挑戰題

一、 選擇題

二、 填充題

1. 4-7+10-13+16-19+…‥+64 =

解答:

1.34

三、 題組題

1.如圖所示,a、b、c、d、e、f 分別為-19、-16、-8、-2、6、

9 六個數中的某一數,而且使得 a+b+c=a+(-5)+f=b+(-5)

+e=c+(-5)+d=d+e+f,則 b+(-5)+e= 。

a  b  c 

-5 

d  e  f

解答:

1.-15 
主題一 數與數線

1-4 整數的乘除與四則運算

壹、 基本題

一、 選擇題

1.( )甲、乙、丙都是負數,若甲數×(-4)=乙數×(-5)=丙數×

(-6),則下列何者正確?

(A)甲>乙>丙 (B)丙>乙>甲

(C)甲>丙>乙 (D)乙>甲>丙
2.( )觀察下列(甲)到(己)的算式,正確的共有多少個?
(甲)(-8)+(-2)+9=-8+[(-2)+9]
(乙)(-40) ´ (-7+12)=(-40) ´ (-7)+40 ´ 12
(丙)90 ¸ [(-3)-38]=90 ¸ (-3)-90 ¸ 38
(丁) (-1 ) 5  =-5
(戊)(-985)×(-400) ´ 5=5 ´ (-400) ´ (-985)
(己) (-2 ) 4  =-(2 ´ 2 ´ 2 ´ 2)
(A)1個 (B)2個 (C)3個 (D)4個
3.( )下列各式中,何者的值為1?
(A)(-1) ´ (-1 ) 2  (B) (-1 ) 2  ´ (-1 ) 3  ´ (-1 ) 3 
(C) (-1 ) 2  ´ 1 ´ (-1 ) 4  ´ (-1 ) 5  (D) -1 2 ´ (-1 ) 2 
4.( )請問 ( -3 2 ) -(-3) ´ [28-3 ´ (-4)] ¸ (-4)之值為何?
(A)60 (B)27 (C)-27 (D)-39
5.( )計算∣4 ´ (5-8)∣-(-3) ´ [ (-10 ) 3  ¸ 50-25]- ( -1 4 ) =?
(A)-107 (B)-122 (C)-124 (D)-146
6.( )下列哪一個算式的值為正整數?

(A)(-3)+(-3)+(-3)+(-3) (B)(-3) ¸ (-3) ´ (-3)

(C) - (-3 ) 3  (D) ( -3 4 ) 


7( )若A= 12345 2  -2345 ´ 12345,則下列有關A的敘述何者正確?
(A)A是一個八位數 (B)A最後四位皆為0
(C)A是一個奇數 (D)A是一個質數
8.( )媽媽帶了 400 元上市場買做水餃的材料,已知絞肉一斤 80

元,高麗菜一斤 35 元,水餃皮一斤 40 元,若趙先生想買 2 斤絞

肉、4 斤高麗菜及 3 斤半水餃皮,則媽媽所帶的錢是不夠或是剩

餘多少元?

(A)剩餘 20 元 (B)剩餘 40 元 (C)不夠 20 元 (D)不夠 40 元

3  4 
9.( ) (-5  ) ¸ ( - ) 的值與下列何者相等?
4  5 
3  4  23 4  23 5  4  23 
(A) - (5  ¸ ) (B)  ¸ (C) - ´ (D) (- ) ¸ ( - ) 
4  5  4  5  4  4  5  4 
2 3  4 
10.( )2.若 甲 ´ = 乙 ´ = 丙 ´ = -1 ,則下列何者正確?
13  16  19 

(A)甲>乙>丙 (B)乙>甲>丙 (C)丙>甲>乙 (D)丙>乙>甲

48  3 1  5 
11.( )計算 - ¸ [  ¸ ( - ) 2 ´ ( -2 ) 2 ] ´ ( - ) =?
5  2  2  2 

(A)1 (B)-1 (C)16 (D)-16

1  1  1  1  1 
12.( ) 1´ ( - ) ¸ ´ ( - ) ¸ ´ ( - ) =甲,代表甲數的點會落在數線上
2  4  6  8  10 

哪一段?

(A)a (B)b (C)c (D)d

1  1  1  1 
13.( )化簡 (1-  )÷(1-  )÷(1-  )÷……÷(1-  )=?
2  3  4  100 
1  1 
(A)  (B) 25 (C)  (D) 99
25  100 
9  7  7 
14( ) (-4 . 3 ) ¸ ( - ) ¸ 1 . 6 =甲, 1. 6 ¸ ( -2 . 1 ) ¸ ( - ) =乙, (-4 . 3 ) ¸ ( - ) ¸ ( -2 . 1 ) 
5  3  3 
7  9 
=丙, (- ) ¸ 1 . 6 ¸ ( - ) =丁,請問下列那一個數最小?
3  5 

(A)甲 (B)乙 (C)丙 (D)丁

4  4 4  1  1 
15.( )若 (-  ) ´ a = 1 ,則a=? (A) - (B)  (C) 2  (D) - 2
9  9  9  4  4 
1  1 3 
(1 + ) ¸ (  - 1 ) ´
16.( )求  3  3  8 之值為何?
3 3 1 16
- - - -
(A)  4 
(B)  8  (C)  3  (D)  3 
4  1  3 
-  ¸ 1 . 75 + 6 ¸ ( - ) =
17.( )計算 7  4  4  ?
3  4  4  3 
-8 -8 -7 -7
(A)  7  (B)  7  (C)  7  (D)  7 
1 1  25 25 
18.( )計算 (3 - 18 ) ¸ (  + ) =? (A)-18 (B)18 (C) - (D) 
2  3  2  2 
7 3 
19.( )  ¸ ( -3  ) 與下列哪一個算式相等?
3  5 
7 3  7 3 
(A)  ¸ ( -3 + )  (B)  ¸ ( -3 - ) 
3  5  3  5 
7 3  1  3 
(C)  ¸ ( -3 ) ¸ (D) 7 - 3 ´ ( - ) ´
3  5  3  5 

20.( )8 5  ÷2 1 的值等於下列哪一個算式的值?


7  5 

(A)(8÷2)+( 5  ÷ 1 ) (B)(8 5  ÷2)+ (8 5  ÷ 1 )


7  5  7  7  5 

(C)(8 5  ÷2)+ 1  (D)(8÷2 1 )+( 5  ÷2 1 


7  5  5  7  5 

1  1  1  1 
21.( )a= 1 + 1  +  1  +  +  ,b= 1 - 1  +  1  -  +  ,
3  9  27  81  243  3  9  27  81  243 
121  61  61  121 
則b÷a之值為? (A)  (B)  (C)  (D) 
243  243  121  61 

22.( )某人想利用慢跑來減肥,已知平均每慢跑 20 分鐘可以消耗

80 大卡的熱量,此人吃了 2  包的餅乾,已知一包餅乾的熱量為

522 大卡,試問他至少需要慢跑幾分鐘才能把他所吃下的熱量
消耗掉? (A) 81 分鐘 (B) 83 分鐘

(C) 85 分鐘 (D) 87 分鐘

23.( )假設W、X、Y、Z代表由1、2、3、4、5、6、7、8、9等九個

W Y 
數中選出的四個不同的數字,試求  + 的最小值為何?
X  Z 
2  3  17  25 
(A)  (B)  (C)  (D) 
17  17  72  72 
3  7 
24.( )如果 a 代表的是  的倒數與  的倒數和,而 b 代表的是 
2  4 
3 7 
( + ) 的倒數,試問 a 與 b 何數比較大?
2  4 

(A) a>b (B) a<b

(C) a=b (D) 無法判斷


25.( ) 3  的倒數是哪一個數?

11  2  5  17 
(A)  (B)  (C)  (D) 
2  11  17  5 
11 8  ö 33 
26.( ) æç ´ ÷ ¸ =?
è 17  27  ø 34 
81  4  16  9 
(A)  (B)  (C)  (D) 
16  9  81  4 
3  10  13 
27.( ) 2 ´ ¸ =?
5  3  9 
338  5  9 
(A) 6 (B)  (C)  3  (D) 
27  13  5 
5 2 ´ 7 2  ´ 11  5 ´ 7 ´ 13 
28.( )  ÷  2 =?
2 ´ 3 2  ´ 6  2  ´ 3 
125  39  117  385 
(A)  (B)  (C)  (D) 
39  125  385  117 
3  9 
29.( )池中插著一根竹竿,  露出水面,露出水面的部分長  公
4  8 

尺,則原長為多少公尺?
27  32  3  2 
(A)  (B)  (C)  (D) 
32  27  2  3 
3  2  5 
30.( )6  ×  ÷  =a,則 a 的倒數為何?
7  3  2 
12  7  75  7 
(A)  (B)  (C)  (D) 
7  12  7  75 
11  2 
31.( )甲、乙兩生計算  ÷2  的過程如下:
21  3 
11  2  11  3  11  7  11 
甲生:  ÷2  =  ×2  =  ×  = 
21  3  21  2  21  2  6 
11  2  11  8  11  3  11 
乙生:  ÷2  =  ÷  =  ×  = 
21  3  21  3  21  8  56 

請問甲、乙兩生的計算,何者正確?

(A) 甲(B) 乙(C) 兩人都錯(D) 無法判斷

1  5  3 
32.( )求 (  ) 4  ÷(  ) 2  ÷(  ) 3  之值為何?
2  3  2 
1  1 
(A) 150 (B)  (C)  (D) 75
150  75 
1  3  4  5  10 
33.( )31. 若甲數=  ÷  ÷  ÷  ÷  ,則甲數的倒數為為何?
18  16  15  12  9 
12  5  16  3 
(A)  (B)  (C)  (D) 
5  12  3  16 

34.( )已知-  ×甲=1,試求甲=?
13 
7  2  2  7 
(A)  (B)  (C) -  (D) - 
3  13  13  3 
3  3 
35.( )□÷ 3  =  ,則□應填入何數?
4  2 
2  5  2  45 
(A)  (B)  (C)  (D) 
5  2  3  8 
4  3 
36.( )計算 16÷43÷[  ×  ] 2  之值為何?
3  2 
1  1 
(A) 16 (B) 4 (C)  (D) 
16  4 

37.( )下列的算式從哪一步開始錯誤?
5 2  5 5 
- ÷ ( - )
3  3  6  4 
3 5 5 
(A) =  ¸ (  - )
3  6  4 
3 5  3 5 
(B) =  ¸ -  ¸
3  6  3  4 
6  4 
(C) =1×  -1× 
5  5 

(D) = 


38.( )下列各算式的計算結果,何者與 8÷  ÷2 不相等?

1  1  1  1  1 
(A) 8×3×  (B) 8÷  ×  (C) 8÷(  ×2) (D)8÷(  ÷2)
2  3  2  3  3 
6  3  4 
39.( )  ×[48÷8÷  ]÷  =?
5  5  3 

(A) 12 (B) 9 (C) 16 (D) 18

40.( )如果 a  = 2  =  c  = 4  ,則a+b+c=?


2  b  10  8 

(A) 8 (B) 10 (C) 12 (D) 14

1  7  3  4 
41.( )甲數的  等於乙數的  ,乙數的  等於丙數的  ,若丙數為
3  8  4  5 

45,則甲數=?

(A) 124 (B) 126 (C) 128 (D) 130

4  3 
42.( )16÷43÷[  ×  ] 2  =?
3  2 
1  1 
(A) 16 (B) 4 (C)  (D) 
16  4 

43. ( )下列敘述何者一定正確?(其中 a、b、c 都代表數)

(A) a-b=b-a (B) (a-b)-c=a-(b-c)

(C) a×b×c=a×c×b (D) (a÷b)÷c=a÷(b÷c)。


44. ( )下列各式何者錯誤?(其中 a、b、p 都代表數,且 a≠b、p

≠0)

(A) p×( a-b)=p×a-p×b (B) ( a-b)×p=a×p-b×p

(C) p÷( a-b)=p÷a-p÷b (D) ( a-b)÷p=a÷p-b÷p。

解答:

1.B 2.C 3.B 4.D 5.B 6.C 7.B 8.D 9.B 10.B

11.D 12.C 13.B 14.B 15.D 16.A 17.C 18.A 19.B 20.D

21.C 22.D 23.D 24.A 25.A 26.C 27.A 28.A 29.C 30.B

31.A 32.A 33.A 34.D 35.D 36.C 37.A 38.D 39.B 40.B

41.B 42.C 43.C 44.C


二、 填充題

1. [12 +∣(– 6 ) × 3∣÷ ( – 5 ) – ( – 13 )]÷ ( – 7 )

2. 根據科學理論,從海平面開始每往上每升高 100 公尺,氣溫就下

降 0.6℃。小美準備和友人登山,已知當日海平面的溫度是 20℃,當

小美抵達海拔 3400 公尺的高度時,該地的溫度是 度。

3. 計算 (-3) + (-3) + (-3) + (-3) + (-3) + (-3) + (-3) + (-3) + (-3) + (-3)

4. (-81)÷(-9)+5×(-4)=

5. 38+(-5)×[35-(-4)×(-5)]÷3=?

6. [24+∣(-3)×8∣÷(-6)-(-12)]÷(-2)=?

7. 9999×5689=?

8. 1234×758+1234×255-1234×13=?

9. 29×(-45)-55×29+90×(-29)=?
|- 5 +( -8 |-|-
)  5 -( -8 |

10. 計算 = 。
|-5 -8  |+|-5 +8 

26 2 
11. 若 a ´  = - ,則 a= 。
35  7 
5  5 
12. ¸ ( - ) - ( -6 ) ´ 2 =
9  9 
3  5  2 
13. ( - 2  ) ´ ¸ ( - ) =
5  4  7 
7  5  2  3 
14. ( -  ) ¸ + 1  ´ =
6  3  3  5 
4  4  2  4 
15. ( - 4 . 5 ) ´ - ¸ + =
9  9  11  9 

16. 請求出下列算式的值: (- 9 ) ´ 23 - 60 =


17. 請求出下列算式的值:
( - 12 ) ´ ( 35 - 20 ) =

18. 請求出下列算式的值:
( - 60 ) + ( -32 ) ¸ ( -4 ) =

19. 2×(-23)×5×(-8)×25= 。

20. (-12)÷[(-3)×4]= 。

21. 4×4-3×3×3= 。

22. 965×579+965×421=

23. 138-(-5)×[-10+3×(-4)]= 。

24. (- 15) ¸ (- 3 ) 的值為

25.  - 32 的值為 。

26. (- 49 ) ´ 7359 - 2641 ´ 49 = ?

27. (-5)´( -125)´2´(-4) ´(-1)=________。

解答:

1.-1 2.0.4 3.-30 4.-11 5.13 6.-16 7.56884311

5  5
8.1234000 9.-5510 10.  11.  - 12.11 13.91/8
8  13 

14.3/10 15.23 16.-267 17.-180 18.-6 19.46000 20.1

21.-9 22.965000 23.28 24.5 25. - 9 26. -490000

27.5000
三、 綜合、題組題

1. 計算下列各式:

(1)  (-16) ´ 9

(2)  (-16) ´ (-9)

(3)  16 ´ (-9)

2. 計算下列各式:

(1)  (-16) ¸ 4

(2)  72 ¸ (-9)

(3)  (-64) ¸ (-8)

(4)  27 ¸ (-3)

3. 計算下列各式:

(1) (-2)×5+8

(2) 2×5-8

4. 古老師班上規定,全學期的綜合表現成績將以八十分為基準,以

平時表現加減分評定。平時表現加減分是以「優點」和「缺點」計算,

每得一個「優點」可加 1 分,每得一個「缺點」將扣 1.5 分,請幫老

師結算下列學生的綜合表現成績。
姓名 優點 缺點 綜合表現成績

趙小中 5 2

錢小華 4 3

孫小民 1 2

李小國 4 4

5.計算下列各式:

• 12 + (-2) ´ [21 + (-3) ´ (-3)] =


‚ (­14) ´ 2­7[9­(­15) (­3)] =
6.計算下列各式:

•3 ´ (-8+3)+(-2) ´ (-6)+(92-8) ¸ (2-5) =

‚ {[(-6) + (-2)] ¸ (5 - 9)} ´ 3


7.計算下列各式:

•432 ´ 69-432 ´ 86+432 ´ 17=

‚76 ´ (-118)+76 ´ 195+(-77) ´ 76=

8.計算下列各式:

(1) (-2) ´ 5-8

(2) 8-(-2) ´ 5

9.計算下列各式:

(1)|12-10| ´ 3 ´ (-7)

(2)(-2) ´ (-3) ´ (-4) ´ (-5)


10. 計算下列各式:

(1)(-3) ´ 5 ¸ (-7)

(2)3 ´ 5 ¸ (-7)

(3)(-3) ´ (-5) ¸ 7

(4)3 ´ (-5) ¸ (-7)

11. 計算下列各式:

(1)15×8= 。

(2)(-15)×8= 。

(3)15×(-8)= 。

(4)(-15)×(-8)= 。

(5)(-15)×0= 。

(6)0×(-8)= 。

12. 求出下列各式的值:
• 5 ´ 7 =
‚ ( - 6 ) ´ ( -8 ) =

13. 求出下列各式的值:
• 23 ´ 43 =
‚ ( - 35 ) ´ ( -24 ) =

14. 求出下列各式的值:

• (- 6 ) ´ 9 =
‚ 3 ´ ( -7 ) =

15. 求出下列各式的值:

• ( - 63 ) ´ 14 =

‚ 31 ´ ( -46 ) =

16. 求出下列各式的值:
• 1 ´ 9 =
‚ 1 ´ ( -5 ) =

ƒ 1 ´ 95 =
„ 1 ´ ( -54 ) =

17. 求出下列各式的值:
• 8 ´1 =
‚ ( - 4 ) ´ 1 =

ƒ 34 ´1 =
„ ( - 42 ) ´ 1 =

18. 求出下列各式的值:

• ( - 1 ) ´ 9 =

‚ ( - 1 ) ´ ( -5 ) =

ƒ 8 ´ ( -1 ) =

„ ( - 4 ) ´ ( -1 ) =

19. 求出下列各式的值:
• 0 ´12 =
‚ 0 ´ ( -11 ) =
20. 求出下列各式的值:
• 14 ´ 0 =
‚ ( - 21 ) ´ 0 =

21. 求出下列各式的值:

• ( - 2 ) ´ ( -3 ) ´ 4 =

‚ ( - 4 ) ´ 5 ´ ( -6 ) =

22. 求出下列各式的值:

• ( - 12 ) ´ ( -32 ) ´ 8 =

‚ ( - 42 ) ´ 4 ´ ( -16 ) =

23. 求出下列各式的值:

• ( - 2 ) ´ 4 ´ 6 =

‚ ( - 3 ) ´ ( -5 ) ´ ( -7 ) =

24. 求出下列各式的值:

• ( - 22 ) ´ 36 ´ 6 =

‚ ( - 3 ) ´ ( -25 ) ´ ( -17 ) =

25. 求出下列各式的值:

• ( - 3 ) ´ ( -4 ) ´ 0 ´ 15 =

‚ ( - 5 ) ´ 0 ´ ( -5 ) ´ ( -5 ) =

26. 求出下列各式的值:

• (- 93 ) ´ ( -415 ) ´ 0 ´ 150 =


‚ ( - 53 ) ´ 0 ´ ( -53 ) ´ ( -53 ) =

27. 計算下列各式的值,並比較•、‚兩式的結果是否相等。

• [( - 4 ) ´ 8 ] ´ ( -20 ) =

‚ ( - 4 ) ´ [ 8 ´ ( -20 )] =

28. 求出下列各式的值:
• 19 ´ 5 ´ 20 =
‚ ( - 39 ) ´ 4 ´ ( -25 ) =

29. 求出下列各式的值:
• 5 ´ 23 ´ 20 =
‚ ( - 125 ) ´ 73 ´ 8 =

30. 求出下列各式的值:
• 40 ¸ 5 =
‚ ( - 40 ) ¸ ( -5 ) =

31. 求出下列各式的值:

• ( - 20 ) ¸ 4 =

‚ 20 ¸ ( -4 ) =

32. 求出下列各式的值:
• 0 ¸ 25 =
‚ 0 ¸ ( -41 ) =

33. 求出下列各式的值,並比較•、‚兩式的結果是否相等。

• 8 ´ [( -5 ) + 3 ] =
‚ 8 ´ ( -5 ) + 8 ´ 3 =

34. 求出下列各式的值,並比較•、‚兩式的結果是否相等。

• [( - 16 ) + 5 ] ´ 4 =

‚ ( - 16 ) ´ 4 + 5 ´ 4 =

35. 請求出下列各式的值:

• 3 2  + ( -2 ) 2  =

‚ 6 3  - ( -4 ) 2  =

ƒ ( - 3 ) 3  + ( -5 ) 2  =

36. 連連看(其中 a、b、c 都代表數)

a+b=b+a ● ●加法的結合律

(a+b)+c=a+(b+c) ● ●乘法的結合律

a×b=b×a ● ●加法的交換律

(a×b)×c=a×(b×c) ● ●乘法的交換律

37. 若 a、b、c 都代表不為 0 的數,請在下列各空格中填入適當的運

算規則(加法的結合律、乘法的結合律、加法的交換律、乘法的交換

律):

(a+b)+c = (a+b)+c

= a+(b+c) ---- 。

= a+(c+b) ---- 。
= (a+c)+b ---- 。

:(其中 a、b、c 都為任意


38. 請在下列各空格中填入「=」或「≠」

數)

(1) (a+b)-c a+(b-c)

(2) (a-b)-c a-(b-c)

(3) (a×b)÷c a×(b÷c)

(4) (a÷b)÷c a÷(b÷c)

39. 請在下列各空格中填入「+」或「-」,其中甲、乙、丙、丁為

四個任意數:

(1) 甲數+(乙數+丙數)=甲數 乙數 丙數

(2) 甲數+(乙數-丙數)=甲數 乙數 丙數

(3) 甲數-(乙數+丙數)=甲數 乙數 丙數

(4) 甲數-(乙數-丙數)=甲數 乙數 丙數

(5) 甲數+(乙數-丙數+丁數)=甲數 乙數 丙數 丁數

(6) 甲數-(乙數-丙數+丁數)=甲數 乙數 丙數 丁數

40. 甲:a+0=a 乙:a-0=a 丙:a × 0=0

丁:a ÷ 0=a 戊:a ÷ 0=0

以上一定正確的有 。(a 代表數)

41. 計算下列各式之值:
(1) 25×13×4= 。

(2) 29×80×125= 。

42. 計算下列各式之值:

(1)360÷12÷3= 。

(2)360÷3÷12= 。

(3)承上,(1)、(2)的答案是否一樣?

(4) 「因為 a÷b÷c=a÷c÷b,所以除法具有交換律」


,這句話是否正確?

(其中 a、b、c 都代表不為 0 的任意數)

43. 計算下列各式之值:

(1)360÷(12÷3)= 。

(2)360×12÷3= 。

(3)360×(12÷3)= 。

(4)100÷15÷3= 。

44. 計算下列各式之值:

(1) 125×(50+80) = 。

(2) 75×(400-4) = 。

(3) 321×419-321×19= 。

(4) 625×73+625×17= 。
解答:

1.(1)-144 (2)144 (3)-144 2.(1)-4 (2)-8 (3)8 (4)-9

3.(1)-2 (2)2 4.82、80、81、81 5.•-48 ‚224 6.•-12 ‚6

7.•0 ‚0 8.(1)-18 (2)18 9. (1)-42 (2)120

15  15  15  15 


10. (1)  (2)  - (3)  (4)  11. (1) 120 (2) -120
7  7 7  7 

(3) -120 (4) 120 (5) 0 (6) 0 12.35、48 13.989、840

14.-54、-23 15.-882、-1426 16.9、-5、95、-54

17.8、-4、34、-42 18.-9、5、-8、4 19.0、0

20.0、0 21.24、120 22.3072、2688 23.-48、-105

24.-4752、-1275 25.0、0 26.0、0 27. 相等

28.1900、3900 29.2300、-73000 30.8、8 31.-5、-5

32.0、0 33.相等 34.相等 35.13、200、-2

36. 連連看(其中 a、b、c 都代表數)

a+b=b+a ● ●加法的結合律

(a+b)+c=a+(b+c) ● ●乘法的結合律

a×b=b×a ● ●加法的交換律

(a×b)×c=a×(b×c) ● ●乘法的交換律

37. (a+b)+c=(a+b)+c
=a+(b+c) ---- 加法的結合律 。

=a+(c+b) ---- 加法的交換律 。

=(a+c)+b ---- 加法的結合律

38. (1) = (2) ≠ (3) = (4) ≠ 39. (1) +,+ (2) +,-

(3) -,- (4) -,+ (5) +,-,+ (6) -,+,-

40.甲、乙、丙 41.(1) 1300 (2) 290000

42.(1)360÷12÷3=30÷3=10 (2)360÷3÷12=120÷12=10

(3)是 (4)不正確

43.(1)360÷(12÷3)=90 (2)360×12÷3=1440

20 
(3)360×(12÷3)=1440 (4)100÷15÷3= 

44. (1) 125×(50+80)=6250+10000=16520

(2) 75×(400-4)=30000-300=29700

(3) 321×419-321×19=321×(419-19)=321×400=128400

(4) 625×63+625×17=625×(63+17)=625×80=50000
貳、 進階題

一、 選擇題

1.( )下列那個選項是錯誤的?

(A) (- 18 ) ¸ (9 - 3 ) = (- 18 ) ¸ 9 + 18 ¸ 3 

(B) (9 - 3 ) ¸ (- 18 ) = 9 ¸ (- 18 ) - 3 ¸ (- 18 ) 

(C) (- 18) ´ (9 - 3 ) = (- 18 ) ´ 9 - 18 ´ (- 3 ) 

(D) (9 - 3 ) ´ (- 18 ) = 9 ´ (- 18 ) + 3 ´ 18 

解答: 1.A
二、 填充題

1. 求出下列算式的值:
15 ´ [( -2 ) + ( -4 )] - 26 ¸ 2 =

2. 求出下列算式的值:
( - 37 ) ´ ( -4 ) ´ 3 ´ 25 =

3. 求 519×71+281×(-65)+519×29+281×(-35)之值。

4. 求︱3×(4-16)︱÷(-3)÷(-3)之值。

5. 求[(-5)-8 × (-13)+17-∣(-15)-(-6)∣]×[(-54321)
+54321]之值。
(
6. 計算 1234567 - 234567 ´ - 4 5  - (- 4 ) 的值為

)  。

7. (- 32 ) ´ 1234 ´ (45679 - 56789 ) ¸ (- 2468 ) ´ (- 625 ) = ?

8. - 2 5  ´ [(- 5 )5  + 10 5 ] = 。

2 2 2
9. (-9 )+(-1234)¸(5-6)+(-5 )´(-10 )´0+1234+(-

2
9) =________。

10. (-9 )+(-16)¸(5-9) ´(-52 )+(-102 )=______

解答:

1.-103 2.11100 3.23800 4.4 5.0 6.1234567

7. 111100000 8. -3100000 9.1234 10.-209


三、 綜合、題組題

1. 求出下列各式的值:
• 576 ¸ 48 =
‚ ( - 703 ) ¸ ( -19 ) =

2. 求出下列各式的值:

• ( - 576 ) ¸ 48 =

‚ 703 ¸ ( -19 ) =

3. 求出下列各式的值:

• 8 ´ ( -3 ) - ( -5 ) =

‚ 36 ¸ ( -4 ) + 6 =

4. 求出下列各式的值:

• 8 ´ ( -3 ) - [( -5 ) ´ 2 + 4 ] =

‚ 36 - [ 12 ¸ ( -4 ) + 6 ] =

5. 求出下列各式的值:

• 105 ´ ( -23 ) =

‚ ( - 47 ) ´ 97 =

6. 求出下列各式的值:

• ( - 137 ) ´ 37 + ( -137 ) ´ 63 =

‚ 44 ´ ( -47 ) + 56 ´ ( -47 ) =

7. 請根據上面的算式,求出下列各式的值:
• 6 3  = 6 ´ 6 ´ 6 =

‚ ( - 2 ) 4 = ( -2 ) ´ ( -2 ) ´ ( -2 ) ´ ( -2 ) =

ƒ 7 2  = 

„ ( - 3 ) 5  =

解答:

1.12、37 2.-12、-37 3.-19、-3 4.-18、30

5.-2415、-4559 6.-13700、-4700 7.216、16、49、-243


參、 挑戰題

一、 選擇題

1. ( )旺來商店以每公斤 60 元的價格買進花生 18 公斤,又以每

公斤 170 元的價格買進腰果 6 公斤,後來花生有 8 公斤壞掉,腰果有

6 公斤送給旺來伯的好朋友,旺來伯將剩下這些花生與腰果混合後,

以每台斤多少元的價格賣出,會使旺來商店賠 1200 元?

(A)50 元ˉ(B)60 元ˉ(C)90 元ˉ(D)54 元 (註:1 台斤=0.6 公斤)

解答: 1.D
二、 填充題

1. 求出下列算式的值:
( - 12 ) ´ [ 13 ´ ( -3 ) + 9 ] - 24 ¸ ( -3 ) =

2. 求出下列算式的值:

54 ¸ ( -6 ) - [ 4 3  ´ ( -2 ) + 8 ] =

3. 求出下列算式的值:
( - 40 ) ¸ 8 + 46 ¸ [ 48 ¸ ( -6 ) - ( -3 ) ´ ( -5 )] =

4. 觀察上面的算式,並求出下列算式的值:
1 + 11 + 101 + 1001 + 10001 =

解答:

1.368 2.111 3.-7 4.11115


三、 綜合、題組題

1. 請求出下圖中,著色部分的面積是多少平方公分。

13.4 

4.3 

6.6 

4.3 

單位:公分

2. 我們知道:6×1388=8328,如果(a×100000+6)×1388=832808328,
試求下列各小題的值:
• a × 1388 × 79 + a × 21 ×1388= 。
‚ (10000a+6)×1388= 。
3. (1)我們知道:(753×752)+(753×752)=2×753×752,
求 2×753×752-753×753-(-752)×(-752)之值。
(2)利用(1)的結果,說明 2×753×752 與 753×753+(-752)×(-752)
的大小關係。

` `

解答:

1.86 2.• 832800000 ‚83288328

3. (1)-1 (2)2×753×752 < 753×753+(-752)×(-752)


主題二 指數律與科學記號
壹、 基本題

一、 選擇題

1. ( ) 2 5 是下列那個式子的簡要寫法?

(A) 2 + 2 + 2 + 2 + 2  (B) 2´ 5  (C) 2 ´ 2 ´ 2 ´ 2 ´ 2  (D) 5´ 5 

2.( )若 甲= 4 3 ,請問下列哪個選項是正確的?

(A)甲=4+4+4=12 (B)甲=4×4×4=64

(C)甲=3+3+3+3=12 (D)甲=3×3×3×3=81
3.( )趙先生喜從天降,中了樂透彩頭獎,個人獨得一億二千四百
三十二萬元,若將趙先生中獎的金額寫成科學記號 a ´ b n  ,則下列何
者正確?
(A)a=12432 (B)n>b (C)n=8 (D)a+b+n>20
4.( )已知一奈米=10 -9 公尺,則2×10 -4 cm 相當於多少奈米?
(A) 20奈米 (B) 200奈米 (C) 2000奈米 (D) 20000奈米

5.( )下面三個數a= 2 . 496 × 10 8 、b=35.47×10  、c=0.1475× 10 9  的
大小關係為何?
(A)a>b>c (B)b>c>a (C)c>a>b (D)c>b>a
6.( )設a=-10 -1 、b=-10 -2 、c=-10 -3 、d=-10 -4 ,則a、b
、c、d四數中最大者為何?
(A)a (B)b (C)c (D)d
7.( )若(6×10 -3  )÷□=1.2×10 -13  ,則□可以科學記號表示為何?
(A) 5×10 9  (B) 5×10 10  (C) 5×10 11  (D) 5×10 12 
8.( )若一個質子的重量是 1.67×10 -27 kg,則 20.04×10 -3 kg 是幾個

質子的重量?

(A)1.2×10 22  個 (B)1.2×10 23  個 (C)1.2×10 24  個 (D)1.2×10 25  個

9.( )已知人類的血管全長大約是九萬六千公里,若將九萬六千這
個數以科學記號寫成 9. 6 ´ 10 n  ,則 n=?

(A)2ˉ(B)3ˉ(C)4ˉ(D)5

10.( ) 若 0.0070008009 可以寫成 7×10 m  +8×10 n  +9× 10 k  ,則 m

+n+k=?(A)-10 (B)-20 (C)-30 (D)-40

11.( ) 若 10 8 ×10 7  ×10 15 =10 n  ,則 n=?

1  1  1 
(A)8+7+15 (B)8×7×15 (C)  +  +  (D)8÷7÷15
8  7  15 

12.( )8 3 ×(125) 3 ×10 -7 ×10 17 ×4×25×7 的計算結果可以科學記號表

為何?

(A)7×10 19  (B)7×10 20  (C)7×10 18  (D)7×10 21 

13.( )下列何者錯誤?

2 3 3 4
(A) (-2 ) >(-2 ) (B) 2 >2

3 3 1 0
(C) (-2 ) <2 (D) (-2 ) <(-2 )

14.( )天堂城發行大樂透,中頭獎的機率為 50 萬分之一,表成科

學記號應為下列哪一個?

(A) 5×10–5 (B) 5×10–6 (C) 2×10–6 (D) 2×10–5

15.( )下列敘述何者錯誤?

(A) 74 = 7 + 7 + 7 + 7

(B) (– 2)3 = ( – 2 ) × ( – 2 ) × ( – 2 )

1
(C) ( – 8 ) ÷ 3 ÷ 2 = ( – 8 ) × 

3 2 13
(D) 27 × 9 = 3

n
16.( )有關科學記號 a × 10 的敘述,下列何者正確?

(A) 1<a (B) 1<a<9 (C) a<10 (D) 1 £ a<10

17.( )若 A = 8,B = 2.4 × 103,則 B 是 A 的多少倍?

(A) 0.3 (B) 3 (C) 30 (D) 300

2
18.( )計算 [- ( -3 ) +3 ] ÷6-4 之值為何﹖

(A) -2 (B) -3 (C) -5 (D) -6

19.( )一天(24 小時)是 a 秒鐘,下列何者是 a 的科學記號表示法?

3 3
(A) 3.6 × 10 秒 (B) 1.4 × 10 秒

4 4
(C) 8.64 × 10 秒 (D) 9.6 × 10 秒

20.( )下列哪一個等式成立?

5
(A) (–7 ) = (–7 ) + (–7 ) + (–7 ) + (–7 ) + (–7 )

(B) (–2 )6 =[ (–2 )3]2 = (–8 )2

(C) (–1 )16 = –16

(D) (–2 )4 = –24

21.( )已知 1 莫耳個相當於 6×1023 個,若有電子 1.5×1024 個相當

於有幾莫耳個電子?

(A) 1.5 莫耳 (B) 2.5 莫耳 (C) 6 莫耳 (D) 4 莫耳

22.( ) 9 ´ 10 10 這個數是幾位數?


(A)9 (B)10 (C)11 (D)12

23.( ) 7. 325 ´ 10 -4  這個數在小數點後的第幾位開始不為 0?

(A)4 (B)5 (C)6 (D)7

24.( )某地區的人口約有「三千萬」人,若表示成科學記號應

為下列何者?

(A)  3´10 3  (B)  3´10 6  (C)  3´10 7  (D)  3´10 10 


25.( ) 10 -5 = ,則 m=?
10 m 

(A)5 (B)50 (C)10 (D)-5


26.( )若 a = 8 ´10 -3 、 b = 5´10 7 ,請以科學記號表示  =?

(A)  1. 6 ´10 -10  (B)  1. 6 ´10 10  (C)  6. 25 ´10 -10  (D)  6. 25 ´10 10 

27.( )利用科學記號將一個很大或很小的數記為 a ´ 10n  時,其中 a 

與 n 的限制條件為何?

(A)0< a <10, n 為整數 (B)0< a <10, n 為正整數

(C)1 £ a <10, n 為整數 (D)1 £ a <10, n 為正整數

28.( )若 10 x  ×10 3 ÷10 5 =10 14 則 x=?

(A) 13 (B) 14 (C) 15 (D) 16

29.( )有一計算機只能夠顯示最多八位數,所以超過八位數的值

,就會以科學記號表示法的形式顯示。例如:輸入35000000+

77000000=就會得到1.12 08,這個結果表示 1. 12 ´ 10 8 ,則:輸入「


68000000+74000000」會顯示什麼結果?

(A) 1.42 07ˉ(B) 1.42 08 (C) 1.42 09ˉ(D) 1.42 10

30.( )下列有關指數律的敘述何者錯誤?

(A)若 a ¹ 0,則 a 0 =1 (B) a m ´n  = (a m ) n 

(C) ( a ´ b ) m  = a m  + b m  (D) ( a m  ´ b n ) p  = a mp  ´ b np 

31.( )7 個(-2)相乘可以用下列那一個算式表示?

(A) 7 ´ ( -2 )  (B) (-2 ) 7  (C)  7 -2  (D) (-2 ) -7 

32.( ) ( - 1 ) 1234  = ?

(A) -1 (B) 1 (C) -1234 (D) 1234

33.( ) (8 ´ 49 ) 3 =  2 a  ´ 7 b  ,求 a+b=?

(A) 15 (B) 8 (C) 18 (D) 9

34.( )下列算式何者正確?

(A)  5 3 = 5 ´ 3  (B)  23  + 2 4  = 2 7 

(C)  4 2 = 4 + 4  (D)  7 3 = 7 ´ 7 ´ 7 

35.( ) a = (-0 . 3 ) 14 、 b = (-0 . 3 ) 15  、 c = (-0 . 3 ) 16 ,下列何者正確?

(A) a>b>c (B) c>b>a (C) c>a>b (D) a>c>b

36.( )下列哪一個數讀作「三百零三億三千零三萬三千三百三十

三」?

(A) 300330330033 (B) 300303333033


(C) 30330003333 (D) 30330033333

37.( )10000000= 10 a  ,則 a=?

(A)6 (B)7 (C)8 (D)9

38.( )0.001= 10 a  ,則 a=?

(A) 2 (B)-2 (C) 3 (D)-3

39.( )關於科學記號的表示法,下列何者正確?

(A)  123000 = 123 ´10 3  (B)  123000 = 12 . 3 ´10 4 

(C)  123000 = 1 . 23 ´10 5  (D)  123000 = 123 ´ 1000 

40.( ) 2. 357 ´10 5  此數末尾共有幾個 0?

(A)1 (B)2 (C)5 (D)8

41.( ) 9 ´10 -4  此數小數點後有幾個 0?

(A) 5 (B) 4 (C) 3 (D) 2

42.( ) 4. 6832 ´10 -8  =a,小數 a 在小數點後第幾位才出現不為 0

的數字?

(A) 7 (B) 8 (C) 4 (D) 12

43.( )下列何者是 3150000 的科學記號表示法?

(A)  315´10 4  (B)  3150´10 3  (C)  31. 5 ´ 10 5  (D)  3. 15 ´10 6 

44.( ) 2´10 5  是 8´10 7 的幾倍?

(A)  2. 5 ´10 -3  (B)  2. 5 ´ 10 -1  (C)  5 ´ 10 -3  (D)  5. 2 ´ 10 -4 
45.( )若 31000 = 3 . 1 ´ 10 a 、 0. 0025 = b ´10 -2 , a ´ b 之值為何?

(A) 7.5 (B) 1 (C) 12.5 (D) 10

46.( )若 a = 3 ´10 -3 、 b = 5´10 8  ,請以科學記號表示 a ´ b =?

(A)  15 ´10 -24  (B)  15´10 5  (C)  1. 5 ´10 11  (D)  1. 5 ´10 6 

47. 下列敘述何者正確?

3  4 
(A)0 的倒數是 0 (B) 1  的倒數為 1 
4  3 
5  8  6  7 
3 3 1 1
(C) æç ö÷ >  æç ö÷ (D) æç ö÷ >  æç ö÷
è 2 ø è 2 ø è 2 ø è 2 ø

解答:

1.C 2.B 3.C 4.C 5.C 6.D 7.B 8.D 9.C 10.B

11.A 12.D 13.B 14.C 15.A 16.D 17.D 18.C 19.C 20.B

21.B 22.C 23.A 24.C 25.A 26.A 27.C 28.D 29.B 30.C

31.B 32.B 33.A 34.D 35.D 36.D 37.B 38.D 39.C 40.B

41.C 42.A 43.D 44.A 45.B 46.D 47.D


二、 填充題

1. 以下是幾個長度單位的換算,為方便使用起見,請換算為公尺,

並使用 10 的次方表示。

(1)1 公里(km)=1000 公尺=_______公尺

(2)1 公分(cm)=0.01 公尺=_______公尺

(3)1 公釐(mm)=0.001 公尺=_______公尺

(4)1 微米(μm)=0.000001 公尺=_______公尺

(5)1 奈米(nm)=0.000000001 公尺=_______公尺

2. 以下是幾個重量單位的換算,為方便使用起見,請換算為公克,

並使用 10 的次方表示。

(1)1 公噸(t)=1000000 公克=_________公克

(2)1 公斤(kg)=1000 公克=_________公克

(3)1 毫克(mg)=0.001 公克=_________公克

3. 下表為國小所學的位名及位值,請將位值用科學記號表示:

位名 千位 百位 十位 個位 十分 百分 千分

位 位 位

位值 1000 100 10 1 0.1 0.01 0.001

十的

次方
4. 光年是一種長度單位,指的是光走一年的距離。若光在真空中的

速度為 1.08 ´109 公里時 ,等於每秒走 公尺。

5. 電腦儲存資料的容量是以位元組(Bytes)為單位,以下是幾個電

腦儲存資料容量的常用單位,請將之換算為位元組並以 2 的次方表

示。

(1)1KB=1024 bytes=________bytes

(2)1MB=1024KB=________bytes

(3)1GB=1024MB=________bytes

6. 電腦儲存資料的容量是以位元組(Bytes)為單位,儲存一個中文

字需要 2 個位元組,小美有一個 128MB 的行動碟,若用以儲存單純沒

有任何格式的中文檔案,最多可存 字(請以科學記號表示)

( 1MB = 2 20 Bytes )
7. 計算 2 ´ (- 5 2 ) - 10 2  =
8.計算 (4 3  - 35 )´ (- 2 )2  =
9. 8 - [- 4 ´ (- 25 ¸ 5 ) - 2 3  + 15 ] =
10. 紅火蟻入侵台灣除了對生態環境中土棲性動物造成傷害,也影響

農作物的成長與收成造成經濟上極大的損失,紅火蟻一個蟻巢中最多

約有 50 萬隻個體,每公頃可形成約 1,000 個蟻巢,試問一公頃的土

地最多有 隻紅火蟻。(請以科學記號表示)

11.(-1)1 + (-1)2 + (-1)3 + (-1)4 + … + (-1)30 =______。


a b c d e
12. 45768 = 4×10 + 5×10 + 7×10 + 6×10 + 8×10 ,求 a + b + c +

d + e =______。

13. 14 億的科學記號表示為 。

14. 請寫出 0.000000000123 的科學記號=

15. 請寫出  200  的科學記號=


0 . 000000005 

16. 請寫出  35 的科學記號=


1 . 4 ´ 10 11 

17. 請寫出 81 ´ 10 12  ¸ 9 ´ 10 12 的科學記號=

15
18. 天文學中,一光年的距離約為 9×10 公尺,若某行星與地球相

距 150 光年,則此距離用科學記號表示應為 公尺。

1  1 
19. 計算 -  5 
´ ( - ) 2  =
2  2 
1  1 
20. 計算 ( -  ) ´ ( -7 ) 3  ´ ( - ) 4  =
7  7 

21. 計算 - 2 2  ´ ( - ) 2  =

1  1  3 
22. 計算 - (  ) 4  ¸ ( - )  =
10  10 

23. 計算 ( 3  ) 2  ¸ ( -4 ) 2  ¸ ( -1 99 ) =

7  1 
24. 計算 ( -  ) ´ 0 . 125 ´ (  3 ) =
8  5 
解答:

1. (1) 10 3  (2)  10-2  (3)  10-3  (4)  10-6  (5)  10-9 

2. (1)  10 6  (2)  10 3  (3)  10-3 

3.
位名 千位 百位 十位 個位 十分 百分 千分
位 位 位
位值 1000 100 10 1 0.1 0.01 0.001
十的次方  1 
10 3  10 2  10 1  10 - 1  10 - 2  10 -3 

4.  3 ´ 108 公尺 秒 5.(1)210 (2)220 (3)230 6.  6.4 ´ 1021 

7.-150 8.116 9.-19 10.  (5 ´10 5 ) ´ ( 1 ´10 3 ) = 5 ´10 8  (隻)

11.0 12.10 13.  1. 4 ´ 10 9  14.  1. 23 ´ 10 -10  15.  4 ´ 10 11 
1  1 
16.  2. 5 ´ 10 -10  17.  9 ´ 10 24  18.  1. 35 ´ 10 18  19.  - 7
=-
2  128 
1  36 1  7
20.  21.  - 22.  23. -169 24.  -
49  25  10  8000 
三、 綜合、題組題

1.計算下列各式:

(1)  38 ¸ 37 

(2)  (-3)5 ¸ 34 

(3)  2 3

(4)  (42 ) 3 

(5)  10-3 

(6)  2-4 

(7)  (1024) 0 

2.請將 100000000000000000 寫成 10 的次方形式。

3. 請用科學記號表示下列各式:

(1)400000000

(2)2100000000

(3)0.000293

(4)0.00000000000112

4. 計算下列各式:

(1) (-1)+(-1) 2  -(-1) 3 

(2) (-1)-(-1 2  ) -(-1 3 )

(3) (-1)× (-1) 2  ×(-1) 3


(4) (-1)÷(-1 2  )÷(-1 3 )

(5)  (-1)10 ¸ (-1)8 ´ (-1)5 

5. 請將以下數值以科學記號表示:

(1)地球的質量約有『6300000000000000000000』公噸

(2)人類 DNA 的大小約有 0.0000000075 公尺

(3)臺灣人口約有二千三百萬人

(4)臺灣島上最遠兩地相隔約四百公里

6. 光年是一種長度單位,指的是光走一年的距離。若光在真空中的

速度為 1.08 ´109 公里時 ,請回答下列問題:

(1) 1 光年=____________公里

(2)已知從 x 星到 y 星相距 14 光年, y 星到 z 星相距 16 光年,如果要

從 x 星到 y 星,再從 y 星到 z 星,共多少公里?

(3)若太空船每小時可走 2.7 ´ 108 公里,要從 x 星到 y 星,再從 y 星到 z 

星,中途不休息需花費多少小時?

7.若 甲= 6´10 3  ,乙= 4 ´10 -2  ,請回答下列各題:

(1)甲×乙=?(本小題的答案請以科學記號表示)

(2)甲÷乙=?(本小題的答案請以科學記號表示)

(3)甲+乙=?

(4)甲-乙=?
8. 若 甲= 12 2 ,乙= 8 2  ,則下列選項是正確的,請打「○」
;錯誤的請

打「×」並給與訂正。

( ) (1)甲-乙= (12 - 8 ) 2 = 4 2  = 16 

訂正:

( ) (2)甲+乙= (12 + 8 ) 2 = 20 2  = 400 

訂正:

2  2 
12 2  æ 12 ö æ 3 ö 9 
( ) (3)甲÷乙= 12 2 ¸ 8 2  = 2 
=ç ÷ =ç ÷ =
8  è 8  ø è 2 ø 4 

訂正:

( ) (4)甲×乙= 12 2 ´ 8 2  == (12 ´ 8 ) 2  = 96 2  = 96 ´ 2 = 192 

訂正:
9. 計算下列各題:
(1) 2 3  ´ 5 3  =
(2) 4 3  ¸ 2 3  =
(3) 5 4  ´ 2 4  ¸10 6  ´ 0 ´ 8 =
(4) 25 ´ 32 =
(5) 16 ´ 125 ´ 3 =

10. 計算 [ ( – 3 )2×( – 2)3 – ( – 4 )×3 + 3 ]×( – 2 ) =?

11.計算下列各式的值:
2 3 2 4
(1) ( 11 × 13 × 17 ) × ( 13 × 17 )。

3 4 3 2 4
(2) ( 3 × 5 × 17 ) ÷ ( 3 × 5 × 17 )。

11 11 11 11 11 □
12. 5 + 5 + 5 + 5 + 5 = 5 ,求□。

13. 比較下列各組數的大小:

(1) 224,413,86。

3 2 2 2 3 2 3 2
(2) 2 × 3 × 5 ,2 × 3 × 5 ,2 × 3 × 5 。

14. 新竹市政府指出:民國 95 年,新竹市每人每日垃圾清運量約為

0.7 公斤,請以此計算新竹市一個月(30 天)會產生多少噸的垃圾?

(新竹市人口數約為 12 萬人)

15. 已知甲、乙兩地的距離是 1600 公里,如果在地圖上量出甲、乙

的距離為 40 公分。試問此「地圖」比真實的地理位置縮小為原來的

幾倍?

16.

1.計算下列各式並將結果表示成科學記號:

(1) 8 × 10–9 + 2.1 × 10–7。

(2) 3 × 104 + 4 × 106。

2.比較下列各題大小

(1) 6.8 × 10–5 和 2.8 × 10–4 哪一個較大?

(2) 4 .6× 105 和 8.6 × 104 哪一個較大?


17. 將一條 64 公分的尼龍繩對摺後剪開,會得到 2 條尼龍繩,再把 2

條尼龍繩對摺後剪開,會得到 4 條尼龍繩,則依此步驟做下去,當對

摺 15 次後剪開,則每一條尼龍繩的長度是多少公分?

2
18. ( 0.1 ) =?

19. 2 × 10 –25 × 5 × 1020 =?

a b 2
20. 已知-243 = -3 、128 = 2 ,則 a – b =?

21. 以科學記號表示下列各數:

(1) 78000 =______。

(2) 30400 =______。

(3) 0.0000357 =______。

(4) 298.73 =______。

22. 將下列各小題用科學記號表示:

(1) 2 × 108 × 3 × 102 =______。

(2) 3 × 10-3 × 9 × 107 =______。

(3) 4 × 10-6 × 8 × 10-2 =______。

23. 已知 a = 3.1 × 103,b = 5.1 × 104,則:

(1) a + b =______。(用科學記號表示)

(2) a × b =______。(用科學記號表示)

24. 在下列 □ 內填入適當的數字:


19 19 □
(1) 2 + 2 = 2 。

13 12 □
(2) 2 – 2 = 2 。

25. 求下列 □ 內的數字:

10 3 
(1)  8 
= 10□。
10 
10 7 
(2)  3 
= 10□。
10 
4 □
(3) 8 = 2 。

26. 判斷下列算式的化簡是否正確,正確的打ˇ,不正確的請訂正

9 3 9÷3 3
(1)5 ÷5 =5 =5

□正確 □訂正:

3
(2)-5 =(-5)×3=-15

□正確 □訂正:

3 3 3 3
(3)5 ×2 =(5×2) =10

□正確 □訂正:

27. 試比較下列各分數的大小

1  1
(甲) ( -2 ) 2 ¸ 3  (乙) ( -2 ) 2 ¸ (丙) 3 ¸ ( -2 ) 2  (丁)  ¸ ( -2 ) 2 
3  3 

28. 計算以下的值,並以科學記號表示結果 
10 10 ¸ ( 4 ´ 10 5 ) 

29. 計算下列的值 

(1 )(  ) 3 


( 2 )( - ) 4 

30. 求下列中 a 的值: 


( 1 ) 3 7 ´ 3 4  = 3 a 
( 2 )( -5 ) 9 ´ ( -5 ) 11  = ( -5 ) a 

31. 求下列中 a 的值: 


( 1 ) 2 15 ¸ 2 12  = 2 a 
( 2 )( -3 ) 4 ¸ ( -3 ) 2  = ( -3 ) a 

32. 求下列中 a 的值: 


( 1 )[( - 4 ) 3 ] 5  = ( -4 ) a 
2  2
( 2 )[(  ) 4 ] 2  = (  ) a 
3  3 

33. 計算下列的值: 
(1 )( 2 ´ 5 ) 3 
(2 ) 2 3  ´ 5 3 

34. 計算下列的值: 

( 1 )(  ) 3 


( 2 )( - ) 3 

35. 以科學記號表示法紀錄下列各數:

(1)90000

(2)2500

(3)73200000

36. 以科學記號表示法紀錄下列各數:

(1)0.00002

(2)0.0000073
(3)0.00918

37. 將 0.0000000002006 用科學記號表示。

38. 將 95600000000 用科學記號表示。

39. 一個中文字在電腦中占 2bytes,若小明的電腦尚硬碟可儲存

300000000000 個中文字,請以科學記號表示小明的電腦硬碟尚有多

少 GB 的容量。

(1GB= 10 9  bytes)

40. 計算 (5. 2 ´ 10 3 )´ (5 ´ 10 8 ) 的值,並以科學記號表示其結果。

41.計算 (8 ´ 10 9  ) ¸ (5 ´ 10 3 ) 的值,並以科學記號表示其結果。

42. 利用我國的命數系統報讀 50023834000,讀作 。

43. 下列哪些是科學記號的表示方式?(請將對的全部選出來)

(A) 93 ´ 10 6  (B) 1. 6 ´ 10 -24  (C) 9. 99 ´ 10 9  (D) 0. 7 ´ 10 23  (E) 3´ 2 10 

44.  10 -2  ´ 10 -5  所代表的數與下列哪些數相同?(請將對的全部選出

來) 。

(A)0.0000007 (B)0.00000001 (C)0.0000001 (D) 
7000000 
1  1 
(E)  (F) 
10000000  1000000 

45.  6 ´ 10 23  個氫原子質量為 1.008 公克,那麼一個氫原子質量平均為

公克。(以科學記號表示)

46. 如果將光速以每秒 3 ´ 10 8 公尺來計算,那麼光一分鐘行進多少公


里?(以科學記號表示)

47. 比較大小:

2 4
(1) (-5) _____(-5) 。

(2) (-7)5 _____(-7)4。

48. 求 32´ ( 1 . 5 ) 5  之值。

49. 求  1  的科學記號為__________.。
2500000 

11 12 13 14 15
50. 1 + 1 + 1 + 1 + 1

51. ( 175 ¸ 52 ) ´ ( 28 ¸ 26 )

52. 求下列各式的值:

(1) 3 2  = 

(2) - 3 2  =

(3) ( - 3 ) 2  =

(4) - ( 3 2 ) =

53. 求下列各式的值:

1
(1) (  ) 2  = 


(2) ( -  ) 3  =

(3) ( - 0 . 2 ) 3  =

(4) - ( 1 . 1 ) 2  =

54. 求下列各式的值:
(1) 3 0  = 

2
(2) (  ) 0  = 

(3) - 7 0  =

(4) ( - 1 . 234 ) 0  =

55. (1) 729 = 3 a  ,求 a 之值為何?

(2) 24 = 2 a ´ 3 b ,求 a+b 之值為何?

56. 求下列各式中□的值:


(1)  5 2 ´ 5 4  = ( 5 ´ 5 ) ´ ( 5 ´ 5 ´ 5 ´ 5 ) = 5 


(2)  ( -2 ) 3 ´ ( -2 ) 1  = [( -2 ) ´ ( -2 ) ´ ( -2 )] ´ ( -2 ) = ( -2 ) 

57. 求下列各式中□的值:

2  2  2  □
(1)  ( - ) 7 ´ ( - ) 4  = ( - )
3  3  3 

(2)  ( 0 . 57 ) 100 ´ ( 0 . 57 ) 200  = ( 0 . 57 ) □

58. 求下列各式中□的值:

7 ´ 7 ´ 7 ´ 7 ´ 7  □


(1)  7 5 ¸ 7 2  = = 7 
7 ´ 7 

3 ´ 3 ´ 3  □
(2)  3 3 ¸ 3 1  = = 3 

59. 求下列各式中□的值:
1 1 1 □
(1)  (  ) 5 ¸ (  ) 2  = (  )
5  5  5 


(2)  ( -0 . 42 ) 3.  ¸ ( -0 . 42 ) 0  = ( -0 . 42 ) 

60. (1) ( 5 ´ 2 ) 3  = ( 5 ´ 2 ) ´ ( 5 ´ 2 ) ´ ( 5 ´ 2 ) = 5 a ´ 2b

求 a+b 之值為何?

a b
(2)  ( 0 . 3 ´ 7 ) 10 = ( 0 . 3 )  ´ 7

求 a+b 之值為何?

61. 已知 a、b 是有理數

(1) 2 5  ´ 3 5  = (a ´ b) 5 = c 5 

求 a+b+c 之值為何?

5 3
(2) (  ) 3  ´ (  ) 3  = (a ´ b) 3 = c 3 =d
3  5 

求 a ´ b + c ´ d 之值為何?

62. 求下列各式中□的值:

(1) ( 3 2 ) 3  =  32  ´ 32  ´ 3 2 = 32 ´3 =3□

1  1
(2)  ((  ) 4 ) 2  = (  ) □
5  5 

(3)  (( 0 . 3 ) 3 ) -4  = ( 0 . 3 ) □

63. 求下列各式中□的值:

(1) ( - 7 ) 3  ´ ( -7 ) 4  ¸ ( -7 ) 2  = (-7)□

(2)  5 9 ¸ 5 2  ´ 5 3  = 5 □


64. 求下列各式中 x 的值:

x
(1)  4 ´ 2 3 ´16 = 2 2  ´ 2 3  ´ 2 4  =2

x
(2)  11 10 = 121 2  ´11 3  ´11 

65. 求下列各式中□的值:

(1) [((-5 ) 3 ) 2  ´ ( -5 ) 4 ] =(-5)□


(2)  ( 3 2 ) 0  ´ ( 3 0 ) 2  = 3 

66. 求 a、b 之值:


(1) - 2 . 3 0  ´ 3 2  ´ ( - ) = a

(2) - 3 3  - ( -3 ) 3  ¸ ( -1 ) 2  + ( -2 ) 1  = b

67. (1) 1000 = 10 ´10 ´10 = 10 a  ,求 a 之值為何?

1  10 b 
(2)  0 . 01 = = = 10 c  ,求 b+c 之值為何?
100  10 2

68. 以下為常用的長度單位換算,請以科學記號表示。

(1) 1 公里= 公尺。

(2) 1 公里= 公分。

(3) 1 公厘= 公尺。

(4) 1 奈米= 公尺。

69. 以下為常用的重量單位換算,請以科學記號表示。

(1) 1 公噸= 公斤。

(2) 1 公克= 公斤。


(3) 1 毫公克= 公克。

(4) 1 毫公克= 公噸。

70. 計算下列各式的值,並以科學記號的形式表示:

(1) 2000 ´ 3210000 =

(2) 8 ´10 2  ´ 5 ´10 -5  =

71. 以科學記號的形式表示下列各數:

(1) 246800000=

(2) 0.00000523=


(3) =
2 ´ 10 5 

(4) 三億四千二百萬=

2  3 
72. 計算 ( -  ) 2  ´ =
3  4 2 

73.計算下列各式:

2  2  2 
(1)  ( - ) 2 ´ ( - ) 6  = ( - ) a ,則 a=_______
3  3  3 
7  7 
(2)  [( -1  ) 4 ] 5  = ( -1  ) b  ,則 b=_______
4  4 
8  11  8  11 
(3)  ( - )8 ´ ( - ) 8  = [( - ) ´ (  )] c  ,則 c=________
9  13  9  13 
3 3 3
74.  (  ) 2 , (  )3  , (  ) 4 的大小關係為   。
2  2  2 
2  3 

75. æç -  3  ö÷ , æç - 3ö
÷ , æç -3ö
÷ ,這三個數從大到小的次序
è 11 ø è 11 
ø è 11 ø

為 。
3  1 
76. 計算(-4)×(-  )2×5-(-4)2×  的值(答案若為分數,請以
10  5 
最簡分數表示)


2  5  2
77. 計算(4-  )÷  ×0.75- æç ö÷ 的值(答案若為分數,請以最簡分數表
3  4  è 3 ø

示)

3 3  4  1 
78. 計算  - ´ 1 . 25 - ( -8 ) ¸ - ( - ) 4 的值(答案若為分數,請以最簡分
4  4  5  2 

數表示)
3

79. 計算 8 ¸ æç - 1 ö÷ 1  æ 1 ö


´ (  2  ) + ç - ÷ ¸ 0 . 625 的值(答案若為分數,請以最簡分
è 2 ø - 2  è 4 ø

數表示)

3  2  3  2 
80. 計算 ( 12  - 1 )  - ( - 1 5 )  ´ ( 5 2 )  ¸ [( 2 3 )  - 1 ] 的值(答案若為分數,請以最

簡分數表示)

解答:

1  1 
1.(1)3 (2)-3 (3)8 (4)4096 (5)  (6)  (7)1
1000  16 

2.  10 17  3. (1)  4 ´ 108  (2)  2.1´ 109  (3)  2.93 ´ 10-4 

(4)  1.12 ´ 10-12  4. (1)1 (2)1 (3)1 (4)-1 (5)-1

5. (1) 6.3 ´ 1021 公噸 (2) 7.5 ´ 10-9 公尺 (3) 2.3 ´ 107 人 (4) 4 ´ 102 公里

6. (1)9460.8 公里 (2)18921.6 公里 (3)80 小時

7. (1) (2)
6 ´ 10 3  ´ 4 ´ 10 -2  (6 ´ 10  ) ¸ (4 ´ 10  )
3  -2 

= 6 ´ 4 ´ 10 3 - 2  6 ´ 10 3  æ 6 ö æ 10 3  ö


= =ç ÷´ç ÷
= 24 ´ 10  4 ´ 10 - 2  è 4 ø çè 10 - 2  ÷ø
= 2 . 4 ´ 10 2  = 1 . 5 ´ 10 3 - ( - 2 )  = 1 . 5 ´ 10 5 

(3) (4) 

6 ´ 10 3 + 4 ´ 10 - 2  6 ´ 10 3 - 4 ´ 10 -2 


= 6000 + 0 . 04  = 6000 - 0 . 04 
= 6000 . 04  = 5999 . 96 

8.(1)× 訂正:甲-乙= 12 2 - 8 2  = 12 ´ 12 - 8 ´ 8 = 144 - 64 = 80 

(2)× 訂正:甲+乙= 12 2 + 8 2  = 12 ´ 12 + 8 ´ 8 = 144 + 64 = 208 

(3)○

(4)× 訂正:甲×乙= 12 2 ´ 8 2  == (12 ´ 8 ) 2  = 96 2  = 96 ´ 96 = 9216 

9. (1)1000(2) 2 3 (3)0(4)800(5)6000 10.114
13 24 6
11. (1) 112  ´13 3  ´17 7  (2) 3´17 2  12.12 13. (1) 4 >2 >8

2 3 2 3 2 2 2 3
(2) 2 × 3 × 5 >2 × 3 × 5 >2 × 3 × 5

14.  (0 . 7 ´120000 ´ 30 ) ¸ 1000 = 2520000 ¸ 1000 = 2520 噸


15.  16. 1.(1) 2.18 × 10–7(2) 4 .03× 106
4000000 

16.2.(1) 2.8 × 10–4>6.8 × 10–5(2) 4 .6× 105>8.6 × 104

1 1 
17.  64 ´ (  )15  = 9 
= 2 -9  18.10-2 19. 10 –4 20.-44
2  2 

21. (1) 7. 8 ´10 4  (2) 3. 04 ´10 4  (3) 3. 57 ´10 -5  (4) 2. 9873 ´10 2 

22. (1) 6´10 10  (2) 2. 7 ´10 5  (3) 3. 2 ´10 -2 


23. (1) 5.41 × 104 (2) 1.581 × 108 24.(1)20 (2)12

25. (1)□=-5。(2)□=4。(3)□=12。

26. (1)錯誤 應為 59÷53=59-3=56

(2)錯誤 應為-53=-5×5×5=-125

(3)正確 53×23=(5×2)3=103

4 3  3 3 3 27 
27. 乙>甲>丙>丁 28.2.5×10 29.  ( 1 )(  ) 3 = (  ) ´ (  ) ´ (  ) =
4  4  4  4  64 
3  3  3  3  3  3 3  3  3  81 
( 2 )( - ) 4 = ( - ) ´ ( - ) ´ ( - ) ´ ( - ) = -(  ´ ´ ´ ) = -
2  2  2  2  2  2  2  2  2  16 

30.(1)10 (2)20 31.  ( 1 ) 2 15 ¸ 2 12  = 2 15 - 12  = 2 3 , \ a = 3 

( 2 )( -3 ) 4 ¸ ( -3 ) 2  = ( -3 ) 4 - 2  = ( -3 ) 2 , \ a = 2  32.(1)15 (2)8

4  4 4  4  64 
33.(1)1000 (2)1000 34. ( 1 )(  ) 3 = (  ´ ´ ) =
5  5  5  5  125 
4  4  4  4  4 4  4  64 
( 2 )( - ) 3 = ( - ) ´ ( - ) ´ ( - ) = -(  ´ ´ ) = -
5  5  5  5  5  5  5  125 

35. (1)90000=9×10000=9× 10 4 

(2)2500=25×100=2.5×1000=2.5× 10 3 

(3)73200000=7.32×10000000=7.32× 10 7 

36. (1)0.00002= 100000  =
2 2 
10 5 
= 2 ´10-5 

(2)0.0000073= 10000000  =
73 73 
10 7 
= 73 ´10-7 = 7.3 ´ 10-6 

(3)0.00918= 100000  =
918 918 
10 5 
= 918 ´10-5 = 9.18 ´ 10-3 

37.  2. 006 ´ 10 16  38.  9. 56 ´ 10 10  39.  6 ´ 10 2  GB 40.  2. 6 ´ 10 12 

41.  1. 6 ´ 10 6  42. 五百億二千三百八十三萬四千


43. B、C、E 44.B、E 45.  1. 68 ´ 10 -24  46.  1. 8 ´ 10 10 

47.(1) < (2) < 48.27 49.  4 ´ 10 -7  50.5 51.20

52. (1) 3 2  = 9 (2) - 3 2  = -9 (3) ( - 3 ) 2  = 9 (4) - ( 3 2 ) = -9

1 1  3  27
53. (1) (  ) 2  =  (2) ( -  ) 3  = - (3) ( - 0 . 2 ) 3  = - 0. 008 
7  49  2  8 
2
(4) - ( 1 . 1 ) 2  = - 1. 21  54. (1) 3 0  = 1 (2) (  ) 0  = 1 (3) - 7 0  = -1

(4) ( - 1 . 234 ) 0  = 1 55. (1) a=6 (2) a+b=4 56. (1) 6 (2) 4

57. (1) 11 (2) 300 58. (1) 3 (2) 2 59. (1) 3 (2) 3

60.(1)6 (2)20 61.(1) 11 (2) 2 62.(1) 6 (2) 8 (3) -12

63. (1) 5 (2) 10 64. (1) 9 (2) 3 65. (1) 10 (2) 0

66. (1) a=6 (2) b=0 67. (1)a=3 (2)b+c=-2

68. (1)  10 3  (2)  10 5  (3)  10 -3  (4)  10 -9 

69. (1)  10 3  (2)  10 -3  (3)  10 -3  (4)  10 9 

70. (1)  2000 ´ 3210000 = 6 . 42 ´10 9  (2)  8 ´10 2  ´ 5 ´10 -5  = 4 ´10 -2 

71. (1) 246800000= 2. 468 ´10 8  (2) 0.00000523= 5. 23 ´10 -6 


(3) = 5 ´ 10 -6  (4) 三億四千二百萬= 3. 42 ´ 10 8 
2 ´ 10 5 
1  3 3 3
72.  73.(1)8 (2)20 (3)8 74.  (  ) 4 > (  )3  > (  ) 2 
12  2  2  2 
3  2 
14  3 
75.  æ 3ö
ç- ÷ 、 æç -3ö
÷ 、 æç -  3  ö÷ 76.-5 77.  78.  9  79.78
è 11 ø è 11 
ø è 11 ø 9  4 
27
80. 
20 
貳、 進階題

一、 選擇題

1.( )下列哪一個式子錯誤?

(A) ( – 2 )3> ( – 2 )5>( – 2 )7

(B) 23<24<25

3 4 5
(C) ( – 2 ) > ( – 2 ) >( – 2 )

1  10 1  11 1  12
(D) (  ) > (  ) > (  )
2  2  2 

2.( )  的科學記號表示為下列哪一個選項?
500000 

(A) 10 -5  (B) 5 ´ 10 -5 (C) 2 ´ 10 -4  (D) 2 ´ 10 -6 

3.( )下列何選項之值是負的?

5  2 
æ 3 ö æç - 4  ö÷
(A) (- 1) ´ (- 1  ) (B) ç -  ÷ ´ ç
2  4 
÷ (C) - (- 1) 2005 (D) (- 5 8 )´ (- 4 )7  ´ (- 3 ) 6 
è 2 ø è 5  ø

4.( ) (- 1)3  - (- 1 )2  + (- 1 )1  - (- 1 ) 0 =?

(A) - 4 (B) - 3 (C) - 2 (D)0

5.( )關於指數的運算,下列何者正確?

(B) (- 3 2  )  = -3 8 

(A) (- 5)2  ´ (- 5 )5  = (- 5 ) 10 

(C) (- 7 )6  ¸ (- 7 )-6  = (- 7 ) 0  (D) (- 2)5  ´ (- 5 ) 5  = 10 5 

6.( )已知 b = a ´ 10 -5  ,其中 5< a <6。

甲說: a ´ 10 -5 是 b 這個數的科學記號表示法。

乙說:由 b = a ´ 10 -5  可知 a = b ´ 10 5  ,其中 b ´ 10 5  是 a 這個數的科學記號


表示法。

丙說: b 是一個負數。

請問誰的說法正確?

(A)甲、乙、丙 (B)甲、乙

(C)甲 (D)甲、乙、丙三者皆錯

7.( )5.88×10 6 -5.8×10 6 -3×10 4  的計算結果,可以科學記號表示

為何?

(A) 5×10 2  (B) 5×10 3  (C) 5×10 4  (D) 5×10 5 

8.( )計算 ( - 1 ) 0  + ( -1 ) 1  + ( -1 ) 2  + LL + ( -1 ) 100  = ?

(A)0 (B)–100 (C)–1 (D)1

9.( ) ( 3 2  ´ 3 3 ) 4  = ?

(A)  3 24  (B)  3 9  (C)  3 20  (D)  3 96 

10.( ) 40 2020  是 40 2019  的幾倍?

(A) 40 (B) 1 (C) 2 (D) 2019

2  2  2 
11.( ) A = (- ) 5  、 B = (- ) 6 、 C  = (- ) 7  ,下列何者正確?
3  3  3 

(A) A>B>C (B) B>C>A (C) B>A>C (D) C>B>A

12.( )比較三數的大小 

A = 560  、 B = 4 90 、 C  = 3120 

(A) A>B>C (B) C>B>A (C) B>A>C (D) A>C>B


1  2  4  8  16 
13.( )  - - + - = ? 
- 2  ( -2 )  ( -2 )  ( -2 )  ( -2 ) 5 
2 3  4 

1 1  1 
(A) 0 (B)  - (C)  (D) 
2  2  32 

14.( )下列那一個算式的答案和 ( 2 10 ´ 3 8  ´ 40 ) ¸ ( 2 5  ´ 3 3  ´ 8 ) 相等?

(A)  2 2  ´ 3 2  (B)  2 5 ´ 3 5  ´ 5  (C)  2 5 ´ 3 11  ´ 32  (D)  25  ´ 3 5  ´ 5 5 

15.( ) (15 ´ 3 5  + 5 ´ 3 5 ) ¸ 5 ´ 3 7 =?

(A)  15 ´ 3 -3  (B)  15´ 3 3  (C)  4 ´ 3 -2  (D)  4´ 3 2 

2
16.( )  以科學記號表示為何?
5 ´ 10 5 
2
(A)  ´ 10 -5  (B)  0. 4 ´10 -5  (C)  4 ´10 -4  (D)  4 ´10 -6 

17.( )「光年」就是光一年所走的距離,約為 9. 4608 ´10 12 公里,

已知甲星球距離地球約 432 光年,試問用科學記號表示約為幾公

里? (9 . 4608 ´ 432 = 4087 . 0656 ) 

(A)  4087. 0656 ´10 12  (B)  40870656´10 9 

(C)  4. 0870656 ´10 15  (D)  0. 40870656 ´10 16 

18.( ) 10 10  是 10 20 的幾倍?

(A) 2 (B)  10 -10  (C)  10 10  (D)  10 2 

19.( )已知 1 奈米= 10 -9  米、1 毫米= 10 -3 米,若 1 毫米=A 奈米,

求 A=?

(A)  10 -3  (B)  10 -6  (C)  10 6  (D)  10 -12 

20. ( )下列何者錯誤?
13 16  19  7  7  7 
(A)  - >- >- (B) (- )6  >(- )4  >(- )2 
14  17  20  5  5  5 
5  5  5 
(C) (0 . 9 )3  >(0 . 9 )5  >(0 . 9 )7  (D) (- )7  >(- )5  >( )3 
3  3  3 

解答:

1.C 2.D 3.A 4.A 5.D 6.C 7.C 8.D 9.C 10.A

11.B 12.B 13.C 14.B 15.C 16.D 17.C 18.B 19.C 20.D
二、 填充題

– 9 – 6 – 10
1. 若 1 奈米 = 10 米,1 微米 = 10 米,1 埃米 =10 米,則 1 微

米是 1 埃米的______倍。

2. 若 a × 4 ×105 = 26 × 108,則 a =____________。 ( 用科學記號

表示 )

3. 若目前百米田徑賽的最快紀錄是 10.49 秒,也就是說平均跑 1 公

n
尺的最快紀錄為 1.049 × 10 秒,則 n = ______。

4 3 2 2 4 a b c
4. 已知 25 × 27 × 4 × 125 × 9 = 2 × 3 × 5 ,則 a =______,b =______,

c =______。
3 2 4
5. 計算 10 × 100 × 1000 的末尾有______個零。

6. 計算 2 5  ´ 2 4  ¸ ( 2 3 ) 2 的值(以指數記法表示)



3  3  3  3  3 
7.  5  × 5  × 5  × 5  × 5  = 5  ,a= 。

8. 5+5+5+5+5=5 a,a= 。

9. 求 a、b 之值:

(1)  ( -81 ) 4 ¸ 3 4  = 3 a  (2)  - 3 4 ¸ ( -3 ) 3  = 3 b 

解答:

1.  10 -6  ¸ 10 -10  = 10 4  2.  a = (26 ´10 8 ) ¸ ( 4 ´10 5 ) = 6 . 5 ´10 3  3.-1

4. a =4,b =17,c =14。 5.9 6.23 7.15 8.2

9. (1) a=12 (2) b=1


三、 綜合、題組題

3
。若 A
1. 用科學符號(即科學記號)可將 1234 表示成「1.234×10 」

9
,則 A 為幾位整數?
的科學符號可表示成「1.23456×10 」

2. 1002005 是 1002004 的______倍。

3. 已知 123 = 1728,那麼:

3
(1) 120 =______。

3
(2) ( 1.2 ) =______。

3
(3) ( 0.12 ) =______。
1  2 3
4. 求 (- ) 2  ´ (  ) 3  ´ (  ) 2  的值
6  5  2 
2 7 14  1 2 
5. 求 ( ) 3  ´ (  ) ¸ - (  ) 的值
3  8  9  2 
1  1 7  7 
6. 求 [(2 - ) 2  - (  - 2 ) 2 ] ´ ( - ) 3  ¸ ( - ) 2  的值
2  2  5  3 
1 1  3
7. 求 (  ) 2 + ( - ) 2  - [ -(  ) 2 ] 的值
4  2  2 

8. 計算下列的值,並以科學記號表示法紀錄下列各數:

(1) (9 ´105 ) ´ (3 ´104 ) =

(2) (4 ´108 ) ´ (8 ´104 ) =

9. 計算下列的值,並以科學記號表示法紀錄下列各數:

(1) (8 ´105 ) ¸ (2 ´103 ) =

(2) (3 ´107 ) ¸ (6 ´102 ) =

3  4  5 
10.( )是非題:請問下列式子是否正確。 0. 5  < 0. 5  < 0. 5  。

11. ( )是非題:請判斷下列敘述是否正確。光的速度是每秒 3× 10 

11 
公尺,若太陽到地球的距離為 4.9× 10  公尺,則光從太陽到地

11  8 
球所需的時間的算式為 4.9× 10  ÷3× 10  。

12. 在空格中填入適當的數:


( - 2 ) 3 ´ ( -2 ) 5  ´ ( -2 )  = (-2 ) 

13. 在空格中填入適當的數: 


3 12  ¸ 3 6  ¸ 3 2 =3 

14. 在空格中填入適當的數:


éæ 2ö ù æ 2 ö □
êç - ÷ ú = ç -  ÷
êëè 3 ø úû è 3 ø

15. 在空格中填入適當的數:

12
2512  ´ 4 12 = □


2ö 1  4 
16. 求 æç - ÷ ¸ 5  ´ 2  的值。
è 3 ø 3 

17. 試比較下列各題中三數的大小:(以 a 、 b 、 c 表示) 

a  = 0. 23 ´ 10 -8  ;  b = 15 ´ 10 -9  ;  c  = 7. 6 ´ 10 -10 

18. 試比較下列三數的大小:(以 a 、 b 、 c 表示) 

a  = 81´ 10 15  ;  b = 1000 ´ 10 14  ;  c  = 0. 032 ´ 10 19 

19. 將下列各數以科學記號來表示

(1) 135000 = 
(2) 42 . 5 ´ 10 5  =

(3) 0 . 000302 ´ 10 8  =

20. 以科學記號表示下列各數:

(1)0.00035407=

(2)235.82=

21. 計算後並以科學記號作答:
13 . 5 ´ 10 2  + 2 . 5 ´ 10 2  =
3  3 3  3 
22.  (- ) 2 、 - ( ) 2 、 (- ) 3 、 - (- ) 3 四個數字,由大到小排列的順序為
5  5  5  5 

__________________。

7  1  7  1  7  1  7  1 
23.  (- ) ´ ( - ) 2 、 (- ) ´ ( - ) 3  、 (- ) ´ ( - ) 4 、 (- ) ´ ( - ) 5  四個數字,由
9  3  9  3  9  3  9  3 

大到小排列的順序為__________________。
解答:

1. 10 位數 2.100 3. (1)1728000 (2)1.728 (3)0.001728

1 10 13
4.  - 5.-1/12 6.0 7.41/16 8. (1)2.7×10 (2)3.2×10
250 

9.(1)4×102 (2)5×104 10.× 11.× 12.9 13.4 14.20 15.100

16.4/3 17. b>a>c 18. c>b>a 19. (1)  1. 35 ´10 5  (2)  4. 25 ´10 6 

(3)  3. 02 ´10 4  20. (1)  3. 5407 ´10 -4  (2)  2. 3582 ´10 2 

3  3  3  3
21.  1. 6 ´10 3  22.  (- ) 2  > - (- ) 3 > (- ) 3  > - ( ) 2 
5  5  5  5 
7  1  7  1  7  1  7  1 
23.  (- ) ´ ( - ) 3  > (- ) ´ ( - ) 5  > (- ) ´ ( - ) 4 > (- ) ´ ( - ) 2 
9  3  9  3  9  3  9  3 
參、 挑戰題

一、 選擇題

1.( )  2. 5 ´10 6  的倒數為何?

(A)  2. 5 ´10 6  (B)  4 ´10 -7  (C)  2. 5 ´10 -6  (D)  4´10 7 

2.( )若 a=2 5 ×3 3 ×7 5 ,b=2 4  ×3 2  ×7 6 ,c=2 6 ×3 4  ×7 4 ,則下列何

者正確?

(A) b>a>c (B) a>b>c (C) c>a>b (D) b>c>a

3.( )一莫耳約有 6´10 23 個粒子,那麼 2. 4 ´10 30 個粒子,等於幾莫

耳(以科學記號表示)?

(A)  0. 4 ´10 7  (B)  4´10 6  (C)  2. 5 ´10 7  (D)  2. 5 ´10 -7 

4.( )SARS 傳染期間人心惶惶,生命健康是最重要的,以下四

種口罩,何者能有效避免感染病毒?『1 奈米= 10 -9 m 』(SARS 病毒大

小僅 80-160 奈米)

甲- 織布縫隙是 10 -12 m 

乙- 織布縫隙是 10 -9 m 

丙- 織布縫隙是 7 ´10 -8 m 

丁- 織布縫隙是 2 ´ 10 -7 m 

(A)甲、乙 (B)乙、丙 (C)甲、乙、丙 (D)乙、丙、丁


解答:

1.B 2.A 3.B 4.C


二、 填充題

3  4 3 52 
1. 計算  (3 50 + 3 51 ) - (  ´ 3 51  + ) =
4  7  7 
- 2  ( -2 ) 2  ( -2 ) 3  ( -2 ) 4 
2. + + + =
- 2  - 4  - 8  - 16 

解答:

1.0 2.0
三、綜合、題組題
1 3 9 27 
1.計算  + + + 的值。
-3 (-3) (-3) (-3) 4 
2 3

1 3  9  27 
2. 計算  - + - 的值。
- 3  ( -3 )  ( -3 )  ( -3 ) 4 
2  3 

1 3 9 27
3. 計算  -  2  -  3  -  4  的值(答案若為分數,請以最簡分
( -3 )  ( -3 )  ( -3 )  ( -3 ) 

數表示)

3  3 
462 ö æ 395 ö
4. 計算 æç - ÷ ´ç ÷ 的值(答案若為分數,請以最簡分數表示)
è 237 ø è 385 ø

1  1  1  1  2  2  4 1 
5. 計算 (5  - 14  + 9  ) 3  ´ ( 3  ´ 4  + ) - (  - 2  ) 4  的值(答案若為分數,請
3  2  6  5  7  7  3  3 

以最簡分數表示)

1  1  1 3
6. 計算 (1  ) 2  - ( 3  ) 2  + (  - 1 ) 3  - (  - 1 ) 3  的值(答案若為分數,請以最簡
2  2  2  2 

分數表示)
5  5  1  2 1  2
7. 計算(-  )÷{  -[(-  ) -(-  ) ]}+1.75 的值(答案若為分
9  6  3  2 

數,請以最簡分數表示)

解答:

4 2 1  33
1.0 2.  - 3.  - 4.8 5.13 6.  - 10 7. 
3  3  4  28 
主題三 分數的運算

3-1 因數與倍數

壹、 基本題

一、 選擇題

1. ( )小皮有 200 個數碼寶貝,要分送給幾個好朋友。為公平起

見,每個人要分一樣多個,而且要全部分完,則它可以造福幾

個好朋友? (A)50 (B)60 (C)70 (D)80

3 2 3 2
2. ( )下列哪些數是 5 ×7 ×11 的倍數? (A)5 ×7×11

4 3 4 4 3
(B)5 ×7 ×11 ×13(C)5×7×11×13(D)5 ×7 ×17

3. ( )下列敘述何者錯誤?(A)5 是 15 的因數(B)6 是 12 的

倍數(C)1 是 15 的因數(D)21 是 3 的倍數

4. ( ) 已知 297 被一個整數整除,所得的商不可能為下列何

者?(A)27(B)33(C)66(D)99

5. ( )若 A=15×16×17×18×19,則下列何者是 A 的因數?

(A)55(B)65(C)75(D)85

6. ( )設 a、b、c 都是正整數,a≠1,如果 a = b × c 時,那麼

下列哪一個敘述是正確的?(A) a 是 b 的倍數,a 也是 c 的倍數

(B) a 是 b 的因數(C) a 是 c 的因數(D) b 是 c 的倍數


7. ( )下列有關倍數的敘述何者正確?(A) 300 是 0 的倍數

(B) 因為 33 = 6.6 × 5,所以 33 是 5 的倍數(C) 個位數字是 3、

6 或 9 的整數,一定是 3 的倍數(D) 300 是 1 的倍數

8. ( )草坪上有 60 隻羊,要全部分完。若 3 個人均分,每個人

可以分到 20 隻,當下列哪一種人數時,不能均分?(A) 4 人(B)

5 人(C) 6 人(D) 7 人

9. ( )32 顆果凍不能平均分給幾位同學而沒有剩餘?(A) 1 (B)

2 (C) 4 (D) 6

10. ( )二位數中 9 的倍數共有多少個?(A)9 個(B)10 個(C)11 個

(D) 12 個

11. ( ) 72 位同學參加啦啦隊比賽,想排成一長方形的隊伍,且

每行及每列的人數不得少於 4 人,問有幾種排法? (A)12 (B)6

(C)4 (D)3

12. ( )拔河比賽時,雙方人數的總和一定是下列那一個數的倍數

(A)2 (B)3 (C)5 (D)9

13. ( )若四位數 543a 為 2 的倍數,則 a 有幾種可能?(A) 4(B) 5

(C)9 (D)10

14. ( )某數的所有因數為:1、2、4、8、16、32,某數是多少? (A)32

(B)64 (C)128 (D)136


15. ( )哪幾組數互質?(A)甲乙(B)丙丁(C)甲乙丙丁(D)甲乙丁

甲:27,28、乙:17、43、丙:12,51、丁:21,50

16. ( )a 是一個正整數,其所有正因數有 1、2、4、7、14、28,

則 a 最大為何?(A) 4 (B) 7 (C) 28 (D) 56


91
17. ( )設 a 為整數,欲使  為整數,則 a 可能為下列何數?
a + 1 
(A)1 (B)7 (C) 12 (D)13

18. ( )設 a、b、c 是三個相異正整數,如果 a÷b = c 時,那麼下

列哪一個敘述是正確的?(A) a 是 b 的因數(B) b 是 c 的因數(C)

a 是 b 的倍數(D) b 是 c 的倍數

19. ( )11070 可以被某數整除,請問所得的商數不可能是下列哪

一個數?(A) 27 (B) 36 (C) 54 (D) 63

20. ( )判別下列各數中,有幾個數是 18 的倍數?432、1881、

6516、、99998、20016(A) 1 (B) 2 (C) 3 (D) 4

21. ( )設 a = 23 × 32 × 5× 112,則下列哪一個選項不是 a 的因數?

(A) 22 × 3 × 112 (B) 32 × 5 (C) 23 × 52 × 11 (D) 2 × 3 × 5 × 11

22. ( )下列何者不是 52 的因數?(A) 50(B) 51(C) 52(D) 53

23. ( )下列各數何者為 3 的倍數也是 5 的倍數?(A) 3742 (B)

5745 (C) 8750 (D) 2712


24. ( )如果四位數 261 □ 是 2 的倍數,又是 3 的倍數,那麼 □

內可以填入哪個數字呢? (A) 2 (B) 4 (C) 6 (D) 8

25. ( )判斷下列哪一選項是 11 的倍數?

(A)11235(B)38347(C)11111(D) 516175

26. ( )若六位數 82□091 是 11 的倍數,那麼 □ 內可以填入

(A)6(B)7(C)8(D)9

27. ( )如果六位數 48 □095 是 11 的倍數,那麼 □ 內可以填入

(A) 0 (B) 2 (C) 4 (D) 6

28. ( )如果七位數 432 □905 是 11 的倍數,那麼 □ 內可以填

入 (A) 3 (B) 4 (C) 5 (D) 6

29. ( )下列哪一個敘述是正確的?(A) 500 是 0 的倍數(B) 因為

45 = 2.5 × 18,所以 45 是 18 的倍數(C) 個位數字是 3、6 或

9 的整數,一定是 3 的倍數(D) 2500 是 1 的倍數

30. ( )下列哪一個數是質數?(A) 1(B) 51(C) 71(D) 91

31. ( ) 從 1 到 99 之間的所有整數,刪去 2、3、5、7 的倍數後,

所剩下的最大整數是? (A) 91(B) 93(C) 97(D) 99

32. ( )20~50 中,質數有幾個?(A)7(B)8(C)9(D)10

33. ( )下列敘述何者正確?(A) 13 = 5 + 8,所以 13 是合數(B)2

是質數也是合數(C)最小的質數是 1(D)質數中有奇數也有偶數
34. ( ) 在大於 41 且小於 71 的整數中,最大的質數為 a,最小

的質數為 b,若 a – b = c,則:(A) c 是質數(B) c 有 5 個因

數(C) c 為 3 的倍數(D) c 為 7 的因數

35. ( ) 有關質數與合數的敘述,何者正確?(A)質數必為奇數,

(B) 合數必為偶數(C) 1 是質數(D)在 1 到 10 的整數中,合數

的個數比質數多

36. ( )下列哪一個選項的敘述是正確的?(A)所有偶數都是合

數(B)所有質數都是奇數(C)0的倒數就是0(D)1不是質

數也不是合數

37. ( )下列何者不是「質數」?(A) 1 (B) 43 (C) 59 (D) 83

38. ( )下列何者不正確? (A) 6 個 3 連乘可記為 6 3  (B) 1 可以

整除任何整數 (C) 14 的質因數有 1, 2, 7 (D) 1 到 20 之間所

有質數和為奇數。

39. ( )下列何數是 2 的倍數也是 3 的倍數?

(A) 2174812 (B) 18930005 (C) 1547 (D) 2104878。

40. ( )有一整數,質因數有 2, 3, 7,則此數可能為何?(A) 21

(B) 70 (C) 237 (D) 252。

41. 若 x = 12 ´ 13 ´ 14,則下列何數不是 x 的因數?(A) 1 (B)

42 (C) 144 (D) 182


解答:

1.A 2.B 3.C 4.C 5.C 6.A 7.D 8.D 9.D 10.B

11.B 12.A 13.B 14.A 15.D 16.C 17.B 18.C 19.D 20.C

21.C 22.D 23.B 24.C 25.D 26.C 27.A 28.D 29.D 30.C

31.C 32.A 33.D 34.C 35.D 36.D 37.A 38.A 39.D 40.D

41.C

二、 填充題

1. 請列出 96 的所有的正因數 ,及所有的質因數

2. 在 100 到 200 的整數中,所有 3 的倍數有 個?

3. 在 100 到 500 的數中,是 3 的倍數和 5 的倍數的整數有

個?(包含 100 和 500)

4. 在 100 到 500 的數中,是 3 的倍數或 5 的倍數的整數有

個?(包含 100 和 500)

5. 10×13×15×16×18 的標準分解式為

6. 兩堆芒果,每堆 10 個一數,各餘 3 個和 4 個。如果把這兩堆合

起來再 5 個一數,那麼最後會剩餘幾個?

7. 求 1575 的標準分解式?

8. 寫出 1500 的質因數?

9. 由「大到小」寫出 1125 的因數,其中排名第 3 的是?


10. 若某數為 21 的倍數與 504 的因數,則此數可能有幾個?

11. 欲使 1200×a 是完全平方數,則 a 的最小值為何?

12. 如果七位數 432 □ 905 是 11 的倍數,那麼 □ 內可以填入哪

些數字呢?

13. 如果四位數 262 □ 是 2 的倍數,又是 3 的倍數,那麼 □ 內

可以填入哪些數字呢?

14. 請判別下列各數何者為 9 的倍數? 何者為 11 的倍

數? (A)1284723 (B)2576431 (C)88902

(D)111065 (E) 134112 (F) 396275 (G)2120778

(H)333360

15. 四位數 17□3 含有因數 3,則 □ =______。

16. 甲數的所有因數為:1、2、4、8、16、32,甲數是______。

17. ( 64 , 144 ) =______。

18. 如果五位數 1357□ 是 2 的倍數也是 3 的倍數,則 □ 內可以

填入那些數字______。

19. 求下列各組數的最大公因數與最小公倍數:

(1)(105 , 196 , 112)=______。

﹝105 , 196 , 112﹞=______。

(2)(22 × 32 × 5 , 22 × 33 × 7)=______。
2 2 2 3
﹝2 × 3 × 5 , 2 × 3 × 7﹞=_________。

20. 請判別下列各數何者為 3 的倍數?

(A) 1284723 (B) 2576431 (C) 88902 (D) 111065

(E) 134112 (F) 396275 (G) 2120778 (H) 333360

21. 將 48 個橘子、36 個蘋果、60 個梨子分裝在幾個禮盒中,使同

一種水果在每盒內的個數一樣,則最多可裝幾盒?

22. 下列哪幾組數互質?

甲:1,3 乙:2,9 丙:12,35 丁:27,123

23. (1)四位數 17□3 含有因數 3,則□內可以填那些數字?

(2)五位數 217□0 含有因數 3,則□內可以填那些數字?

(3)四位數 7□32 是 3 的倍數,則□內可以填那些數字?

(4)五位數□1701 是 3 的倍數,則□內可以填那些數字?

24. (1) 如果五位數 8□703 是 9 的倍數,那麼 □ 內可以填入哪些

數字呢?

(2) 如果六位數 3□2703 含有因數 9,那麼 □ 內可以填入哪

些數字呢?

25. (1)45 的所有因數為: ,45 的質因數為:

(2)某數的所有因數為:1、2、3、4、6、9、12、18、36,

某數是______。它的質因數有:
(3)3579 的所有因數中,最大者與最小者的和是______。

( )
26. (1)填入適當的數:315=3×( )=3×5×( )=3×5×3×( )=3 ×5×7

(2)求 552 的標準分解式

(3)若 3780 的標準分解式為 22× 3□× 5×7,那麼□=____

27. 請判斷 23456、234432 是不是 11 的倍數?__________

28. (1)48 的質因數有____個 (2)整數 735 有______個相異質因數

29. (1)若 x =12 × 18 × 24,將 x 質因數分解=______。

(2) 設 a = 22 × 27 × 49 × 50,將 x 質因數分解____________。

30. 判斷 67、77、83、91 四數中哪些是質數?__________

哪些是合數?_________

31. 寫出 36 的所有因數,並指出 36 的質因數因數為_________

32. 將 1440 作質因數分解,並寫出 1440 的質因數

33. 將 5 ´ 5 ´ 5 ´ 5 寫成指數形式 a k  (其中 a 為質數)

34. 將 4 ´ 4 ´ 4 ´ 4 ´ 4 寫成指數形式 a k  (其中 a 為質數)

35. 小瑄拿了一盒巧克力糖,盒裡裝了 24 顆的巧克力糖,她想將巧

克力糖平分給朋友吃,但是每個朋友能夠分到巧克力糖的數量

要相同;若每個朋友至少要分到 2 顆,請問她有哪幾種的分法?

請你幫她一一寫出來

36. 寫出 10 以下的所有正整數,
並判斷這些數中哪些是質數? 哪些是合數?

37. 寫出 10 的因數? 哪些是 10 的質因數?

38. 111、222、333、444、555、666 六數中:

(1)含有因數 2 的數 (2)含有因數 3 的數 (3)

含有因數 5 的數

39. 回答下列問題,並說明理由(1)1 是 308 的質因數嗎?(2)4

是 308 的質因數嗎?(3)7 是 308 的質因數嗎?(4)11 是 308

的質因數嗎?

40. 將 120 作質因數分解,並寫成標準分解式

41. 將 255 作質因數分解,並寫成標準分解式

42. 將下列各述中的質數圈起來 1、2、3、4、5、6、7、8、9、10、

11、12、13、14、15

43. 判別下列哪些數是 976 的因數,並將這些數圈起來 1、2、3、5、

8、16、32、61、122、976

44. 280 這個數有多少個相異的質因數?

45. 兩數 555 與 3 2 ´ 5 ´ 7 擁有的相異質因數中,相同的有哪些?

46. 990 到 1000 的整數中有幾個有因數 3?

47. 將 30 到 90 的整數去掉 2、3、5 的倍數後,剩下的整數中最大

與最小的為何?
48. 小雞想將 54 顆雞蛋分堆,每堆中雞蛋最少 3 顆,最多 18 顆,

則可能有幾種分法?

49. 若 a 是正整數,且 a 的因數有 1、 7、 9、 21,則 a 的最

小值是

50. 所有小於 100 的正整數中,最小的合數是 ;最大的質

數是 。

51. 寫出 49 ´ 49 ´ 49 的標準分解式=________

52. 以 121 這個數為例,它的因數恰好有 1、 11、 121 三個因數,

試問所有正整數中,符合「恰好有三個因數」的最小整數

為 。

53. 已知一個三位數,百位數字為 9、十位數字為 1,如果它可以

同時被 5 和 7 整除,那麼這個三位數為 。

54. 求 5566 所有的質因數

55. 有 48 個好吃而且相同的糖菓,要分成若干份,若每一份的糖

菓數目都要相等,可以分成多少份,請寫出有哪幾種不同的分

法?

56. 寫出 2280 的標準分解式。

57. 設一個四位數 67□2 是 11 的倍數,則 □=

58. 下列那些數能被 11 整除?答:   (全對才給分)


(1)4444 (2)333 (3)6479 (4)77077 (5)2385625 (6)

43579

59. 下圖中有 25 個數:(1) 比 16 大的質數有那些?ˉˉˉˉ 。

(2) 比 10 小的合數有那些?ˉˉ 。

60. 將 125000 作質因數分解,並寫成標準分解式

61. 33333334 除以 3 的餘數是多少?

解答:
1.正因數:1、2、3、4、6、8、12、16、24、32、48,質因數:2、3
5.26×33×52×13
2 2 
2.33 3. 27 4.187 個 6.2 個 7. 1575 = 3 ´ 5  ´ 7 

8.2,3,5 9.225 10.24 個 11.70 12.6 13.2、8

14.9 的倍數 ACGH、11 的倍數 ABCEFG 15.1、4、7 16.32 17.16

18.2 或 8 19.(1) 7,24 × 3 × 5 × 72 (2) 22 × 32 ,22 × 33 × 5 × 7

20.ACEGH 21.12(盒) 22.乙、丙 23.(1)1、4、7 (2) 2、5、8

(3)0、3、6、9 (4)3、6、9 24.(1) 0、9 (2) 3 25.(1) 1、3、

5、9、15、45,3、5 (2)36,2、3 (3)3580 26.(1)105、21、7

(2)23×3×23 (3)3 27.不是、是 28.(1)2 (2)3

29. (1)26×34 (2)22×32×52×72×11 30.質數:67、83,合數:77、91

31.因數 1、2、3、4、6、9、12、18、36,質因數為 2、3 32. 2 5 ´ 3 2  ´ 5 、


質因數:2、3、5 33. 5 4  34. 2 10  35.
36.質數:2、3、5、7,合數:4、6、8、9、1037 37.因數是 1、2、

5、10 質因數是 2、5 38.(1)222、444、666 (2)111、222、333、

444、555、666(3)555 39.(1)不是,因為 1 不是質數(2)不是,因

為 4 不是質數(3)是, 308 = 7 ´ 44 且 7 是質數 (4)是, 308 = 11 ´ 28 且

11 是質數 40. 2 3 ´ 3 ´ 5  41. 3 ´ 5 ´ 17  42.2、3、5、7、11、13

43.1、2、8、16、61、122、976 44.3 個 45.3、5 46.4 個

47.小的是 31 大的是 89 48.4 49.63 50.4、97 51.  7 6 


52.4 53.910 54.2,11,23

55.1、2、3、4、6、8、12、16、24、48 份 56. 2  ´ 3 ´ 5 ´ 11 


3
57.3

58.1345 59.(1)17、19、23(2)4、6、8、9 60. 2 3  ´ 5 6  61.1

貳、 進階題

一、 選擇題

1. ( ) 1250 到 1260 中,3 的倍數有多少個?(A) 1 個 (B) 2

個 (C) 3 個 (D) 4 個

2. ( ) 小於 100 的正整數中,可以被 2 或 3 整除的數共有個?

(A) 50 個 (B) 33 個 (C) 16 個 (D) 66 個


3. ( ) 1 至 200 的偶數中,3 的倍數共有 a 個,5 的倍數共有 b

個,請問 a + b =?(A) 99 (B) 86 (C) 73 (D) 60

3 2 2
4. ( )設 a = 2 × 3 × 7 × 13 ,則下列哪一個選項不是 a 的因數?

(A) 22 × 3 × 132 (B) 32 × 7 (C) 23 × 72 × 13 (D) 2 × 3 × 7 × 13

5. ( ) 5148、3198、29381、38142 四數中,是 2 的倍數有 a 個,

3 的倍數有 b 個,9 的倍數有 c 個,11 的倍數有 d 個,則 a + b

+ c + d =?(A) 10 (B) 9 (C) 8 (D) 7

6. ( ) 一個六位數字 214a7a 是 6 的倍數,請問 a 值共有多少種

可能呢?(A) 1 (B) 2 (C) 3 (D) 4

7. ( )有一個五位數 2005□,是 3 的倍數,且用 5 除之會餘 3,

請問 □ 內為何數?(A) 2 (B) 4 (C) 6 (D) 8


8. ( ) 若 2 ×7 是整數 112 的因數,則□不可能是下列哪一個

數?(A) 0 (B) 1 (C) 4 (D) 5。

9. ( ) 若 2×3□×7×11 是整數 1386 的倍數,則□可能的值有幾

個?(A) 1 (B) 2 (C) 3 (D) 4

10. ( )若 54 可以分解 a ´ b ,其中 a、b 均為正整數,則下列哪

一個不可能是 a - b  的值?(A)15(B)25(C)29(D)53

11. ( )若 a = 15 ´ 16 ´ 17 ´ 18 ´ 19 ´ 20 ´ 21 ´ 22 ´ 23 ´ 24 ´ 25 ,則下

列何者錯誤?(A) a 的相異質因數共有 8 個(B) a 是 10 4 的倍數


(C) a 的最大質因數與最小質因數的和仍是 a 的因數(D) a 所有

相異質因數的和是質數

12. ( )1 到 1000 的整數中,可被 3 或 4 整除的共有幾個?(A) 83

個 (B) 249 個 (C) 499 個 (D) 500 個

13. ( ) A = 9 × 10 × 11 × 12 × 13 × 14 × 15,則 A 的質因數有幾個?

(A) 5 (B) 6 (C) 7 (D) 8

14. ( ) 設 2 × 4 × 6 × … × 20 = B,則 a 共有幾個質因數?

(A)3 (B) 4 (C) 5 (D) 6

15. ( ) C = 21 × 23 × 25 × 27 × 29 ×…× 59,則 C 的相異質

因數有幾個?(A) 16 (B) 20 (C)22 (D) 24

解答:

1.D 2.D 3.D 4.C 5.A 6.B 7.D 8.D 9.C 10.C

11.D 12.D 13.B 14.C 15.A

二、 填充題

1. 小叮噹用 46 張大小相同且邊長為 4 ㎝的磁磚,幫大雄建一個長

方形的遊戲場地,則周長最大可能是 ㎝,最小是

㎝。

2. 有一個二位質數 8□,若此數加 7 後是 2 的倍數,則□=

3. 已知有一六位數 3158□4 是 36 的倍數,則□=


4. 有一批長方形大理石磚,每個石磚的長都是 35 公分、寬都是

10 公分,請問至少要 塊這種大理石磚才能鋪成一個正方形

( 不可重疊、切割或留下空隙 )

5. 將 96 個蘋果、60 個水梨、36 個甜柿分裝在幾個盒子裡,使每

個盒子裡的同一種水果數量都一樣多。請問最多可裝成幾盒?

此時每個盒子中的蘋果、水梨、甜柿各有幾個?

6. 888 減去 a 之後是 9 的倍數,則 a 值最小為______。

7. 小於 50 的整數中,可以被 2 或 3 整除的數共有______個。

3
8. 下列是將正整數甲分解成質因數乘積的計算過程,甲= 2 × a =

3 3 3 2 3
2 × 3 × b = 2 × 3 × 5 × d = 2 × 3 × 5 × d = 2 × 3 ×

3
5 ,則 a= ,b =______,c =______,d =______

9. (1)7777 減去 a 之後可以被 9 整除,a 值最小為______。

(2)一個四位數 357□,不能被 2、3、5 整除,而且個位數字

不是 1,請問此四位數是______。

10. (1)24 的標準分解式為______。

(2) 用 24 塊相同的正方形,全部排成一長方形,若長方形的長

與寬互換,算同一種的長方形,則可排出______種不同形狀的

長方形。

11. 2 是不是 32 ´ 31 的因數?


3 c d
12. (1) 4536= a × b × 7 ,則 a + b + c + d =?

a b c d
(2)有一個五位數字 8408□的標準分解式為 2 × 3 × 7 × 11

×13,則 □ 內的數字應是多少?a + b + c + d 的值是多少?

(3)有一個五位數字 2032□的標準分解式為 2a × 3b × 5c ×7× 11d

,則 □ 內的數字應是多少?a - b + c - d 的值是多少?

13. (1)若 148×甲=乙×乙,則甲數最小值是 ,此時乙數為

(2)若 504×丙=丁×丁,則丙數最小值是 ,此時丁數為

(3)若 504×A=B×B×B,則 A 數最小值是 ,此時 B 數為

14. (1)625 可以分解成 a、b 兩數的乘積,如果 a、b 都是合數,求

a + b =______。(2) 72 可以分解成 a、b 兩數的乘積,如果 a、

b 都是合數,寫出 a + b 所有可能的答案 =______。

15. 下列各數何者含有因數 2?9992、8884、2223、 22 ´ 3 2 

16. 請判斷 999992 是不是 3 的倍數?

17. 請判斷 1111、111111、11111111 是不是 11 的倍數?

18. 2×2×2×2×2×2×3×3×3×7×11=133056,請將 133056 寫成標準分解

19. 將下圖的質數連起來,可形成一個英文字母,請問這個英文字
母是那個字呢?

20. 寫出 20 以下的所有正整數,並判斷這些數中哪些是質數?哪些

是合數?哪些是 20 的因數?哪些是 20 的質因數?

【能力指標:N-3-02、N-3-01 分年細目:7-n-09、7-n-10】

21. 將 800 作質因數分解,並寫成標準分解式

22.  2 5  ´ 5 3  是不是 2 4  ´ 5 3 的倍數?

23. 下列各數: a =9119, b = 2 ´ 3 2 ´ 5 3  ´ 7 , c =9210201。請以 a , 

b , c 回答下列問題:

(1) 2 的倍數有哪些? (2) 3 的倍數有哪些? .

(3) 5 的倍數有哪些? (4) 11 的倍數有哪

些? .

24. 求 23×72+23×26 的標準分解式 .

25. 請從 3、 4、 5、 7 四個數中,挑選出三個數,排成一個三位

數,使其成為 3 的倍數,則此三位數可以是多少?(請任寫出

四個答案)

26. 介於 20 至 30 之間(不含 20、 30 這兩個數)所有的「合數」

相加時,其中有一個「合數」沒有加到,得到結果是 148,請
問沒加到的那個數是多少?

27. 請在下列空格中,填入所有適當的 0~9 數字,使得條件成立。

若五位數字 873□5 為 3 的倍數,則 □=ˉˉˉˉ 。

28. 請在下列空格中,填入所有適當的 0~9 數字,使得條件成立。

若五位數 4□325 除以 3 會餘 2,則 □ 內可以填入哪些數?

解答:

1.376 ㎝,200 ㎝ 2.3、9 3.6 4.14 5. 蘋果有 8

個、水梨有 5 個、甜柿有 3 個。 6.6 7.32 8. 375,125,

3
25,5 9.(1)1(2)3577 10.(1)2 ×3(2)4 11.是

12.(1)10(2)4、6(3)8、0 13.(1)37、74(2)14、84(3)147、42


14.(1)50(2)22、18、17 15.9992、8884、 22 ´ 3  16.不是 17.

是 18. 2 6 ´ 3 3  ´ 7 ´ 11  19.P 20.2、3、5、7、11、13、17、19,4、

6、8、9、10、12、14、15、16、18、20,1、2、4、5、10、20,2、5 21. 2 5  ´ 5 2 

22.是 23.(1)b(2)b,c(3)b(4)a,c 24. 2 ´ 3 ´ 13 ´ 23 

25.345,354,453,435,534,543 26.25

27.1、4、7 28.0、3、6、9

參、 挑戰題

一、 填充題
2
1. 若 P=a ×b×c,且 a、b、c 均為質數,則 P 共有 個正

因數

2. 四個連續正整數的乘積為 3024,求此四數。

6、7、8、9

3. 有一個五位數字 9172□,它的標準分解式為 2a × 3b × 7c × 11d×

13,則 □ 內的數字應是多少?a – b + c – d 的值是多少?

(a、b、c、d 可能是 0)

4. 從 1 到 99 之間的所有整數,刪去 2、3、5、7 的倍數後,所剩

下的最大整數是______。

5. 甲乙兩人在不同的保險公司上班,甲每上班 5 天休假 1 天(連

上 5 天班後休息一天)
,乙每上班 7 天休假 1 天(連上 7 天班後

休息一天),若恰好甲、乙兩人同在今天星期四休假,則:

(1) 下次兩人在同一天休假是______天後。

(2) 下次兩人同在星期四休假是______天後。

6. 林先生買了一張壁報紙,長 108 公分、寬 78 公分,想要裁剪成

很多個相等的正方形,而正方形是越大越好,那麼可以剪成

______個正方形。

解答:

1.12 2.6、7、8、9 3.4 4.97 5.(1) 24(2) 168 6.234


主題三 分數的運算

3-2 最大公因數與最小公倍數

壹、 基本題

一、 選擇題

1. ( )下列何者與 35 互質?(A)12(B)15(C)21(D)30

2. ( ) 下列哪一組數的最大公因數為 30?(A) 48,108

(B) 36,98(C) 72,144 (D) 90,120

3. ( ) a 是一個正整數,其所有正因數有 1、2、4、7、14、28,

則 a 與 210 的最大公因數為何?(A) 4 (B) 7 (C) 14 (D) 28

4. ( )若(126 , a)= 18,[a , 126]= 378,則 a 之值為多少?

(A) 36 (B) 54 (C) 72 (D) 81

5. ( )已知 290= 2 ´ 5 ´ 29 ,則下列哪一個數與 290 互質?

(A)2002(B)2001(C)2000(D)1999

6. ( )計算 [ 18,(12,16)] 之值為何?(A) 18 (B) 36 (C) 72

(D) 144

7. ( )下列各組數的最大公因數,何者為「1」?(A) 147、189

(B) 34、187 (C) 84、138 (D) 47、93

8. ( )已知 70 = 2 ´ 5 ´ 7,請問 70 與下列那一個數「互質」?

(A) 2000 (B) 2003 (C) 2006 (D) 2009


9. ( )「甲、乙、丙、丁」四個數,甲 =(34,85)、

乙 =[34,85]、丙 =(37,79)
、丁 =[37,79]
,請判別這四

數的大小關係?(A) 甲<丙 (B) 乙<丙 (C) 乙<丁 (D) 丁<

10. ( )菲原、柯西、袁太三人想求 70、 98、 210 三數的最小

公倍數及最大公因數,但是他們三人對於「最小公倍數」和「最

大公因數」的求法並不是很清楚,以下三個長方形圖記內是他

們的計算過程。請你判斷下面哪一個結果是正確的?

(A) 依袁太的算法可得:[70,98,210]= 7 ´ 2 ´ 5 ´ 7 ´ 15

(B) 依柯西的算法可得:[70,98,210]= 10 ´ 7 ´ 14 ´ 3

(C)依菲原的算法可得:(70,98,210)= 2 ´ 5 ´ 7

(D) 依袁太的算法可得: (70,98,210)= 2 ´ 7

2  70,98,210  10  70,98,210  7  70,98,210 


7  35,49,105  7  7,98,  21  2  10,14,  30 
5  5,  7,  15  1,14,  3  5,  7,  15 
1,  7,  3  <柯西的算法>  <袁太的算法>
<菲原的算法> 

11. ( )能同時整除 18 與 24 的正整數,共有幾個?

(A) 4 個 (B) 6 個 (C) 8 個 (D) 很多個,數不完。

12. ( )水果不滿 200 個,8 個一數剩 3 個,12 個數也剩 3 個,這 


些水果個數不可能是下列哪個數? (A) 195 (B) 189 (C) 27

(D) 123

13. ( )下列各組數何者互質?(A) 11, 22 (B) 21, 27 (C) 65, 91

(D) 23,33

14. ( )成功國中學生數一年級有 1053 人、二年級有 858 人、三

年級有 975 人、不分年級、全校每班的人數都相等。那麼一年

級的班級數至少幾班?(A) 25 班 (B) 27 班 (C) 31 班

(D) 39 班

15. ( )若 a = 24 ´ 32 ´ 5 ´ 72、 b = 22 ´ 33 ´ 52 ´ 7,

則下列何者不為 a、 b 的公因數?

(A) 24 ´ 33 ´ 52 ´ 72 (B) 2 ´ 32 ´ 5 (C) 3 ´ 5 ´ 7

(D) 1

16. ( )若 a=2×32×7,b=2×3×5×91,下列何者錯誤?

(A)a、b 的最大公因數是 42

(B)a、b 的最小公倍數是 2×32×5×7×91

(C)a、b 共同的質因數有 2、3、5、7

(D)17 是 a+b 的因數

17. ( )哪一組數的最大公因數為 30?(A) 48,108(B) 36,98(C)

72,144(D) 90,120
解答:

1.A 2.D 3.C 4.B 5.D 6.A 7.D 8.B 9.C 10.B

11.A 12.B 13.D 14.B 15.A 16.C 17.D

二、 填充題

1. 試找出小於 15 且與 540 互質的數有哪些?A:_____

2. ○
1 (36,90,105)= ○
2 [36,90,105]=

3 2 3 2 2 2 2 2 2
3. ○
1 (2 ×3 ×5×7,3 ×5×11 ,2 ×3 ×5 ×7 ×11 )=


2 以標準分解式表示下題:

[23×32×5×7,33×5×112,22×32×52×72×112]=

4. [(2×32×5×73,33×52×72×11),24×33×74×13]=

(以標準分解式表示)

5. 求(1575,2295)=

6. 若(a,60)=15,【a,60】=180;則 a=

7. 奇異果 90 個,全部分裝成若干盒,每盒個數相同且每盒至少 5

個,但不超過 30 個;問共有________種不同的裝法

8. a 是小於 500 的正整數,滿足 a 除以 13 餘 5 的正整數 a 共有__


1  1 
9. 已知某數分別乘以  與  ,所得的積均是正整數;則此數的
18  24 
最小值為________。

10. 某正整數除以 8 餘 6,除以 12 餘 10,除以 15 則不足 2,則此

數最小值為________。

11. 一庭院長 427 公分,寬 140 公分,要舖設大小相同正方形磁磚

將庭院『全部舖滿』
,最少需要_________塊

12. 若甲、乙兩數的最大公因數為 6,最小公倍數為 1080,且

乙數 = 54,求甲數=__________

13. 某條長 100 公里的快速道路上,自起點開始每隔 150 公尺設立

一盞照明路燈,每隔 500 公尺設立限速標誌。已知起點同時設

有照明路燈和限速標誌,請問共有幾處同時設置有照明路燈和

限速標誌?A:_____

14. 求出下列各小題的值:○
1 ( 60 , 144 ) =______。


2 ( 35 , 42 , 63 ) =______。○
3 (105 , 63 , 273)=______。

15. 求出下列各小題的值:

2 4 3 2 5

1 ( 2 ×5 ×7 ×13 , 5 ×7 ×13 ) =______。

4 2 3 5 2 4 2 3 2

2 (2 ×3 ×7 ×13 , 2 ×3 ×5 ×13 , 2 ×3 ×5 ×7×11 ) =______。

4 3

3 (2 ×3×5 ×7 , 88×125 , 3800)=_______。

3 5 11
16. ○
1 寫出 5 、5 、5 所有的公因數:


2 寫出 112、196、700 的所有公因數:
17. 求出下列各小題的值:○
1 [72 , 48]=______。


2 [48 , 36 , 80]=______。○
3 ﹝147 , 196 , 112﹞=______。

18. 求出下列各小題的值:

2 2 3

1 ﹝2 × 3 × 5 , 3 × 7﹞=______。

4 2 2 3 3

2 ﹝2 × 5 × 11 , 2× 5 × 7, 2 × 11﹞=______。

2 2

3 ﹝336 , 8×105, 2 × 3 × 5﹞=________。

19. 兄弟兩人在園區上班,哥哥每上班 4 天休假 1 天,弟弟每上班

3 天,休假 1 天,若恰好甲、乙兩人同在今天星期四休假,則:


1 下次兩人在同一天休假是______天後。


2 下次兩人同在星期四休假是______天後。

20. 求 108 與 135 的最大公因數 A:_______

21. 求 23 ´ 52 ´ 7 2  與 24 ´ 5 ´ 73  的最大公因數 A:_______


2 2 
22. 求 3 ´ 5 ´ 7 與 105 的最大公因數 A:_______

23. 甲、乙兩個警員繞校圓巡邏,甲每 1 小時一次,乙每 100 分鐘

一次,若甲、乙兩人繞校圓巡邏,現在在校門口相遇了,下次

在校門口相遇最久需要經過_________分鐘後。

24. 若 a 為正整數,648 除以 a 餘 18,747 除以 a 餘 12,則 a 可以

是那些數?A:____

25. 求 28、70、140 的最大公因數_____________


26. 有 64 個男生與 48 個女生混合編班,每一班的男生人數、女生

人數都要一樣多,請問最多可以編成_______個班。

27. 求 48 與 84 的最小公倍數_____________

4  2  2  3 
28. 求 2  × 3  × 5 與 2  × 3  × 7 的最小公倍數_____________

29. 求 144 與 2 3 ´ 3 2  ´ 5 的最小公倍數_____________

30. 求 120、96、30 的最小公倍數_____________

31. 小瑄用長 8 公分、寬 6 公分、高 4 公分的積木堆成一個實心的

正方體,所有的積木都依相同的方向排列,請問這個正方體的

邊長最小是_______公分

32. 小瑄與小玫、小吾跑操場,小瑄 3 分鐘跑操場一圈;小玫 5 分

鐘跑操場一圈;小吾 6 分鐘跑操場一圈。若他們三個從操場的

起跑點向同ㄧ個方向出發慢跑,請問再過______分鐘,他們三

人才能再次相遇在起跑點。

33. ( [24 , 18 ] ,  48 ) =________

34. [25 , (36 , 40 )] =________

35. 求 9、99、999 的最大公因數________

36. ○
1 求 3、6、9 的最大公因數________


2 求 3、6、9 的最小公倍數________

37. ○
1 (36 ,60 ,120)=________○
2 ﹝36 ,60 ,120﹞=________
38. 請以標準分解式表示:
( 2 5 ´ 3 , 3 2 ´ 5 , 2 ´ 5 3 ´ 7 )= 、

[ 2 5 ´ 3 , 3 2 ´ 5 , 2 ´ 5 3 ´ 7 ]=

3 2 2 2 3
39. 設甲=2 ×3 ×5 ×11,乙=2 ×3 ×7×11,以標準分解式表示

則(甲,乙)= ;[甲,乙]=

40. (35×72×11,23×34×7)= (標準分解式)

41. A=4×5、B=2×14、C=35,求[A,B,C]= (標準分解式)

42. 從 100 到 300 的整數中,能同時被 20 和 28 整除的數有哪

43. 請以標準分解式表示:
(6×15×20×35,8×12×15×49)=   、

[6×15×20×35,8×12×15×49]=  

44. 1547、1983、217481、31482 四數中,哪些數不是 3、9 和 11

的公倍數?________

45. 求 145 與 87 的最小公倍數_________ 。

46. 求 200、400、700 的最小公倍數_________ 。

47. 求 2 4  ´ 3 2  、 2 3  ´ 3 3 、 2 2 ´ 5 的最小公倍數_________ 。

48. 求 210  ´ 3 4  、 310  ´ 5 4  的最大公因數_________ 。

49. 求 2 5 ´ 3 2  ´ 7 、 5 4  ´ 7 2 的最小公倍數_________ 。

50. 若 A = 2 ´ 3 ´ 5 ´ 6 、 B = 3 ´ 4 ´ 5 ´ 7 ,則(A,B)=_________ 。

51. 有一個二位數可同時被 5、10、15 整除,請問滿足此條件的正


整數有________________。

解答:
3 3 2 2 2
1.1、7、11、13 2.○
1 3 ○
2 1260 3.○
1 45 ○
2 2 ×3 ×5 ×7 ×11
4 3 4
4.2 ×3 ×5×7 ×13 5.45 6.45 7.7 種 8.39 9.72 10.118
11.1220 12.120 13.67 處 14.○
1 12 ○
2 7 ○
3 21
2 3 2 2 3 2 2 3
15.○
1 5 ×7 ×13 ○
2 2 ×3 ○
3 2 ×5 16.○
1 1、5、5 、5 ○
2 1、2、14、28
4 2 3 2
17.○
1 144 ○
2 720(or 2 ×3 ×5) ○
3 2 ×3×7
2 3 4 3 2 4 2
18.○
1 2 ×3 ×5×7○
2 2 ×5 ×7×11 ○
3 2 ×3 ×5×7 19.○
1 20 ○
2 140
3 2
20.27 21.2 ×5×7 22.105 23.300 24.21,35,105
4 3 4 2
25.14 26.16 班 27.336 28.2 ×3 ×5×7 29.2 ×3 ×5=720
30.2760 31.24 公分 32.30 33.24 34.100 35.9
5 2 3
36.○
1 3 ○
2 18 37.○
1 12 ○
2 360 38.1、2 ×3 ×5 ×7
2 2 3 3 2 4 2
39.2 ×3 、2 ×3 ×5 ×7×11 40.3 ×7 41.2 ×5×7 42.140、280
5 2 3 3 2
43.2 ×3 ×5 ×7、2 ×3 ×5×7 44.1547、1983、217481 45.435
4 3 4 5 2 4 2
46.5600 47.2 ×3 ×5 48.3 49.2 ×3 ×5 ×7
2
50.2 ×3×5=60 51.30、60、90

三、 綜合、題組題

2 a b 4 3 3 c
1. 已知 a、b、c 為正整數○
1 若(2×3 ×5 ,2 ×3 ×5 ×7)=2 ×3 ×5,

2 a 5 3 b 4
則 a= ,b= ,c= 。 ○
2 [7 ×13 ,2 ×7 ×11 ×13 ]=

23×73×112×135,則 a= ,b= ,c= 。

2. ○
1 一張全開的壁報紙的長為 108 公分、寬為 78 公分,如果想要

以公分為單位,全部裁剪成邊長相等的正方形,則正方形的邊

長最大可為多少公分(請填整數)?這樣的正方形可以剪出幾
個?


2 暑期數學營活動,共有學生共有 162 人參加,其中男生 90

人、女生 72 人,要編列小隊,每小隊同時有男生和女生,而且

各小隊男生人數一樣多、女生人數也一樣多,請問最多可分成

幾小隊?每小隊各多少人?


3 有一批水果,蘋果有 48 個,梨子有 24 個,水蜜桃有 36 個,

想要裝成水果禮盒,每盒中,每種水果的個數分別相同,則最

多可以裝成幾盒,每盒中各有幾個蘋果、梨子和水蜜桃?

3. 求下列各組數的最大公因數和最小公倍數:


1 2 × 3 × 3 × 3× 7,2 × 2 × 5× 5。

4 2 3 2 3

2 2 × 3 ,2 × 3 × 11,2 × 3 × 5 。


3 96,60。


4 132,176。


5 108,72,90。

4. ○
1 求 5、7、9 的最大公因數


2 求 5、7、9 的最小公倍數

5. ○
1 將 16 與 27 作質因數分解


2 求最大公因數(16 ,27)


3 求最小公倍數﹝16 ,27﹞
6. 快樂國中有男生 360 人,女生 336 人,參加校外教學活動。若

男女生分別分組,每組人數一樣多,而且每組人數必須超過 7

人,則:


1 以每組人數來區別,有那幾種編組分式?


2 每組最多幾人?此時,男女合計共可以編幾組?

7. ○
1 12 的所有質因數


2 小於 12 的正整數中,和 12 互質的數有那幾個?

8. 互質的兩個正整數一定都是質數嗎?請說明理由或舉例。

9. 將下列各數與 28 互質的圈起來:

2、3、5、7、9、10、14、36、45

解答:
1. ○
1 a=1,b=3,c=4 2 a=5,b=2,c=5
○ 2.○
1 26 公分、7 個
2 18 隊、9 人
○ 3 12 盒、蘋果 4 個、水梨 2 個、水蜜桃 3 個

2 3 2 4 3 3 5
3.○
1 2、2 ×3 ×5 ×7 ○
2 6、2 ×3 ×5 ×11 ○
3 12、480(2 ×3×5)
4 3

4 44、528(2 ×3×11) ○
5 39、780(2 ×3×5×13)

4.○
1 1、○
2 315 5.○
1  2  、 3 3  ○
2 1 ○
3 432 6.○
1 3 ○
2 29

7.○
1 2、3 ○
2 5、7、11 8.不一定。互質的兩個正整數沒有共同
的質因數:如(5,9)=1,(4,15)=1 9. 3、5、9、45

貳、 進階題
一、 選擇題

1. ( )高雄開往臺北的火車,自強號每 15 分開一班,莒光號每

20 分開一班,復興號每 36 分開一班,某日三種火車在上午 10

時 30 分同時開出,問最快何時再同時開出?

(A) 11 時 30 分(B) 12 時 30 分(C)13 時 30 分(D) 14 時 30 分

3
2. ( )設甲=2×3×4×5×6×7×8 ,乙=3 ×4×10×28,下列敘述何者

3
錯誤?(A) 甲共有 4 個相異質因數 (B) 3 ×2×5 是甲的因數

5 2 7 6 2
(C) (甲,乙)=2 ×3 ×5×7 (D) 2 ×3 ×5 ×7 為甲、乙的公倍數

3. ( )下列敘述何者正確?

(A)因為 36 和 25 沒有公因數,所以 36 和 25 互質

(B)因為(12,32,75)=1,所以 [12 , 32 , 75 ] =12×32×75

6 4 1  3 2
(C)2 ×3 ×  是 2 ×3 的倍數

(D)20 是 2×3×52×7 的因數。

解答: 1.C 2.B 3.C

二、 填充題
1  1 1 
1. 若  ,  ,  分別乘以正整數 A 後均變為整數,且 A 為三位數,
12  18  21 
則 A 的最小值為 ,最大值為

2. 若 1/(22×52×73)
,1/(23×5×72)
,1/(3×52×7)分別乘以正整數

A 後均變成整數,則 A 的最小值為 (請以標


準分解式表之)

3. 皮卡丘到超商買若干巧克力,只知 6 包ㄧ數,5 包ㄧ數,8 包ㄧ

數都沒有剩下,問皮卡丘至少買______包巧克力。
(不可以一包

都不買喔)

4. 有一百元、五百元、一千元新台幣各 45、75、90 張,要分裝於

紅包袋中,每袋中的各種錢幣數量都要一樣多,問每個紅包最

多多少錢 ,可分裝 個紅包。

5. 90、87 與 x、30 這兩組數,有相同的最大公因數及相同最小公

倍數,求 x=__________

6. 小扁、連發、宋禮三人同時同地出發按順時針方向繞周長 3960

公尺的公園而走,小扁一分鐘可走 660 公尺,連發一分鐘可走

220 公尺,宋禮一分鐘可走 198 公尺,若三人想於原出發點會

合,則最少 分鐘後。

7. 觀光道路上,為了美化,要在道路「兩邊」種行道樹和設置路

燈。每 8 公尺種一棵行道樹,每 50 公尺設置路燈,有設置路燈

處,就不種行道樹。已知此路段為兩公里長,起始點處和路的

終點都是設置路燈。請問:


1 共設置了______盞路燈。 ○
2 共種了______棵行道樹。

8. 甲、乙、丙三家電視台,在每晚八點整會開始播出連續劇,播
放規則如下:甲臺每播 10 分鐘後就接著播廣告 2 分鐘;乙臺每

播 8 分鐘後就接著播廣告 1 分鐘;丙臺每播 17 分鐘後就接著播

廣告 1 分鐘。請問在晚上八點________分時,剛好三台廣告會

同時結束。

9. 要搭車從台北到新竹的客運有三家可選擇,從早上 5:30 開始共

同發出第一班車,甲家每 8 分鐘發一班,乙家每 15 分鐘發一班,

丙家每 12 分鐘發一班。請問:○
1 每隔____分鐘,剛好三家客運

都一起發車。○
2 從早上到晚上 10:30 為止,甲丙兩家共有_____

次同時一起發車。

10. 小瑄將 a、b 兩個正整數作質因數分解,完整的作法如下。已知

a、b、c、d、e 都是正整數,且 a、b 的最大公因數是 6;a、b

的最小公倍數 120,請求出 a+b 的值。

11. a 是一個正整數,其所有正因數中最大的為 36,則 a 與 120 的

最大公因數為______。

12. 若 - 的分子加上-4 之後,則分母應該加上_____,其值不變
5

13. 若 a 是正整數,且 a 的正因數有 1、5、9、15,求 a 的最小值

_________。

1  1  1 
14.  、  、  分別乘以同一個正整數 n 之後,會變成整數,則 n
10  18  25 
的最小值為_________。

15. 若 a  = 3 2 ´ 5 ´ 91 , b = 3 ´ 5 2 ´ 7 3  ´ 83 ,則下列何者錯誤?

A:__________

(A) (a , b ) = 105  (B) [a , b ] = 3 2 ´ 5 2  ´ 7 3  ´ 83 ´ 91 

(C) a 和 b 共同的質因數有 3、5、7 (D)105 是 a + b 的因數

16. 小新有 1000 個長、寬、高分別為 10 公分、6 公分及 5 公分的

長方體積木,他要堆成一個實心正方體(積木之間,以面積相

同的兩面相接),則:


1 若要堆成最小的正方體,則其邊長為幾公分?至少需要幾個

積木?


2 若以 1000 個積木為限,要堆成一個更大的實心正方體,至少

需要幾個積木?

17. 下圖是由 A、B、C 三個長方形及正方形 D 所拼成(無空隙)的

大長方形,它們的邊長都是整數,而且 D 的邊長是質數,如 A

的面積為 35,C 的面積為 56。那麼此大長方形的周長為_______。

18. a,b,c 為整數,短除法過程如下:求○


1 a= ○
2 [a,b,48]=
19. 阿木將兩個正整數 a、560 作短除法的部分過程如下,如果過程

中沒有計算上的錯誤,求(a,560)=_______。

20. 已知用短除法求 a、 b、 c 三正整數的最小公倍


2  a,  b,  c 
數的計算過程如右,若 a、 b、 c 三數的最大公 2  d,  e,  f 
3  g,  h,  i 
2
因數是 44,且 a = 2 ´ 5 ´ 11,求:(1) a、 j,  k,  l 
11 
5,  3,  2
b、 c 三數的最小公倍數。
(使用標準分解式作答)

(2) 指出 g、 h、 i 三數中何者不是 3 的倍數?何者是 3 的

倍數?

解答:
1.最小值 252、最大值 756 2. 23×3×52×73 3.120 包
4.8800 元,15 個 5.261 6.180 分鐘 7. ○
1 82 ○
2 480
8.8:36 9.○
1 120 ○
2 43 10.54 11.12 12.10
13.45 14.450(找 10、18、25 的最小公倍數) 15.B
1 30 公分,90 個○
16.○ 2 980 個 17.54 18.○
1 36 ○
2 720
3 2
19.280 20.○
1 2 ×3 ×5×11 ○
2 g 不是,h、i 是 
三、 綜合、題組題

1. ○
1 若一個三位數同時可以被 28、42 這二個數整除,則這個三位

數最小是多少?最大是多少?


2 若一個三位數同時可以被 6、16、20 這三個數整除,則這個

三位數最小是多少?最大是多少?
5  3  7 
2. ○
1 某一個正整數,它與  、  和  的乘積都是整數,此正整數
6  8  10 
15  25 
最小為 。○
2 某一個正分數,它與  、  的乘積都
16  18 
是整數,此正分數最小為 。

3. 甲乙兩人在操場比賽 5000 公尺長跑,甲的平均速度的 100 公尺

20 秒,乙的平均速度是 1 分鐘可跑 250 公尺,操場一圈是 400

公尺,請問:


1 經過多久後,兩人剛好又在起點碰到


2 承上題,此時,甲跑了多少公尺,乙跑了幾圈?

144 
解答: 1 168、924 ○
1.○ 2 240、960 1 120 ○
2.○ 2 

1 480 秒
3.○ 2 3200 公尺、5 圈

主題三 分數的運算

3-3 正負分數的加減運算

壹、 基本題

一、 選擇題
5  7 
1. ( )媽媽買了  公斤的櫻桃和  公斤的蓮霧,則哪一種水果
7  9 
比較重?(A) 櫻桃 (B) 蓮霧 (C)一樣重 (D)無法比較

3  60  乙數
2. ( ) 已知  = =  ,則甲數 + 乙數 =?
4  甲數  24 

(A) 96 (B) 97 (C) 98 (D) 99

3. ( ) 小雄、小明、小玲合買了一個比薩,已知小雄先吃了全
1  3 
部的  ,小明再吃了全部的  ,小玲再把剩下的吃完,試問誰
4  8 
吃得最少?(A)小雄(B)小明(C)小玲(D) 三人吃得一樣多

4. ( )下列哪一個算式的答案與 - 7 相等?

2  2  3  æ 2 ö
(A) - 7 + (B) - 7 - (C) - 8 - (D) - ç 7 - ÷
5  5  5  è 5 ø
1  1 
5. ( )小明把收入的  拿來當生活費,  拿回家給媽媽,剩下的
4  3 
錢做投資用途,請問小明投資的錢占收入的幾分之幾?
3  7  5  9 
(A)  ˉ(B)  (C)  ˉ(D) 
15  15  12  12 
2  2 
6. ( )若 - 2 的相反數是 p,而 - 2 的倒數是 q,則 q + p =?
3  3 
55  55  24  24
(A)  (B) -  (C)  (D) - 
24  24  55  55 
4  12  8 
7. ( )已知 a=  、b=  、c=  ,試問 a、b、c 三數的大小
5  15  10 
關係為何?

(A) a<b<cˉ(B) b<c<a (C) c<a<bˉ(D) a=b=c

3  60  乙數
8. ( )已知  = =  ,則甲數- 乙數 =?(A) 96 (B) 72
4  甲數  24 

(C) 98 (D) 62

9. ( )有一塊年糕,甲螞蟻要花 15 天獨自搬完,乙螞蟻要花 20

天獨自搬完,請問甲螞蟻每天搬的量比乙螞蟻每天搬的量多出
1  1  1  1 
全部年糕的幾分之幾:(A)  (B)  (C)  (D) 
30  40  50  60 

10. ( )下列哪一選項和 -5  不相等? 
4 4 
3  3 
( A )( -5) + ( - ) (C ) - (5 + )
4  4 
3  3 
( B) - 5 -  ( D) - 5 +
4  4 
11  2  3  1 
11. ( )求  +(-  )-[(-  )-  ] 之值為何?
14  9  14  3 
1  10 
(A)  (B) 1 (C) 2 (D) 
9  9 
12. ( )下列各選項的分數運算過程,何者正確?
1  1  1  1  1  1 
(A)  -(  -  )=  -  + 
2  3  5  2  3  5 
1  3  1  1  3  1 
(B)  +(  +  )=(  +  )+ 
2  6  5  2  6  5 
1  1  1  1  1  1  1  1 
(C)  +  +  -  =  +  +  - 
5  7  10  14  5  10  7  14 
(D)以上皆正確

13. ( )下列各選項的分數之中,何者在數線上的位置最接近代
6  7  9  9 
表 1 的點?(A)  ˉ(B)-  (C)  ˉ(D)- 
7  8  10  10 

解答:

1.B 2.C 3.A 4.B 5.C 6.A 7.D 8.D 9.D 10.D

11.D 12.D 13.C

二、 填充題
1  5  1 
1. 計算 1  -  -  =ˉˉˉˉ
8  6  18 
2  1 
2. 數線上 A ( 2  )、B ( - 4 ) 兩點間的距離為______。
3  2 
3. 下列等式中,在括號內填入適當的數字使得等式會成立。

(1)
27

3
=
( ) - 4 (2) –
2

16 
45  ( ) + 2  15  3  3 ´ ( )
13 5 
4. 計算 ( - ) - ( - ) 的值:__________
16 16
1  5 
5. 計算 299 + 309  = 299 + 309 + a 求 a=______
99  99 
3 7 2 
6. 計算  - [ + (- )] 的值:____________
5 4 3
3 5 
7. 計算  + 的值:__________(答案若為分數,請以最簡分數表
2  3 
示)

8. 在空格中填入「>」
、「=」或「<」。
455 455 
-897    -897-  。
568  568 
9. 試比較下列各題的分數大小:
5  7  3  3  5  17 
(1)  和  (2)  和  (3) 1  和 
9  9  11  10  12  13 
3  4  14  25  30  20 
(4)  和  (5)  和  (6)  和 
5  7  91  75  36  24 
1  3  2  1 
10. 計算[(-  +  )]-[  +  ]=ˉ ˉˉ
3  4  5  7 
6  a b  c 
11. 若 2  = 2  = 1  = ,則式中 a+b+c =______。
10  5  15  10 

12. 已知(-m)+(-n)=-(m+n),若 ­ 2  = ­2 + c ,則 c=

5 6 
13. 計算 ( - ) + ( - ) 的值:___________
13 13

5  2 12 
14. 計算 -  - (  - ) = ____________
7  5  7 
13 2  1 
15. 計算  - (  + ) 的值:________(答案若為分數,請以最簡分
4  15  4 
數表示)

16. 在空格中填入「>」
、「=」或「<」。 

7 8 
  。
6  7 

17. 試比較下列各題的分數大小:

2  2  15  17  12  21 


(1)  1  和 1  (2)  和  (3)  1  和 
9  11  17  19  13  11 
3  4  5 
18. 計算甲= - 、乙= - 、丙= - 三數的大小關係為
2  3  4 

19. 試計算下列各式的值,並化成最簡分數。

11  7  1  5 
(1)  +  =ˉˉˉˉ。(2) 1  -  =ˉˉˉˉ。
36  36  6  6 

20. 計算 |5 1 - 4 1 |+|  1  1 
4  - 6  |+ [( - 4 ) - ( - 7 )] =  。
3  4  4  3 
3 2 4 
21. 計算 (-4 ) + 2 - 1  =
5 5 5
5 7 
22. 計算 (- ) + (- ) 的值。
3 4
2 3  1 
23. 將  + - 的值化成小數為______。
3  4  12 
6 2 
24. 計算  - 的值:_______(答案若為分數,請以最簡分數表示)
7  7 
1  2  1 
25. 計算 3 - 1  + 2  的值:___________(答案若為分數,請以最
2  3  4 
簡分數表示)

26. 求出下列各式的值:
1 2  6 3  1 5  1  4 13  2 
(1)  + (2)  - (3)  + + (4)  + -
3  5  7  4  4  6  9  5  15  3 
1  1  1  1 
27. 計算  +  2  +  3  +  4  =ˉˉˉˉ
2  2  2  2 
1  2  3 
28. 求 ( 5 + ) - ( 4 + ) + ( 1 + ) 的值 =____________。
2  3  4 
8  5 
29. 比較  與  的大小
13  9 
3 14 
30. 計算 ( - ) + 的值:__________
15 15

31. æç -  3 ö÷ - æç - 1 ö÷ + æç - 1 ö÷ 的值等於 。


è 4 ø è 2 ø è 6 ø

- 3 - 2 
32. 計算  - 的值:_____________(答案若為分數,請以最簡
15  15 
分數表示)
1  1  2 
33. 計算-(  -2  )+2  的值:_______(答案若為分數,請以最
5  3  3 
簡分數表示)
2  3 
34. 自然課做實驗,小明量得甲杯裝了 1  公升酒精,乙杯裝了 2  公
5  4 
升酒精,兩杯合起來裝了多少公升酒精?

35. 小香、小山、小鮮分別購買了不同包裝的礦泉水,已知小香買

的礦泉水是 500 c.c.、價格是 20 元;小山買的礦泉水是 800

c.c.、價格是 24 元;小鮮買的礦泉水是 1200 c.c.、價格是 30

元,則ˉˉˉˉ買的礦泉水最便宜。
x  y  x y 
36. 若 x、y、w、u、均為正整數, 規定  w  u  =  + ,則  1  2  之
w  u  3  5 

值為____
3 4
37. 比較  與  的大小
2 ´ 17  3 ´ 17 
3 7 
38. 計算  + (- ) 的值。
7 3

39. 化簡 9 19  - éê 35 + æç - 19  8  ö÷ùú 的值為 。


27  ë 36  è 27 øû

3 - 2 
40. 計算  + 的值。 (答案若為分數,請以最簡分數表示)
5  3 
10  5  1 
41. 計算  - (  -  )的值(答案若為分數,請以最簡分數表
11  143  11 
示)
1
42. 學測數學科合計有卅三題,作答時間為 1  小時,小常在前十題

1  5 
用去  小時,第十一到廿題用去  小時,請問他還有多少小時
4  12 
可以做剩下的題目?
8  甲 乙
43. 已知-  =-  =-1  ,則甲+乙=ˉˉˉˉ
5  10  5 
1  5 
44. 設 P 為正整數,滿足  - 4 < P <  9  的數有______個。
4  8 
3 1  3 4 
45. 計算下列各式的值(1)  + (2)  +
8  6  4  5 
3 15 
46. 計算 ( - ) - (- ) 的值。
13 13

47.  - 1 8  的相反數與 - 1 的倒數的和等於 (9)
9  9 
3  3 
48. 計算 81 - 39  的值。 (答案若為分數,請以最簡分數表示)
5  5 
5  3  5 
49. 計算-  -(2  -  )的值(答案若為分數,請以最簡分數表
6  4  6 
示)
1  3 
50. 三民國中全校學生的  是騎腳踏車上學,  是給家長接送,其
10  5 
餘的部份是住在學校附近,每天早上自行走路來學校,請問走

路到學校的學生是全校的幾分之幾?
5  7 
51. 已知 A+  =B+  ,則 A 與 B 的大小關係為何?
7  9 

52. 小明昨晚上網只做了兩件事,其中花了  小時上拍賣網買遊戲


機,  小時上網聊天,則小明共花了多少分鐘在網路上。

4  3  5 7 
53. 計算下列各式的值: (1) 1 + 3  (2)  + 1 
5  10  6  20 
3 5 
54. 計算 ( - ) - 的值。
4 9
3 13 
55. (-4  ) - ( - ) =   。(以小數作答)
10  25 
112  é 5  æ 62  24 öù
56. 計算 -  - ç - ÷ 的值。 (答案若為分數,請以最簡分
87  êë 29  è 87  29 øúû

數表示)
1 3  5 
57. 計算  + - 的值(答案若為分數,請以最簡分數表示)
13  13  13 
1  1 
58. 小常買了一個大比隡,小明吃掉  ,小雯吃掉  ,剩下給小常
3  4 
全部吃掉,請問小常吃了全部的幾分之幾?

6  2  ( ) 8 
59. 在下列空格內填入適當的數:(1)  = =  。(2) - 
9  ( )  18  18 

( ) 16 
=  = 。
9  ( ) 
1  2  9 
60. 已知 ( 3  + 2  ) =  5  ,則其中的 Δ 代表______。
12  D  12 
3  æ 1  2 ö
61. 計算 + ç + ÷ 的值
4  è 3  5 ø

3 4 5 
62. 計算  - (- ) + (- ) 的值。
2 3 4

63. 7  +  æç - 5  7  ö÷ =____________。(用小數作答)。
16  è 10  ø

64. 下列各選項何者錯誤?答:   (全對才給分)


3  2  5 
(1)最小的質數是 1 (2)0÷(-  )=0 (3)-9  的倒數為-9 
2  5  2 

(4)0 的相反數為 0 (5)(4,9)=1 (6)5 是 0 的一個因數


5 4  5 - 4 
(7)(-21)÷0=0 (8)  - = = 1 
6  5  6 - 5 
65. 計算下列各式之值

8 3 2  7 1 3 
(1)  + + = 。(2) 2 + - 2  = 。
5 19 5 12 27 12
53 29 5  1 33 4 5 
(3)  + 3 - 3  = 。(4)  + - + =
52 48 48 18 58 58 18

66.  是一個小於 1 的最簡分數,請問□裡可以填哪些正整數?
15 
67. 大毛、二毛、三毛和小毛四兄弟在一場籃球賽中,分別投籃 10、

11、12、13 次,結果分別命中 7、8、9、10 次,則此次比賽中,


命中次數
______的命中率最高。 (投籃命中率= )
投籃次數

æ 1  1 ö 2 
68. 計算 ç 3 + 4  ÷ + 2  的值
è 3  2 ø 3 
-5 3 -7 
69. 計算  - ( + ) 的值。
7 4 3
3 5  - 7 
70. 計算  - - (  ) 的值。(答案若為分數,請以最簡分數表示)
17  17  17 
4 5 
71. 在空格中填入「>」
、「=」或「<」。-    -  。
7  9 
5 ( ) 3 ( ) 4 ( ) 7 ( )
= = = =
72. (1) 13  78  (2) 13  91  (3) 3  87  (4) 22  66 
3  ( )  3  ( )  31  ( )  10  ( ) 
= = = =
6  78  7  91  29  87  33  66 

73. 計算下列各式之值

8 9 17  17 1 2 
(1)  + ( - ) = (2) 4 - (3 - 2 ) =
17 17 37 57 3 57
1 2 3 5  23 8 7 1 
(3) 3 - ( - ) + 2  = (4)  -( + + ) =
4 7 4 7 24 63 24 3
9 11 3 5 
(5) 3  - - - = 。
14 38 38 38
1  1 
74. 若一最簡分數的分母是 24,且此分數介於  與  之間,則此分
3  4 

數為______。

5 a  15 
75. 若 - = = 則 a= ,b=
7 42  b
4 2  1  3 
76. 計算下列各式的值(1)  - (2) 3 -
5  3  3  4 
1 1 
77. 計算 3 - (-2 ) 的值。
2 3
1 1  4 
78. 計算 - ( - 2  ) + 2  的值。
(答案若為分數,請以最簡分數表示)
7  5  5 

79. 在空格中填入「>」
、「=」或「<」。-4.21   -4-0.21

80. 請將下列各分數化為最簡分數

21  27  35  66  30  13 


(1)  (2)  (3)  (4)  (5)  (6) 
49  75  25  88  45  91 

81. 計算下列各式之值

1 3 1  7 11 1 
(1)  + + = (2) 2 + - 2  =
6 28 3 12 15 4
15 5 1 2  10 7 11 
(3)  - - - = (4) 5 - (2 + 1 ) =
13 24 8 3 21 96 28
19 20 1 
(5)  - ( - ) = 。
34 51 54
2 6 9 
82. 計算  + + =
7 7 7
1 1 2 
83. 計算  + + =
2 4 3
2 4 6 
84. 計算 1 + 2 + 3  =
7 7 7
1 2 3 
85. 計算 1 + 2 + 3  =
2 3 4
5  2  8 
86. 計算  -(  +  )=
3  5  6 
9  5  2 
87. 計算  +(  -  )=
4  2  3 
2 6 9 
88. 計算  - - =
7 7 7
8  8  25 
89. 計算  -(  +  )=
25  25  7 
1 1 2 
90. 計算  - - =
2 4 3
2 4 6 
91. 計算 1 - 2 - 3  =
7 7 7
1 2 3 
92. 計算 1 - 2 - 3  =
2 3 4
1 1 1 1 
93. 計算 2 - 3 + 4 - 5  =
2 4 8 16
1 1 1 1 
94. 計算  - + - =
2 4 8 16
7  5  1 
95. 計算  –(  +  )=
12  12  4 
7  5  1 
96. 計算(–  )–(  –  )=
6  6  2 
40  3  40
97. 計算(–  )+[ (- ) +  ]=
31  4  31 

13  1  9 
98. 計算(–  )+[2  +(–  )]=
11  6  11 

2  1  1  7 
99. 計算(  –  )+(  –  )=
5  3  3  5 
1  5  4  3 
100. 計算(–  +  )–(  –  ) =
2  3  3  2 
4  7  2  1 
101. 計算(–3  +4  )+(4  –3  )=
5  10  5  10 

102. 在空格中填入適當的數,使得-  =
2  ( ) = 5 
3  18  ( ) 
103. 為何分數的加減需作通分或是擴分?

2  甲數 -12 
104. 若- =  =  ,則甲數+乙數= 。
5  20  乙數 
6  a  4 
105. 設 a 為整數,而且 - >  >  ,則 a= 。
35  105  - 21 
106. 老師買了 5 個 12 吋的披薩請同學們吃,而且每個披薩都平分

成 8 片,若智智吃了 5 片,仁仁吃了 7 片,則他們兩人共吃了

多少個披薩?還剩下多少個披薩?

3  5 
107. (1)若 □+  =–  ,則 □ =
4  4 
2  4 
(2)若(–1  )-□=  ,則 □ =
5  5 

108. 鳴人計畫花數天時間完成一工程,第一天做了  ,第二天做 
10 
2  1 
,第三天又做了  ,問還有
5  4 
1  1  1 
109. 甲=–3  ,乙=-3 +  ,丙 =-3–  ,丁=–3.25。
3  2  2 
請依大小順序填入:
( )>( )>( )>( )。


110. 一隻螞蟻在數線上不知何處,向右行走 1  單位後,又向左行


走 3  單位,結果停留在原點上,求最原始的位置?

17  1  1 2  29
解答: 1.  2. 7  3. 3,13,-8,-8 4. - 5.  6. -
72  6  2  33  60 
19 
7.  8.= 9. (1)< (2)< (3)> (4)> (5)< (6)=

53  2  11 3  13 
- - 2 
10.- 420  11.53 12.  3 13.  13  14. 5  15.  15  16.>
1  1 
17. >、<、> 18. 甲<乙<丙 19.  、  20.2 21.-4
2  3 
41 4  1  11  3  35 
22.  - 23.  1 . 3  24.  25.  4  26.(1)  (2)  (3) 
12  7  12  15  28  36 
15  7  8  5  11  5
(4)1 27.  28. - 2 29.  >  30.  31.  -
16  12  13  9  15  12 
1 4  3  1
32.  - 33.  4  34.  4  公升 35.小鮮 36. - 37.>
15  5  20  15 
40 1  1 138  1 
38.  - 39.  28  40. - 41.  42.  小時 43.19
21  36  15  143  2 
13  11  12  208 
44.5 45.(1)  (2) 1  46.  47.  48.42
24  20  13  153 
3  3 
49. - 2 50.  51.A>B 52.44 分鐘
4  10 
1  11  47 1
53.(1) 5  (2) 2  54.  - 55.-37.8 56.1 57. -
10  60  36  13 
5  29  19 
58.  59. 3,12,-4,-36 60.3 61.  1  62. 
12  60  12 
63. -5.2625 64. (1)(3)(7)(8)

1 853 3 29 1 2 33
7 85 42353 53 1 29113  551 10 33
53 14 79 1 1 5 
- + + += +-= ( --+= 65.(1) 
)+++ == =2 -++(2) +== (3) 
-+ = = (4) 
+ = 66. 2、4、7、8、11、13、14
27 552 19
12 18
485 12
5848
5 58 512
52 18
192748
1819
3 18
48
27 27 58
52 582 523 2 6
1  73  5 
67. 小毛 68.  10  69.  70.  71. <
2  84  17 

72.30 ,39 ,21 ,39 ,116 ,93 ,21 ,20

82 2739 1 17 51723 81198 39 11 27 175311 20 


2 523
1  793 8113  1 3 13 5 1  7 
2-73.(1) 
- +2++ )++=-24 == =-(333- +++ - =-6=-=-- 3) (2) 
) +-(2
- =3+3 1(3) 
= =4- -=(5  -+(4) 
= + (5) 
) = 3  74. 
77 572417
4 3 37 7572417 4314
634175738
243773833 37
738243 24 147 63 387 338 6338 7 24 
2  7  35  6 
75.a=-30,b=-21 76.  、 2  77.  78.  4  79.=
15  12  6  7 
3  9  7  3  2  1 
80.(1)  (2)  (3)  (4)  (5)  (6) 
7  25  5  4  3  7 
7 1 7 11
3 11 1 19
1011 20
177 1311151 17 
19 11
51020116 11
12 2 197 20 1  1 5  17  5 
(2 ++1 + )+=-5= --=+281.
-+--1(1) 
= =-+(5(2) 
- -=-+1- (3) 
)==- 2 (4) 
-= 4  + (5) 
= 82.  83.  1 
966 12 2815 3 3421
46 51
396
1254
282813
4 28
3424
15215181528
543 1334 96 5196 54 27 7  12 
5  11  1 1  13
84.  7  85.  7  86.  - 87.  4  88.  -
7  12  15  12  7 
25 5 1  11  27 5 
89.  - 90.  - 91.  - 5 92.  - 4 93.  - 94. 
7  12  7  12  16  16 
1  1  3  9  4  11 
95.  - 96.  97.  - 98.  - 99.-1 100.  101. 
12 6  4 11 3  5 

( ­12  )  5
102. =
18  ( ­7.5  ) 

103. 讓分母一樣,才有相同的基準可比較或做加減,所以分數的

加減時需通分母,再進行分子的加減合併。

104.26 105.–19

3  7 
106. 兩人共吃了  個披薩、還剩下  個披薩
2  2 
11  1 
107. (1) -2(2) - 108. 未完成  109. 乙>丁>甲>丙
5 20 

110. 2 

三、 綜合、題組題


1. 下列圖形為七巧板(正方形)
,請塗上  的

面積。

2. 運用分數形式表示下列各除法算式的結果(請簡化成最簡分

數):(1)2 ÷ 5= 。(2) 5 ÷ 7= 。

(3)13 ÷ 8= 。(4) 21 ÷ 14 = 。

(5)33 ÷ 10= 。(6) 28 ÷ 49 = 。

3. 下列每一個圖形的面積均為 1 平方單位,而且被平分成若干等

分,請利用分數表示塗色的面積是多少個平方單位?

(1) (2)

4. 是非題:對的打○,錯的打×
- 1  1  1  1 
1.( ). - ( + ) = - 
2  3  2  3 
5  6  8
2.( ).  >  > 
14  21  35 
2  5  13  3  33  4 
解答: 1. 2. (1)  (2)  (3)  (4)  (5)  (6) 
5  7  8  2  10  7 
1  3 
3. (1)  (2)  4. (○)、(○)
4  5 

貳、 進階題

一、 選擇題
A 11 
1. ( )若將分數  的分子減去 5 之後,可以約分為  ,求 A =?
84 21 
(A) 47 (B) 48 (C) 49 (D) 50
2  3 
2. ( )已知 | 甲數 – 2  | + | 乙數 + 3  | = 0,則甲數+乙數
5  4 
8  7  7  8 
=?(A)  - 2 (B)  - 1 (C)  - 2 (D)  - 1
20  20  20  20 
5  6 
3. ( )設 a =  ,b = –  ,則下列各數中何者最小?(A)|a + b|
7  7 

(B)|a – b| (C)|a|–|b| (D)|a|+|b|


1  1  1 
4. ( )有一個山東大餅,阿呆吃了  ,阿花吃了  ,小毛吃了  ,
3  4  6 

則小寶吃剩下的一部分,但沒有吃完,則下列何者不可能是小
4  5  1 
寶吃的份量?(A)  個大餅 (B)  個大餅 (C)  個大餅
27  16  12 

(D)  個大餅

32  32 - 8  21  21 +乙 
5. ( )已知  = ,  =  ,則下列敘述何者正
24  24 -甲  39  39 + 13 

確?(A)甲=8 (B)乙=9 (C)甲+乙=-2 (D)甲-乙=-1

解答: 1.C  2.B  3.C  4.B  5.D

二、 填充題
10  9  8 
1. 比較下列各數大小:a-  ,a-  , a- 
9  8  7 
5  6  7 
2. 比較下列各數大小:  -2b,  -2b,  -2b
3  4  5 
4999  1  5001 
3. 比較下列各數大小:  ,  , 
9999  2  10001 
1  3 
4. 在  與  之間所有的分母為 12 的最簡分數之和為
6  4 
5. 有一個分數的分母為 36,已知其分子減去 5 以後,恰可約分成 

,則原分數為ˉˉˉˉ

3  5 
6. 已知 A+  =B+  ,則 A 與 B 的大小關係為何?
5  8 
7. |2 1 - 4 1 |+|  1  1 
4  - 6  |+ [( - 2 ) - ( - 7 )] = 
3  4  4  3 
21 
8. 將  的分子減去 7,那麼分母應減去多少,其值才會不變?
159 
2 3 
9. 若 a-0.5=0.4 ,  + b = , 請問 a-b 的值為何?
3  5 
14  16 
10. 一分數介於  與  之間,已知此分數之分子是 224,且分母為
15  17 
整數,求此分數為______。

11. 有一個分數的分母為 45,已知其分子減去 6 以後,恰可約分成



,則原分數為ˉˉˉˉ

23 27  29 
12. 試比較下列各數的大小關係: a  =  、 b = 、 c  = 。
15  19  21 
答:______。
16  17  18  7 
13. 比較下列各組數的大小:(1) -  、-  、-  。(2)  , 
15  16  17  10 

7 + 1 
,  7 - 1 
10 + 1 10 - 1 。


14. 小於 21 的正整數 n 中,使  為最簡分數的 n 共有______個。
21 
15. 光武飲料公司推出新包裝飲料,且有甲、乙兩種促銷方案。甲

方案只賣新包裝的飲料,新包裝與舊包裝飲料的價格一樣,但

新包裝每瓶的容量增加 30%。乙方案只促銷舊包裝飲料,但是

買三瓶送一瓶。若以原來三瓶舊包裝飲料的價格去購買,選擇

哪一個方案的飲料實際上會較為便宜?

16. 一數線上 A、B 兩點的距離是 4  個單位長,若 A 點所表示的數


是  - 5 ,則 B 點所表示的數是____________。

17. 有一個水密桃果園近日收成,阿明一人獨自採收,3 天內可採

完,小華一人獨自採收,6 天內可採完,如果兩人一起採收,

請問需多少日可採收完成。

1 1  1  1  1  1 
18.  - + - + - = ______。
4  3  5  4  6  5 
7  11  15
19. 若甲、乙、丙三數都是負數,且甲數 +  =乙數 +  = 丙數 + 
19  19  19 
則甲、乙、丙三數從大到小的次序是____________。

2 3 1 1 
20. 計算  - (- ) + (-1 ) - 2  的值。
3 4 6 4
11 2 9 
21. 計算  - 39 + (1 - ) 的值。
4 5 2
3 3  5  3 
22. 計算 - (  - ) - (  - ) =
7  4  8  7 
5  2  3 
23. 計算 2  - ( -3  ) - 4  =
28  21  14 
6  æ 48  48 ö
24. 計算 99  + ç 98  - 97  ÷ 之值。(請用最簡分數表示)
7  è 45  56 ø
2  1  1  5 
25. 計算 1 - 15  - ( 3  - 4  ) 的值(答案若為分數,請以最簡分數
3  4  2  6 
表示)

26.

除了 1 以外,如何利用這些數線拼出 1?

10  9  8  5  6  7 
解答:1. a-  > a-  > a-  2.  -2b>  -2b>  -2b
9  8  7  3  4  5 
4999  1  5001  13  29
3.  <  <  4.1 5.  6. A>B 7.9 8.53 9. 
9999  2  10001  36  30 
224  11  16  17  18 
10.  11.  12.a>b>c 13.(1)-  <-  <- 
239  45  15  16  17 
7  2  1 
(2)  7 + 1  >  >  7 - 1  14.12 15. 乙方案 16.  - 1 , - 10
10 + 1 10  10 - 1 7  7 
1  7  1 
17.2 天 18.  19.甲>乙>丙 20.-2 21. - 39 22. 
6  20  8 
5  16  1  1 1 1 1 1 1 
23. 1  24. 100  25. - 12 26.  + + - 3 × + 6 × - 3 × =1
84  15  4  2 3 4 6 8 9
主題三 分數的運算

3-4 正負分數的乘除與四則運算

壹、 基本題

一、 選擇題

æ 1 ö æ 1 ö
1. ( )下列哪一個選項所代表的數是 ç -  ÷ ¸ ç - ÷ 的值?
è 2 ø è 3 ø
3 3 
(A) - (B)  (C) - 6 (D) 6 
2  2 

2. ( )建華班上有 35 人戴眼鏡,戴眼鏡的人數占總人數的  ,

那麼建華班上有多少人?(A)49(B)25(C)35(D)60
4  4 
3. ( )計算 5 ´ + 8 ´ 的值為何?(A)3(B)4(C)5(D)6
13  13 
3 3
4. ( )「  ´32 +  ´18」的答案和下列哪一個算式的答案相
499  499 
3  3  3
等?(A) (  +  ) ´ ( 32 + 18 ) (B)  ´ ( 32 ´ 18 )
499  499  499 
3 3
(C)  ´ ( 32 + 18 ) (D)  ´ ( 32 ¸ 18 )
499  499 
5  7 
5. ( )將  和  分別乘上同一個正整數 x 之後,兩分數都變成
8  10 

整數,則 x 的最小值是多少?(A) 40ˉ(B) 30 (C) 20ˉ(D) 10

6. ( )下列哪個選項所得到的結果和其他選項不同?

1 1  2  1  1  2 1 1  2
(A)  ´ ( - ) ´ ( - ) 2  (B)  (- ) ´ ( - ) ´ (  ) 2  (C)  ´ ( - ) ´ (  ) 2 
2  3  5  2  3  5  2  3  5 
1  1  2
(D)  ( - ) ´ ( - ) ´ [ -(  ) 2 ] 
2  3  5 

7. ( )下列哪個選項所得到的結果和其他選項不同?
1 1  2  2  1 1  2  2  1 1  2 2 
(A) [ ´ ( - )] ¸ ¸ ( - ) (B)  ´ [( - ) ¸ ] ¸ ( - ) (C)  ´ ( - ) ¸ [  ¸ ( - )] 
2  3  5  3  2  3  5  3  2  3  5  3 
1 1 2  2 
(D)  - ´ [  ¸ ¸ ( - )] 
2  3  5  3 
7  1  1  3  5  3  1 
8. ( )  +  ÷ (  –  )= (A) 1 (B)  (C)  (D) 
12  12  4  4  12  4  2 
1  1 
9. ( )計算 18×(  -  )-3×(-9 )之值為何? (A) 31 (B) 23
9  3 

(C)-10 (D) 10

10. ( )下列何者正確? (A)(–66) ÷ 4 × 5 = (–66) ÷ 20

1  3  1  5
(B) 3×(  –1) =  –1 (C) 4 ÷  ÷ (– 5 )= 4 ÷ ( - )
5  5  7  7 
2  2 
(D)  - 5 = –5 + 
3  3 
2 5 2 2 2 
11. ( ) 6 ´ ( + ) = 6  ´ A + B ´ ,則下列何者正確?
3 9 3 3 3 
5  2  5  2  2  2 
(A) A =  ,B =  (B) A =  ,B = 6  (C) A =  ,B = 6 
9  3  9  3  3  3 
2  5 
(D) A =  ,B = 
3  9 

1  1  æ - 5 ö 35 
12. ( )求 - 3  - + (- 4 ) ´ ç ÷ 的值為何?(A)6(B)7(C) 
2  2  è 2  ø 2 

(D)20

3  2 
13. ( )大華星期天拿 1000 元外出,買書用去  ,將剩下的  買
10  7 

電影票,吃便當用去 120 元買冰用掉 60 元,大華回家還剩多少

元? (A) 320 元 (B) 300 元 (C) 280 元 (D) 260 元

解答: 1.B 2.A 3.B 4.C 5.A 6.B 7.C 8.B 9.B 10.C
11.B 12.A 13.A

二、 填充題


1.  乘上□為最小正整數,□=__________。
48 
2 15 4 
2. 計算  ´ ´ =
5 16 3
1 5 4 
3. 計算 2 ´ 1 ´ ( - ) =
7 16 3
2 3 14 
4. 計算  ¸ ¸ (- ) =
3 7 15
2 3 14 
5. 計算 1 ¸ (-1 ) ¸ (- ) =
3 7 15
10 3 1 
6. 計算 (- ) ´ ( - ) ¸ ( - ) =
9 4 3
1 3 1 
7. 計算 (-1 ) ´ 1 ¸ (- ) =
2 4 3
3 1 1 3 
8. 計算 (- ) + (- ) ¸ (- ) - (- ) =
5 3 6 10

9. 計算 0 . 115 ´ 10 3  =

7  1 
10. 計算 ¸ ( 0 . 25 ) 2  ´ =
2  8 
8 3  1  3 
11. 計算(1)  ´ = __________ _  (2)  2 ´ = __________ 
15  10  2  10 
1  20  3  5  9 
12. 計算(1)  (- ) ´ = __________ (2)  ( - ) ´ ´ ( - ) = __________ 
8  3  8  2  10 
1 3  ö 22 
13. 計算 æç ´ ÷´ = __________ 
è 17  11 ø 9 
2  1  1 8 
14. 計算 éê 4 ´ ( -5  )ùú ´ éê( -1  ) ´ ùú = __________ 
ë 3  2  û ë 7  11 û
2 6  1  1 
15. 計算(1)  ¸ ( - ) = __________  (2)  (-2  ) ¸ ( -3  ) = __________ 
3  5  3  2 
2  5  3 
16. 計算 (- ) ¸ ¸ ( - ) = __________ 
9  6  10 
1 3  2  2  1  1
17. 計算(1)  ´ ( - ) ¸ = __________ (2) 2 ´ 4  ¸ ( -1  ) = __________ 
6  14  7  3  2  5 

18. 計算(1)  ( -3  )的倒數為 __________  (2)  0.35 的倒數為 __________ 


19. 在一包柏蒂全口味豆當中共有 60 顆豆子,其中  為檸檬口味、 

1  1  1 
為爆米花口味、  為肥皂口味、  為鼻屎口味其餘為嘔吐口
4  5  6 

味;若波特吃了 5 顆檸檬口味、4 顆爆米花口味、3 顆肥皂口味、

3 顆鼻屎口味後,嘔吐口味的豆子是剩餘的____倍。


20. 有六瓶相同的礦泉水,已知胖虎每天喝掉  瓶,則共可喝_天

21. 求下列各式的值:(1) æ 1 ö 3  (2) 1 ´ æç - 13 ö÷ = (3) æç -  5  ö÷ ´ 2  =


ç -  ÷ ´ =
è 2 ø 4  7  è 4  ø è 11 ø 3 

22. 求下列各式的值:
6  æ 1 ö
(1) ´ ç - ÷ =
5  è 2 ø
6  5 
(2) æç -  ö÷ ´ æç - ö÷ =
è 5 ø è 3 ø
64  5 
(3) æç -  ö÷ ´ =
è 25 ø 8 

23. 求下列各式的值:
4  æ 1 ö 5 
(1) ´ ç - ÷ ´ =
15  è 2 ø 3 
16  1  25 
(2) æç -  ö÷ ´ ´ æç - ö÷ =
è 5  ø 12  è 3  ø
21  æ 4 ö æ 15 ö
(3) ´ ç - ÷ ´ ç - ÷ =
20  è 3 ø è 7  ø

24. 求下列各式的值:

3  4 
(1) ( -  )  ´ =
4  3 
20 3  4 
(2) éê ´ ( - )ùú ´ =
ë 27  4  û 3 
20  3  4 
(3) ´ ( - ) ´ =
27  4  3 

25. 在下列各空格中,填入各數字的相反數:

2  6 
(1)  的相反數=__________(2)  (- ) 的相反數=_____
3  7 

(3)  (-1  ) 的相反數=______ (4)  10 2  的相反數=_____

26. 求下列各式的值:

æ 1 ö 3  1  æ 3 ö 2 
(1) ç -  ÷ ¸ = (2) ¸ ç - ÷ = (3) (- 5 ) ¸ =
è 3 ø 4  8  è 4 ø 5 

27. 求下列各式的值:

(1) æ 2 ö 4 
ç - 2  ÷ ¸ =
è 3 ø 5 

1 4 
(2) 1  ¸ æç - ö÷ =
6  è 7 ø
1  2 
(3) æç - 5  ö÷ ¸ ( -4  ) =
è 2 ø 5 

28. 阿明平時走路時,平均每一步的距離為  公尺,如果阿明從一

年二班教室走到福利社要走 120 步,則一年二班教室到福利社

的距離為________公尺。

3  9 
29. 一個長方形的面積為 6  平方公分,已知其長為  公分,則其寬
4  2 

為____公分。


30. 一包糖果裡有 120 顆糖果,其中蘋果口味佔了  、草莓口味佔


了  ,其他為葡萄口味,請問這包糖果裡葡萄口味的糖果有幾

顆?

3  25 
31.  的倒數和  的倒數的乘積為何?______。
5  9 
1  1  7 
32. ( 1 +  ) ÷ (  – 1 ) ×  =______。
4  4  15 
1  1  1  4 
33.  –  × 1  + 1 ÷ ( –  )=______。
4  2  4  3 
3  3  2 
34.  ÷ ( –  ) – ( – 9 ) ×  =______。
7  7  3 
5  3  7  1 
35.  –  × (  +  )=______。
6  5  12  4 
4  3  5 
36. 1 –  ×  ×  =______。
15  7  6 
1  1  3 
37. 1  ÷(  ×  + 1)=______。
3  2  4 
2 3  2 
38. - 5 ´(  - )= ______。
3  10  15 
1 1  1  3 
39. (  - ) × ( – 1  ) ÷ 1  =______。
2  7  2  7 
1  5  4 
40. ( - 4 ) ×  ÷ ( – 1  ) =______。
5  7  5 
2  7  1 
41.  –  ÷ [ ( 2 –  ) × 6 ] =______。
5  2  3 
1  1 
42. 9 ÷ [ 1 – (  +  ) ] =______。
6  2 
1  5  5  3 
43. ( –  ) ÷  ×  ÷ ( –  )=______。
7  42  6  4 
7 8  10 6 
44. 計算下列各式的值(1)  ´ (2)  ´
10  21  9  5 
8 27  14 13 
45. 計算下列各式的值(1)  ´ (2)  ´
54  35  39  28 
3  2 
46. 計算 5 ´ 的值
4  7 
9 4 
47. 計算  ´ 9  的值
10  9 
3  1 
48. 計算  5 ´ 3  的值
7  2 
7 9  ö 6 
49. 計算 æç ´ ÷ ´ 的值,並以最簡分數回答
è 13  21 ø 15 
24  æ 7  9  ö
50. 計算 ´ ç ´ ÷ 的值,並以最簡分數回答
35  è 5  12 ø
2 4  3 5 
51. 計算下列各式的值(1)  ¸ (2)  ¸
3  5  5  6 
4 2  8 3 
52. 計算下列各式的值(1)  ¸ 2  (2)  ¸ 9 
5  3  9  2 
3  2  3  5 
53. 計算下列各式的值(1) 3 ¸ 2  (2) 4 ¸ 2 
4  3  5  3 
4  3  5 
54. 計算 5 ´ ¸ 的值
5  4  3 
26 5  1 
55. 計算  ´ ¸ 3  的值
3  6  3 
2  2  4 
56. 計算 3 ¸ ´ 的值
3  3  9 
5 2  15 
57. 計算  + ´ 的值
7  3  4 
3 2  1 
58. 計算  - ´ 的值
5  5  6 
1 5  3 
59. 計算  + ´ 9 - 的值
3  6  4 
1 2  5 
60. 計算  - ´ 9 + 的值
4  3  8 
1 3  4 
61. 計算  ´ + ´ 7 的值
5  4  5 
4  2 
62. 若黑繩長  公尺,白繩長  公尺,請問黑繩是白繩的幾倍長?
5  3 

63. 計算下列各式,並以最簡分數回答
1  3  2  1  1  3  2  1 
(1) æç10  -  ö÷ ´ æç + ö÷ (2) 10 - ´ +
è 5  5 ø è 3  4 ø 5  5  3  4 
2 1  7  1  2 4 
64. 計算下列各式的值(1)  ´ + ´ (2) æç + ö÷ ´ 3 
5  3  5  3  è 3  7 ø
2  6 
65. 有一條繩子,它全長的  是  公尺,求繩子的全長是多少?
5  7 

66. 一瓶汽水,喝掉了  瓶,還剩下 750 ㏄,請問原來有多少㏄的

汽水?

3  7 
67. 農夫有一座 150 公畝的農場,其中  是草地,草地中有  做走
5  45 
道供遊客行走,請問走道有多少公畝?

4 2 
68. 計算  ´ (- ) 的值。
7 3
9 20 
69. 計算 (- ) ´ (- ) 的值。
5 3
12 15 4 
70. 計算  ´ (- ) ´ ( - ) 的值。
5 16 9
5 9 4 
71. 計算 (- ) ´ (- ) ´ (- ) 的值。
3 2 15

72. 計算  ¸ (-4) 的值。
3
1 4 5 
73. 計算 (-2 ) ´ ¸ 的值。
3 7 6
1 21 2 
74. 計算 7 ¸ (- ) ¸ 4  的值。
5 10 7
6  3  2 
75. 計算 ( -  ) ¸ - ´ ( -1 . 5 ) =
7  5  7 
7 26  35 6 
(1)(- ) ´ (- ) =  (2)(- ) ´ =
76. 計算下列的值  13 21 18 7

12 4 15 
77. 求  ´ (- ) ´ 的值
5 3 16
3 20 1 
78. 求 1 ´ ´ ( -4 ) 的值
4 21 5
2 4 
79. 求下列各數的倒數 (1) - (2)2 (3)1.25 
3 5
3 2 
80. 求 (- ) ¸ 的值
4 3
12 4 5 
81. 求 (- ) ¸ ´ ( - ) 的值
11 33 6

82. 求 (-3 ) ¸ (-5) ´ 3 的值
4
5 2  9 
83. 計算 æç - ö÷ ´ ´ 的值
è 6 ø 15  10 
25  9  1 
84. 計算 æç -  ö÷ ´ 3  ´ æç - ö÷ 的值
è 3  ø 10  è 26 ø
3 1  4 
85. 計算  ¸ 1  ´ 的值。(答案若為分數,請以最簡分數表示)
4  2  9 
1  4 
86. 甲、乙、丙三人合吃一個西瓜,甲吃掉  ,乙吃剩下的  ,丙
3  7 

把剩下的吃完,問那一個人吃得最多?

87. 3.75÷(-6.25)=   。(以最簡分數作答)

88. 1. 875 ¸ ( -0 . 75 ) = __________  (用最簡分數作答)

3 1  4 
89. 計算  ¸ 1  ´ 的值(答案若為分數,請以最簡分數表示)
4  2  9 
5  7  5 
90. 計算 5 ´ ´ 1  的值(答案若為分數,請以最簡分數表示)
6  25  49 
2  - 8 ö æ - 3 ö
91. 計算 2  ¸ æç ÷¸ç ÷ 的值(答案若為分數,請以最簡分數表示)
5  è 15  ø è 4  ø
- 1  - 2  3 
92. 計算 ( ) ´ (  ) ¸ 的值(答案若為分數,請以最簡分數表示)
2  3  5 
1  æ 1  1 ö
93. 計算 ¸ ç + ÷ 的值(答案若為分數,請以最簡分數表示)
12  è 3  2 ø
1  1 
94. 計算 7 - ´ 7 的值(答案若為分數,請以最簡分數表示)
3  3 
5  5 
95. 計算 197  ×65-65×97  的值(答案若為分數,請以最簡分數表
8  8 

示)

5  3 
96. 計算(78+  )×  的值(答案若為分數,請以最簡分數表示)
6  13 

97. 在下列空格中填入「>」、「=」或「<」。
1  1 
5×2  -3   5×(2  -3)
3  3 

98. 明明到大賣場買衛生紙,他看到乾淨牌衛生紙,一包有 425 張,

特惠價 25 元;他又看到柔軟牌衛生紙,一包有 320 張,但只賣

20 元,請計算這二種品牌的衛生紙每張價格差________元。
1  3 
99. 小傑看一本書,第一天看全書的  ,第二天看剩下部份的  ,
7  8 

若兩天共看 130 頁,則此書共有多少頁?

3  1  2 5 
100. 計算 4  -3  +1  ¸ 的值(答案若為分數,請以最簡分數表
4  2  3  6 

示)

3  4  3 4 
101. 計算 2 ´ -  ´ 的值(答案若為分數,請以最簡分數表示)
4  9  4  9 

102. 學期末本班班費結餘 4950 元,這筆錢預備用來舉辦同樂會,

1  1 
將全班分五組。計畫將這筆錢的  買 PIZZA,  買蛋糕,640 元
3  5 

買飲料,剩餘的錢平分給各組作為活動經費,每組可分得多少

錢?
74  4  49  4 
103. 計算求  ×  +  ×  的值(答案若為分數,請以最簡分數
123  5  123  5 

表示)
7  7 
104. 計算 187  ×56-56×87  的值(答案若為分數,請以最簡分數
9  9 

表示)

105. 甲、乙二好友各經營一個農場,有一天二人相見,甲對乙說:

「我用科學方法養雞,平均 5 隻母雞 4 天生 35 個蛋。」乙說:

「那有什麼了不起,我用祖傳秘方,平均 4 隻母雞 3 天就可以

生 20 個蛋,比你強多了。」則誰的方法比較會生蛋?


106. 將一大袋的糖平分裝成 7 小袋,其中每一小袋的重量為 1  公

斤,如果將這一大袋糖平分裝成 3 包,其中 1 包的重量為


公斤。

107. 小常從房間整理出舊的書籍要拿出去賣,5 公斤可賣 2 元,那

麼 57 公斤可以賣多少錢?(不用四捨五入,直接用帶分數表示

價錢)

108. PSP 掌上型遊戲機按定價打八折後的售價為 6400 元,請問原

來的定價為多少元?

109. 請算出下列各題的答案:

2  3  5  6  4 3  1  7  5  3  7  1 
(1) 2 ´ ¸ - (2)  ´ + ¸ (3)  –  × (  +  )
3  7  8  7  5  7  3  2  6  5  12  4 
1  1  7 
(4)(1+  ) ÷ (  –1) × 
4  4  15 

110. 一條繩子長 4  公尺,每 6/5 公尺剪成一段,請問共可剪成

幾段?

3 14 
111. 計算下列各式之值: (1) 1 ´ ´ 46 = 。
23 13
48 7 7  5 3 1 
(2)  ´ ´ = 。(3) 1 ´ 2 ´ 3  = 。
343 12 10 7 5 3

112. 計算下列各式之值:

12 4 3  12 3 4 
(1)  ¸ ¸ = 。(2) 6 ¸ 3 ¸ 2  = 。
25 5 5 125 5 5

113. 計算下列各式之值:

16 4 2  16 4 2 
(1)  ¸ ¸ = 。(2)  ¸ ( ¸ ) = 。
81 9 3 81 9 3

(3)承上,(1)、(2)的答案是否一樣?

114. 計算下列各式之值:
28 9 3  28 9 3 
(1)  ´ ¸ = 。(2)  ´ ( ¸ ) = 。
45 4 2 45 4 2

(3)承上,(1)、(2)的答案是否一樣?

115. 計算下列各式之值:

3 9 1  3 4 3 3 
(1) 1 ´ 14 + 2 ¸ 1  = (2) 2 ´ 1 - 2 ¸ 1  =
7 14 2 4 3 4 4

116. 計算下列各式之值:

56 5 3  1 
(1)  ´ ( + ) = 。(2)  (1313 + 78) ´ =
15 8 7 13
2 3  2 2 
(3)  998 ´ 4 + 998 ´ 5  = (4)  13 ´ 56 - 12 ´ 56 =
5 5 13 13

117. 計算下列各式之值:

39 52 13  31 31 
(1)  ( + ) ¸ = 。(2)  310 ¸ ( + ) =
7 17 119 3 7
19 19 95  95 19 19 
(3)  ( - ) ¸ = (4)  ¸ ( - ) =
11 12 132 132 11 12
1  15  5  5  5  63  17
解答: 1.48 2.  3. - 4. - 5.  6. - 7.  8. 
2  4 3 4  2 8  10 
12 3  5 27 
9.115 10.7 11.(1)  (2)  12. (1) -   
(2) 
25  4  6 32
2  64  5  2  8  1 
13.  14.  15.(1) - (2)  16.  17.(1)  - (2)-10
51  3  9 3  9  8
2  20  3 1  3 13 10
18.(1) - (2)  19. 
= 20.8 21.(1)  - (2) - (3) -
7 7  42 12 8  28  33 
3 8 2 20 
22.(1)  - (2) 2 (3)  - 23.(1)  - (2)  (3)  3 
5  5  9  9 
20 20 3  7 6
24.(1)-1(2) - (3)  - 25.(1)  (2) - (3) - (4)10 -2
27  27  2  6  7 
4 1 25 10 49 5 
26.(1)  - (2)  - (3)  - 27.(1)  - (2)  - (3) 
9  6  2  3  24  4 
3  3  7  9 
28.105 29.  公分 30.50 顆 31.  32.-  33.– 
2  5  9  8 
1  2  32  -1 3 5  1
34.5 35.  36.  37.  38.  39. - 40.  41. 
3  21  33  6  8  3  20 
4  4  1 4  1 
42.27 43.  44.(1)  (2) 1  45.(1)  (2) 
3  15  3  35  6 
9  1  1  6  18  5  18 
46. 1  47. 8  48. 9  49.  50.  51.1)  (2) 
14  2  2  65  25  6  25 
3  16  13  14  61  1 
52.(1)  (2)  53.(1) 1  (2) 1  54. 2  55. 2 
10  189  32  55  100  6 
4  3  8  1  1  3 
56.  2  57.  3  58.  59.  7  60.  - 5 61.  5 
9  14  15  12  8  4 
1  4  1  3  5 
62.是白繩的 1  倍長 63.(1) 8  (2) 
10  64.(1)  (2) 3  65. 
5  5  20  5  7 
1  8 
2  公尺 66. 1050 ㏄ 67. 14 公畝 68. = - 69. 12
7  21
1  8  4
70. = 1 71. = -2 72. = - 73.  _  74.  - 75. –1
12 5  5 
2  5 3 5  4  9
76.(1)  (2) - 77.-3 78.-7 79.(1) - (2)  (3)  80. -
3  3  2  14  5  8 
15  9  1  5  2 
81.  82.  83.-  84.  85.  86.乙 87.-0.6
2  4  5  4  9 
2  9  5  1 
88.-2.5 89.  90.  91.6 92.  93.  94. 5
9  5  9  10 
5  1  1 
95. 6500 96.  18  97. > 98.  99. 280 100.  3 
26  272  4 
8  4  1 
101.  102. 334 元 103.  104.5600 105.甲 106.  3  公斤
9  5  3 
4  2  46  1  7
107. 22  元 108.8000 元 109.(1)  (2)  (3)  (4) -
5  7  105  3  9 
127  2  104 
110. 4 段 111. (1)56(2)  112. 1  (3) 
210  35  7 
2  8  14  14 
113. (1)  (2)  (3)否 114. (1)  (2)  (3)是
3  27  15  15 
16  61  59 
115. (1) 21  (2)  116. (1)  (2) 107(3) 9980(4) 56
21  77  15 

117. (1) 79(2) 21(3)  (4) 5

三、 綜合、題組題
1. 拍賣網站上販售 Hello Kitty 胸章,售價如下:

花形胸章:每 3 個 40 元

方形胸章:每 5 個 35 元

圓形胸章:每 6 個 50 元

蝴蝶形胸章:每 5 個 60 元

根據上述資料,回答下列間題:

25  3  40 
(1)圓形胸章一個多少元? (A)  (B)  (C)  (D)12
3  25  3 

(2)那種胸章最貴? (A)花形 (B)方形 (C)圓形 (D)蝴蝶形

(3)一個花形胸章的價錢是一個方形胸章的幾倍?

35 21 40 21
(A)  (B)  (C)  (D) 
21  35  21  40 

2. 小丸子喝一瓶可樂,第一天喝下整瓶的  ,第二天喝下剩餘的 

30% ,第三天喝下前一天剩餘的一半,則小丸子第二天喝下整

瓶可樂的多少部份? (A) 30%  (B) 28%  (C) 26%  (D) 24% 

小丸子哪一天喝下最多可樂? (A)第一天 (B)第二天 (C)第三

天 (D)一樣多

13 7  6  17
最後剩餘整瓶可樂的幾分之幾?(A)  (B)  (C)  (D) 
50  25  25  50 

3. 索隆在便利超商打工,第一年薪資為 10000 元,第二年加薪 

10% ,第三年加薪 20% ,第四年減薪 30% 

(1)第一年調薪後,索隆的薪水為
(A)10000 (B)11000 (C)12000 (D)13000 元

(2)三年調薪後,索隆的薪水為

(A)10000 (B)12100 (C)8470 (D)9240 元

(3)若索隆三年的調薪順序改為減薪 30%,加薪 20% ,加薪 10% ,

則索隆的薪水與(2)相比將會

(A)增加 (B)減少 (C)不變 (D)不一定

4. 判斷下列推論是否正確,正確打「○」
;錯誤打「×」
,若為錯誤,

必須舉例說明其理由,否則不予計分。

舉例說明
題目 「○」或「×」
理由

兩個分子不為 0 的分數,若分母相

同,則分子愈大該分數愈大。

2  1 
例如:因為 2>1,所以  >  。
3  3 

7  6 
5. 分別用下列方法比較分數  、  的大小
6  5 
(1)將分子通分為相同的數

(2)將分母通分為相同的數

(3)將分數化為帶分數
6  5 
6. 分別用下列方法比較分數  、  的大小
7  6 
(1)將分子通分為相同的數

(2)將分母通分為相同的數

(3)將分數與整數 1 的關係列出

解答:1.(1)A(2)A(3)C 2.(1)D(2)C(3)B 3.(1)B(2)D(3)C

4. ×,若該分數為『負』,則結果相反

7  7 ´ 6  42  6  7 ´ 6  42 
5.(1)  =  =  <  =  = 
6  6 ´ 6  36  5  7 ´ 5  35 
7  7 ´ 5  35  6  6 ´ 6  36 
(2)  =  =  <  =  = 
6  6 ´ 5  30  5  6 ´ 5  30 
7  1  1  6  1  1 
(3)  = 1  = æç1 +  ö÷ <  = 1  = æç1 +  ö÷
6  6  è 6 ø 5  5  è 5 ø
6  6 ´ 5  30  5  5 ´ 6  30 
6.(1)  =  =  >  =  = 
7  7 ´ 5  35  6  6 ´ 6  36 
6  6 ´ 6  36  5  5 ´ 7  35 
(2)  =  =  >  =  = 
7  7 ´ 6  42  6  6 ´ 7  42 
6  1  5  1  1 1 
(3)  = 1 - >  = 1 - 因為(  á )
7  7  6  6  7  6 

貳、 進階題

一、 選擇題

1. ( ) 關於分數的算式,下列何者正確?
37 37  119 119 
(A)  ¸ 10 ¸ 5 = ¸ 2 (B)  ¸ 10 ´ 2 = ¸ 20 
123  123  163  163 
13  2  13  13  2  13 
(C) 28 ¸ = 28  ¸ 2 ´ 3 (D) 28 ¸ = 28  ¸ 3 ´ 2 
15  3  15  15  3  15 
2. ( )小新過年收到的壓歲錢共 x 元,他的分配使用方式如下:
其中一半的錢拿來做為儲蓄存款,另一半的錢則支出在添購一


些用品上,在小新添購的用品當中買新衣服用了  x ,買文具用

1  1 
品用了  x ,買遊戲卡用了  x ,請問下列何者正確?
6  12 

(A)小新買新衣服占所有支出的 

(B)小新買文具用品和遊戲卡所花掉的錢比買新衣服的錢多

(C)小新對於錢的分配使用當中,買新衣服所分配的金額最高

(D)以上皆非

3. ( )年終大特賣,A、B、C 三家商店推出原價為 x 元的相同商

品,各家喊出不同的銷售口號如下:A 商店:
「買一件 x 元,第

二件打五折。」B 商店:
「買一件打九折,第二件打六折。」C

商店:
「買兩件打七折。」如果想要買兩件此商品,到哪一家商

店買比較便宜?(A)A(B)B(C)C(D)三家價錢都一樣

4. ( )下列哪個選項所得到的結果和其他選項不同?
2 4  3  2  2 4  2  3 
(A)  ´ ( - ) ¸ ( - ) ¸ (B)  ´ ( - ) ¸ ¸ ( - ) 
3  7  2  7  3  7  7  2 
4  2  2  3  2 2  4  3 
(C)  (- ) ´ ¸ ¸ ( - )  (D)  ´ ¸ ( - ) ¸ ( - ) 
7  3  7  2  3  7  7  2 
b
5. ( )下列哪一個選項的結果與  ¸ ( -c ) 相同?

1  1 
(A)b÷a÷(–c)(B)b÷a×(–c)(C)b ÷(–c)÷  (D)b×(–c)× 
a  a 
2  2 
6. ( )  3  ÷ 4  可表示成下列哪一個式子?
5  3 
2  2  2  2  2  2
(A) 3×  ÷4×  (B)(3+  ) ÷( 4+  ) (C) 3+  ÷4+ 
5  3  5  3  5  3 
2  2 
(D) ( 3×  ) ÷( 4×  )
5  3 
2  2  2  2 
7. ( )  3  ÷ (-4  ) 可表示成下列哪一個式子?(A)3×  ÷(-4×  )
5  3  5  3 
2  2  2  2 
(B)(3+  )÷(-4+  ) (C)(3+  )÷(-4-  )
5  3  5  3 
2  2 
(D)( 3×  ) ÷(-4×  )
5  3 
5 5 1
8. ( ) ( - ) × [48 × ( - ) ÷ (-2)] ÷ ( - ) =?
9  8  2 
2  2  1  2 
(A)  15  (B)  16  (C)  17  (D)  14 
3  3  3  3 
3  3  3  B
9. ( )  (-5  ) ¸ = ( - 5  ) ´ A =  ´ A ,則下列何者正確?
7  8  7  7 
3  8 
(A) A =  ,B = 38 (B) A =  ,B = 38
8  3 
3  8 
(C) A =  ,B =-38 (D) A =  ,B =-38
8  3 
7  1  3  4  3  3  1 
10. ( )  –  ÷ (  +  )=(A) 1 (B)  (C)  (D) 
10  10  5  5  5  7  2 
5  7  2  1  19  3  11  5 
11. ( )計算  ×  – ( 2  – 1  ) =(A)  (B)  (C)  (D) 
7  3  3  2  6  2  6  2 
21  9 
12. ( )計算 14 ÷( –  ) – ( – 8 ) ×( –  ) =?
8  16 
59  59  5  3 
(A)  (B) –  (C) –  (D) – 
6  6  6  2 
1 3 1  13 
13. ( )計算  ´ [  - 1  ¸ 2  + 1 ] 之值為多少?
2  7  6  18 
1  125  5  1 
(A) –  (B) –  (C)  (D) 
2  343  14  2 
2  3  5 
14. ( )下列哪一個數學式子的答案和「  + ( –  ) ÷  」相等?
3  4  7 
2  3  5  2  3  7 
(A) [  + ( –  ) ] ÷  (B) ( –  ) + (  ×  )
3  4  7  3  4  5 
2  3  7  2  3  7 
(C)  + ( –  ) ×  (D) ( –  ) +  × 
3  4  5  3  4  5 
4 2  1  1  4  2 
15. ( )若 A + ( 1 ¸ 2  ) = 6  ,則 A = 6  ○ ( 1  □ 2  ),此時 ○
5 3 2  2  5  3 
和□應分別填入什麼運算符號?

(A) +、÷ (B) –、× (C) ×、+ (D) –、÷

11 1  1  11 1  1 
16. ( )設甲數 =  + 1  ÷  ,乙數 = (  + 1  ) ÷  ,則甲、乙兩
16  2  3  16  2  3 

數的關係何者正確?

(A) 甲數 = 乙數 (B) 甲數>乙數 (C) 甲數<乙數

17. ( )下列哪一個數學式子的答案和「– 7 ÷ ( 5 – 2 )」相

1  1  1  1 
等?(A) – 7 ×  – 7 ×  (B) – 7 ×  + 7 × 
5  2  5  2 
1 1  1 
(C) ( – 7 ) ×  (D) ( – 7 ) × (  –  )
5 - 2  5  2 

18. ( )附圖為一邊長 1 公分的正方形,將兩邊分成 4 等分及 6

等分,則白色部分面積可以表示成何者?(單位:平方公分)

1  1  1  1 
(A)1-  ×  × 7 (B)1-(  +  ) × 7
6  4  6  4 
1  1  7  7 
(C) 1-(  +  ) × 7 (D) 1-  × 
6  2  4  2 

19. ( )甲、乙、丙三人合吃一塊蛋糕,甲先吃全部的  ,乙吃剩


下的  ,丙再把最後剩下的吃完,那麼吃得最多的是誰?

(A) 甲 (B) 乙 (C) 丙 (D) 一樣多

20. ( )下列哪一個算式的計算過程是正確的?
1  1  3  3  1  1 
(A) 8 ¸ 5 ´ 2  = 8  ¸ 10 (B) 7 - 2 ´ = 5 ´ (C) 4 ¸ 2  = 2 
4  4  5  5  4  2 
3 1  3  4 
(D)  ¸ 2  = ´
5  4  5  9 

21. ( )由已知條件推測數字甲和乙、丙和丁、戊和己之間的大小
1  ö
關係,下列那個人的推測是正確的?小君:如果甲× æç - 1 ÷ =乙
è 15  ø
1  ö 1 
× æç -1 ÷ ,那麼甲>乙。小慧:如果丙×4=丁÷  ,那麼丙>丁。
è 14  ø 5 
4 3
小新:如果戊 - =己 - ,那麼戊>己。
5  4 

(A)小慧(B)小新(C)小君、小慧(D)小君、小慧、小新

解答:

1.C 2.D 3.C 4.D 5.A 6.B 7.C 8.B 9.D 10.B

11.B 12.B 13.D 14.C 15.D 16.C 17.C 18.A 19.A 20.D

21.B

二、 填充題

1. 比較下列各數大小:

2  3  4 
a×(-  ),a×(-  ),a×(-  ),其中 a<0
3  4  5 
23 
2. 計算 - 2  ´ ( -0 . 5 ) =
25 

3. 求下列各式的值:

9  17  63  97  67  31  3  5  8 


(1) ´ ( - ) ´ = (2) ( -  ) ´ ´ ( - ) = (3) ( - 1  ) ´ ´ =
28  63  17  31  15  97  4  14  25 
4  6  8  10 
4. 求下列各式的值:(1) (-  ) ´ ´ ( - ) ´ =
3  5  7  9 
28  3  5  1  1  3  5  8 
(2) ´ ( - ) ´ ( - ) ´ ( - ) = (3) ( - 1  ) ´ ( - ) ´ ( -2  ) ´ ( - ) =
15  7  2  2  2  4  6  15 
1  2  1  2  1
5. (1) ( -  ) 2  ´ ( - ) = 。(2) ( - 1  ) 3  ´ ( - ) 4  ´ (  ) 2  =
2  3  2  3  2 

6. 求下列各式的值:(1) 1  2  7  (2) 1  2 7 
( -  ) ¸ ¸ = ( -  ) ¸ (  ¸ ) =
3  7  6  3  7  6 

(3) 1  2 7  (4) 1  2  7 
( -  ) ¸ (  ´ ) = ( -  ) ´ ¸ =
3  7  6  3  7  6 

1 2  3 
7. A、B、C 三數都是正數,若 A ´  = B ´ = C ´ ,請比較 A、B、C
2  3  4 

的大小。

1 2  3 
8. A、B、C 三數都是正數,若 A ¸  = B ¸ = C ¸ ,請比較 A、B、C
2  3  4 

的大小。

9. A、B、C 三數都是負數,若

1 2  3 
(1)  A ¸  = B ¸ = C ¸ ,請比較 A、B、C 的大小。
2  3  4 
1  2  3 
(2)  A ´ 1 = B ´ 1  = C ´ 1  ,請比較 A、B、C 的大小。
2  3  4 

10. 附圖為一個邊長 3 公分的正方形,將兩邊分成 4 等分及 6 等分,

則陰影部分面積為多少_______?


11.  2  的相反數為 a,倒數為 b,則 a ´ b =?


12. 酒精連瓶重 900 克,用去酒精  後,剩下的酒精連瓶子重 500

克,則瓶子的重量是______克。

3  4  7  9 
13.  (- ) 、  、 (- ) 、 (- ) 四數中,任取三數相乘,則所得最大乘
4  5  6  8 

積為____?

5  9 
14. 寶媽有一天買了一個蛋糕,大寶吃了  ,二寶吃了剩下的  ,
24  19 

三寶吃的是二寶的  ,其他剩下的被小寶吃光,請問吃最多的

是誰?

15. 有一袋糖果,共有 180 顆,小光拿了全部的 1 多 3 個,小武拿




走剩下的  少 3 個,小倩拿走最後剩下的部分,請問小倩拿走
13 

了______個糖果。

3  10  X 
16. 小甜甜做一題分數乘除的計算題:
「  ×  ÷  」,計算的結果是 
8  21  4 

,則 X 的值為______。
14 
5  4 
17. 一條繩子長 14  公尺,如果每 1  公尺剪成一段,請問最多可分
8  5 

幾段?剩下幾公尺?

2  3 
18. 如果  袋米重 2  公斤,則一袋米重多少公斤?
3  4 
1  2  3  4  5  6  7  8  9 
19. ( -  ) ´ ( - ) ´ ( - ) ´ ( - ) ´ ( - ) ´ ( - ) ´ ( - ) ´ ( - ) ´ ( - ) =
2  3  4  5  6  7  8  9  10 
17  4  1  10 
20. (  –  ÷  ) × ( –  )=______。
10  5  3  7 
1  4 
21. 10 ÷[( 2 - )÷23]=______。
3  5 
甲 1  5 
22.  ´ 5 = 2  ¸ 乙 = ,則 甲 + 乙 =______。
4 4  2 
12 2  46 3 
23. 求出  ´ =______。
36 2  23 3 
4  4 
24. 12 × ( – 1000  ) + 12 × 999  =______。
9  9 

25. 101 × 123 × (  – 1 ) =______。
101 
1  1 
26. 一條橡膠水管剪去  後,再剪去剩下部分的  ,此時量得水管
3  4 

長度為 14 公尺,請問這條水管原來長多少公尺?
1  1 
27. 小云有存款 1000 元,買課外讀物花了全部的  ,剩餘的  買文
5  8 

具,其餘的還給媽媽,請問小云還給媽媽______元。

1 1 
28. 計算  + 的值,並以最簡分數回答
2 ´ 3 ´ 4 ´ 5  3 ´ 4 ´ 5 ´ 6 
2  1 
29. 兩個分數 5  、 4  分別乘以同一個正分數 n 之後,會變成整數,
3  4 

則 n 的最小值是多少?

10  9 
30. 比較分數-  、-  的大小
13  11 
1 3 5 7 
31. 計算 -3 ¸ 2 - ´ (- ) 的值。
4 5 7 15
2 1 1 7 
32. 計算 ( - 1 ) ¸ 3  ´ 的值。
3 5 3 4
2 3 1 
33. 計算 -3.5 - ( - ) ¸ 的值。
3 5 6
3 5  58 29 58 
34. 計算(1) 99 ´ 25 + ´ 25  (2) ( - ) ¸
8 8 55 45 15

35. 媽媽從超市買了一瓶 2400c.c.的牛奶,小嫻先喝了  瓶,弟弟


喝了剩下的  ,請問最後剩下多少 cc 牛奶?

1  1 
36. 將長 12  公尺的緞帶,以每 1  公尺的長度裁成一段,請問最多
4  3 

可以裁成幾段?剩下多少公尺?

1 1 
37. 計算 ( - 8 ) ¸ 4 ´ ( -2 ) - 0 . 4 ¸ (  - ) =
2  5 
1 2 2 2 
38. 求 (-2 ) ´ - ¸ 的值
2 5 3 9
2 1 1 4 
39. 求  + ( + ) ´ (- ) 的值
3 2 3 5
5 7 
40. 求 0.125 ´ 8 - (- ) ¸ 的值
2 5
4 5  3 
41. 計算 æç - ö÷ ¸ + 1 . 5 的值
è 5  4 ø 10 
42. 計算 æç -  2 ö÷ ´ 3 + 3 ¸ æç - 9 ö÷ 的值(答案若為分數,請以最簡分數表示)
è 3 ø 5  8  è 4 ø

3 1  2  1 
43. 計算  - ¸ 1  + ´ 3 的值(答案若為分數,請以最簡分數表示)
4  2  3  3 
3  1  é 2  1  ù
44. 計算 -  ¸ êæç1  + ö÷ ´ 3 ú 的值(答案若為分數,請以最簡分數表
4  2  ëè 3  3 ø û

示)
1  1  5  5 1 
45. 計算 9  -  ´ éê - (  - )ùú 的值(答案若為分數,請以最簡分數表
4  4  ë 3  6  2  û

示)

1 1  1 
46. 計算  - ´ ( -0 . 75 ) + ( - ) ¸ 0 . 125 的值(答案若為分數,請以最簡分
2  3  16 

數表示)

47. 化簡 æç 37  - 89 ö÷ ¸1  æ 14  11  ö


- 75 ´ ç - ÷
è 42  98 ø 14  è 45  175 ø

48. 將 10 公升的可樂,用固定每瓶 600 毫公升的容器來分裝,每瓶

都裝滿,請問可以分裝成幾瓶可樂?答: 瓶;最後剩下

毫公升的可樂。

49. 阿祥買了一瓶 1.25 公升的汽水,它倒了 4 杯每杯 240 c . c . 的汽水

請同學喝,請問他還剩下 毫升的汽水給自己喝?

3  1 
50. 有 9  公斤的米,每 1  公斤裝一包,可裝滿 包 ,還剩
4  2 

下 公斤的米不能裝滿一包。

2  7 
51. 某次段考、數學及格者佔全班的  ,英文及格者佔全班的  ,
3  9 

已知兩科皆不及格者,共有6人,佔全班的  ,問兩科皆及格

者共有多少人?數學及格但英文不及格者共有多少人?
15  15  15 15 
52. 計算 12 ´(–  )–19´  +  ´(–6)–8´(–  )的值(答案若
17  17  17  17 

為分數,請以最簡分數表示)

1  1 
53. 做一個 8 吋蛋糕需要  公斤的麵粉,現有 1  公斤的麵粉可以做
5  2 

出 個完整的 8 吋蛋糕,還剩下 公斤的麵粉。


é 1  ù 2  2 
54. 計算求 êæç 3 - 0 . 5 ö÷ ´ 7 - 5 ú ´ 1  - 的值(答案若為分數,請以最簡
ëè 2  ø û 3  3 

分數表示)
5  1  3  5  1  3 
55. 計算(  -  ÷  )÷[(  -  )÷  ]的值(答案若為分數,請以最
6  2  4  6  2  4 

簡分數表示)
1  25  25  25 
56. 計算(6  +  -  )÷(-  )的值(答案若為分數,請以最簡分
4  8  12  8 

數表示)
3  1  2 
57. 計算  ×4  ×(-  )×(-12)的值(答案若為分數,請以最簡
5  6  9 

分數表示)

58. 計算 3.2÷(-1.6)-4×(-1  )的值(答案若為分數,請以

最簡分數表示)

ìé 1  8  æ 1 ö ù ü
59. 計算 íê7 ´  - ´ ç 2 + ÷ ú + 5 2 ý ´ 0 - (- 1 ) 的值(答案若為分數,請
îë 5  11  è 5 ø û þ

以最簡分數表示)

3 4  2 
60. 下列各算式中,有哪些算式的答案與「  ¸ ¸ 1  」相同?
7  5  3 
3 4  3  3 5  5  3 5  3  3 4 3 
甲:  ¸ ´ 乙:  ´ ¸ 丙:  ´ ´ 丁:  ¸ (  ´ ) 
7  5  5  7  4  3  7  4  5  7  5  5 
3 4  3 
戊: ( ¸ ) ´
7  5  5 
1 1 1 1 1 
61. 1 ´ 1 ´1 ´ ×××××´ 1 ´ 1  = 。
2 3 4 49 50

2  3  4  73 
解答:1. a×(-  )< a×(-  )<a×(-  ) 2. 
3  4  5  50 
9 67  1 128 17 
3.(1) - (2)  (3)  - 4.(1)  - (2)-1(3) 
28  15  5  63  10 
1 1 49 4
5.(1) - (2) - 6.(1)-1 (2)  - (3)-1 (4)  -
6  6  36  49 
21 
7. A>B>C 8. A<B<C 9. (1) A<B<C(2)A>B>C 10. 

21 
11. -1 12.180 克 13.  14.二寶 15. 57 顆 16.2
20 
9  1  1
17.(1)8 段(2)  公尺 18. 4  公斤 19. - 20.1 21.150
40  8  10 
29  8 
22.  23.  24.–12 25.–12300 26.28 公尺
10  9 
1  12  11 1
27.700 元 28.  29.  30. > 31.  - 32.  -
90  17  12  5 
39 7  1 
33. - 34. (1)2500(2)  35.1000c.c 36.9 段 剩  公尺
10  66  4 
8  39  7 29 
37.  38.–4 39.0 40.  41.0 42. - 43. 
3  14  30  20 
2  107  1 
44.  45.  46.  47.–19 48.16 瓶;400 毫公升
3  12  4 
3  435
49. 290 50. 6 包;  公斤 51. (1)22(2)2 52.  -
4  17 
1  3  7 20 
53. 7 個;  公斤 54.26 55.  56. - 57.  58.4 59.1
10  8  3  3 
51 
60.甲、乙、丙、戊 61. 

三、 綜合、題組題
1. 是非題:對的打○,錯的打╳

2  3  5  2  5  3  5 
1( ).(  -  )÷  =  ÷  -  ÷ 
7  4  2  7  2  4  2 
2  2 
2( ). - 2  = - 2 + 
5  5 

2. 填進適當的數於空格中,使下圖行、列各算式都正確

1 3  3 
解答: 1. 1(○)2(╳) 2. -1 ¸  ´ (-  ) = 
2  4  2 

0 ´ (-  ) ¸ 1 = 0 

3 1  10 
´ (-4  ) ´ (-  ) = 1
14  5  9 
5 5 
0 ¸  ¸  = 0
18  6 

參、 挑戰題

一、 選擇題
1  5  1  1  5  6  6 
1. ( ) 1 +  =  ,( 1 +  ) × ( 1 +  ) =  ×  =  ,依這樣的計
4  4  4  5  4  5  4 
1  1  1  1 
算方法,則 ( 1 +  ) × (1 +  ) × (1 +  ) × … × ( 1 +  ) =?
4  5  6  100 
4  201  5  101 
(A)  (B)  (C)  (D) 
100  4  100  4 

解答: 1. D

二、 填充題

1  1  1  1 
1. ( 1 +  ) × ( 1 +  ) × (1 +  ) × … × ( 1 +  ) =______。
2  3  4  10 
1  1  1  1 
2. ( 1 +  ) ÷ ( 1 +  ) ÷ ( 1 +  ) ÷ … ÷ ( 1 +  ) =______。
2  3  4  10 
1 1  1  1 
3.  + + + =______。
2 ´ 3  3 ´ 4  4 ´ 5  5 ´ 6 
1  1 
4. 小英班上共有 36 人,全班同學中有  報名了跆拳社,有  報名
3  6 

了空手道社,有  既報名了跆拳社又報名了空手道社,那麼同

學中有______人沒報名這兩個社團。

3  1 
5. 某國山地占全國面積的  ,丘陵是山地的  ,盆地是丘陵的 2
10  3 

倍,高原是盆地的  ,其餘皆是平原。請問:(1) 該國平原占

全國面積的幾分之幾?(2) 如果該國平原的面積是 70000 平方

公里,那麼該國總面積是多少平方公里?

1 1 1 1 
6. 計算(1) (1 + ) ´ (1 + ) ´ (1 + ) ´ L ´ (1 + ) 
2 3 4 10

1 1 1 1 
(2) (1 - ) ´ (1 - ) ´ (1 - ) ´ L ´ (1 - ) 的值。
2 3 4 10
1 2 5 7 120 
7. 求 [( - ) ¸ - ] ´ (- ) 的值
4 3 7 24 21
1 1 1  1 1 1  1 1 1 
8. 觀察以下規律:  = - ,  = - ,  = - 試求下列
2´ 3 2 3 3´ 4 3 4 4´ 5 4 5
各式之值。

1 1 1 1 
(1)  + + + ××××× + = 。
2´ 3 3´ 4 4 ´ 5 49 ´ 50
2 2 2 2 
(2)  + + + ××××× + = 。
1´ 3 3 ´ 5 5 ´ 7 49 ´ 51

9. 試求下列各式之值:

2 2 2 2 
(1)  + + + ××××× + = 。
2´ 3 3´ 4 4 ´ 5 49 ´ 50
1 1 1 1 
(2)  + + + ××××× + = 。
1 ´ 4 4 ´ 7 7 ´ 10 49 ´ 52

11  9  1  7 
解答:1.  2.  3.  4.26 人 5.(1)  (2)2100000 平方公里
2  22  3  30 

11  1  12  50  24  17


6. = 、 = 7.5 8.  9.  , 
2 10 25 , 51  25  52 
主題四 一元一次方程式

4-1 以符號代表數

壹、 基本題

一、 選擇題

1. ( ) 下列何式與 4a 相等? 

(A) a × a × a× a  (B) a+a+a+a  (C) a+4  (D) a 4 

2. ( )下列表示法,何者正確? 
(A) x ÷ ( –5 ) = – 5x  (B) 241y = 241 + y 
1  1 
(C) z .  = 7z  (D) w ÷ ( – 8 ) = –  w 
7  8 

3. ( )下列表示法何者正確? 

1  x  1 
(A) x ÷  =  (B) x ×  = 5x 
5  5  5 
1  1  x 
(C) x÷( –  )=– 5x  (D) x ÷( –  )=- 
5  5  5 

4. ( )下列表示法哪一個是正確的? 

1  x  1 
(A) x ÷  =  (B) x × ( –  ) = 2x 
5  5  2 
2  2x  1 
(C) x ÷  =  (D) x ÷ ( – 8 ) = –  x
3  3  8 
5. ( ) 下列敘述何者正確? 

(A) ( – 4 ) x = x – 4          (B) 4 – 2x = 4 – ( – 2x ) 


(C) 7x ÷ 3 = ( 7 ÷ 3 )x  (D) – ( x – 5 ) = – x + 5 

6. ( ) 下列敘述哪一個是正確的? 

(A) 5a 2 = 5a × 5a  (B) 5a = a × a × a × a × a 


(C) 8a  = 2a × 2a × 2a  (D) a 4 = a + a + a+ a 


7. ( ) 下列哪一個數學式等於 7  x? 

4  4  4  4 
(A) 7 +  + x  (B) 7 ×  + x  (C) ( 7 +  )× x  (D) 7 ×  × x 
5  5  5  5 

8. ( ) -2x 等於下列哪一個式子? 

(A)-2+x  (B)  x-2  (C)(-x)+(-x)(D)(-x)+x 

9. ( )小民的爸爸今年 57 歲,小民比爸爸年輕 26 歲,則 t 年後,

小民的年齡為多少歲? 
(A) 31  (B) 31 + t  (C) 26 + t  (D) 26 

10. (      )小玉的體重比表妹多 8 公斤,若小玉的體重為 x 公斤,則

表妹的體重可以如何表示? 

(A) x + 8 公斤  (B) x – 8 公斤  (C) x × 8 公斤  (D) x ÷ 8 公斤

11. ( ) 有一個二位數,十位數字為 a,個位數字是十位數字的 3
倍還多 1,則下列哪一選項可以用來表示此二位數的個位數

字? 

(A) 3a – 1  (B) 10a + 1  (C) 13a + 1  (D) 3a + 1 

12. ( )把文字敘述“比 y 的一半小 6 的數”寫成一元一次式,應為 

1  1 
(A)  y – 6      (B) 2y – 6  (C)  y + 6    (D) 2y – 6 
2  2 

13. ( )設 x = 33,則 0.5 ( x – 13 )  的值為何? 


(A) 20  (B) 23  (C) –6.5  (D) 10 


14. ( )百貨公司舉辦促銷活動,將原價 x 元的衣服改為 (  x + 1 ) 

元出售。請問下列哪一個敘述可作為此百貨公司的促銷標語? 

(A)原價打五四折再加 1 元  (B)原價打八五折再加 1 元 

(C)原價打對折再加 1 元  (D)原價打八折再加 1 元


15. ( )若 x =  ,則下列哪一個算式的值最小? 


(A) 2x – 1  (B) – 6x  (C) – x + 1  (D)  x + 1 


16. ( )兩年前,阿明的年紀是校長的  ,如果三年後阿明的年紀

是 x 歲,那麼校長今年是多少歲?
x - 5
(A) 5x + 2  (B) 5 ( x – 5)  (C) 5 ( x – 5 ) + 2  (D)  + 2 

17. ( ) 有一對兄弟,五年前兄為 x 歲,弟為 y 歲,其五年後兩

人相差幾歲?(A)x-y-10 (B)x-y (C)x-y+5 (D)x-y+10 

18. ( ) 若 x=2,不是下列哪個方程式的解? 

(A) 3x-2=4 (B) 3x=6 (C) 4x+5=13 (D) 2x-5=1 

19. ( )泡芙每個售價 3x 元,元太買了 4 盒(6 個泡芙裝一盒),



另外又付了  買了一個購物袋,則元太共須付多少元? 

1  1  1  72 
(A) 72  x  (B) 12  x  (C) 24  x  (D)  x  。
6  6  6  6 

20. ( ) 兄比弟大  x  歲,則  10  年後兄比弟大多少歲? 

(A)  x +10  (B)  x +20  (C)  x  (D) 2 x 

21. ( ) 在『  7x – (–6x)  』一式中位置介於  7x  與  (–6x)  之間的

『–』怎麼讀?  (A)只能讀作『減』(B)只能讀作『負』(C)可以

讀作『減』,也可以讀作『負』。

22. ( ) 若 2x=-2x,則 x=  (A)不存在  (B) 2    (C) 0    (D)-2


23. ( ) 已知一個長方形的長為 p 公分、寬為 q 公分,則這個長

方形的面積=?(A)p×2+q×2  (B)p×q  (C) (p+q)×2  (D)p+q 

平方公分。

1  a 
24. ( ) 已知 a 為正整數,則 a 的倒數是  (A)  (B)  (C)a 
a  1 

(D)沒有倒數。

1  1  a 
25. ( ) 已知 a 為正整數,則  的倒數是  (A)  (B)  (C)1  (D) 
a  a  1 

沒有倒數。

26. ( ) 若阿馬哥從今天起每天存 100 元給慈善機構,經過 x 天

後,阿馬哥存款總數如何表示?

(A) 100+x (B) 100-x (C) 100 ¸ x (D) 100 ´ x

27. ( ) 若美雲從今天起每天存 10 元,經過 x 天後,美雲存款總

數為何?

(A) 10+x (B) 10-x (C) 10 ¸ x (D) 10 ´ x


28. ( ) 若公司分紅共 m 元,  給爸,其他給媽媽,則媽媽分得

多少元?
4 4
(A) 9 ´ m (B) 5+4+m (C)  ´m (D)  ´m
5  9 

29. ( ) 若 A、B 兩數之積為 18,且 A 數的值等於 y,則 B 數為


y 1
何? (A)18y (B)18 ¸ y (C) 18 ´ (D)  ¸ 18 
3  y 

30. ( ) 若 a、b 兩數之和為 48,且 a 數的值等於 x,則 b 數為何?

(A)x+48 (B)x-48 (C)48-x (D)48+x


31. ( ) 已知一台斤等於  公斤,則某物重 b 公斤,相當於幾台

3 3  3 3 
斤? (A) b ¸  (B)  ¸ b (C) b ´  (D)  ´ b
5  5  5  5 


32. ( ) 已知 1 公斤等於  台斤,則某物重 a 台斤,相當於幾公

5  5  5  5 
斤? (A)a+  (B)a-  (C)a ´  (D)a ¸  公斤
3  3  3  3 

33. ( ) 下列哪一個選項與-8x 的意義相同?

(A)x-8 (B)(-1) ´ x+x+x+x+x+x+x+x

(C)(-1) × x × x × x × x × x × x × x × x  (D)-x-x-x-x-x-x-x-x

34. ( ) 下列哪一個選項與 5a 的意義相同?

(A)a+5 (B)a ´ a ´ a ´ a ´ a (C)5 ´ 10+a (D)a+a+a+a+a


35. ( ) 代數式 5x-4x+7x 可以化簡成下列何式?

(A)16x (B)8x (C)x+7 (D)1+7x

36. ( ) 代數式 3x+8x 可以化簡成下列何式?

(A)11x (B)24x (C)11 (D)3+8x

2 x  2 
37. ( ) 代數式  ¸  與下列何式相等?
3  7 
6x  14x  6x  21 
(A)  (B)  (C)  (D) 
21  6  14  2 x 

7
38. ( ) 代數式 (-15x) ¸ 可以化簡成下列何式?

120x 120 7x 7
(A) - (B) - (C) - (D) -
7  7 x  120  120 x 

解答:

1.B 2.D 3.C 4.D 5.C 6.C 7.C 8.C 9.B 10.B

11.D 12.A 13.D 14.D 15.B 16.C 17.D 18.D 19.A 20.C

21.A 22.C 23.B 24.A 25.B 26.D 27.D 28.D 29.B 30.C

31.A 32.D 33.D 34.D 35.B 36.A 37.B 38.A


二、 填充題

1.  7 - y ¸ 2 可化簡為 。

8
2.  x ¸ æç - ö÷ + 9 可化簡為 。
è 3 ø

æ 1 ö
3. a ¸ ç - ÷ 可簡記為 。
è 8 ø

4. 8 - (- 6 ) ´ x 可化簡為 。

2 1 
5.  x ¸  + 可化簡為 。
3  3 

8 4 
6.  æç -  y ö÷ ¸ 可化簡為 。
è 3  ø 5 

7. 若 x = -2 ,則 x 2  + 7 = 。

1 1 
8. 若 y =  ,則 y ¸  - 5 = 。
3  2 

9. 某日學校營養午餐發放小蕃茄,七年 7 班班長宣布每人拿 4 個
剛剛好發完,若該班有 x 人,則袋中小蕃茄應有 顆。

10. 一大盒鱈魚燒 x 元,內裝 5 小包,則平均每一小包 元。

11. 小華期中評量 7 科總分 x 分,則平均每科 分。

12. 一包色紙內有 12 張大小相同且邊長為 x 公分的正方形色紙;若

將這些色紙依序平鋪在書桌上(均不重疊),則書桌被色紙覆蓋

的面積為 平方公分。

13. 校慶運動會同學們表演跳舞,要圍成邊長 x 個人的空心正方

形,則全部需要 位同學才能圍出。

14. 正三角形底邊長 a 公分,高 8 公分,則面積為 平方公分。

15. NONO 百貨公司週年慶全面依定價打八折優待顧客,則售價 x 

元的物品,定價是___元。

16. 一個蛋糕 x 元,切成同樣大小的 8 小塊,若小明買了 3 小塊,

則應付______元。
17. 一容器的容量是(2.5+x)公升,則它的容量是______立方公分。

18. 已知有兩個大、小不同的正方形,其周長相差 50 公分,假設小

正方形的周長是 3a 公分,則大正方形的邊長是______公分。

19. 一箱蘋果有 x 個,平分給一群學生,若每位學生分到 8 個,則

最後不足 14 個,則學生有______人。

20. 真好吃餐廳單點一客義大利麵要 x 元,若點義大利麵套餐需要

再加 100 元,則一客義大利麵套餐是 元。


21. 已知寶寶的身高是媽媽身高的  倍再加 10 公分,若媽媽的身高

是 x 公分,則寶寶的身高是 公分。


22. 一個正三角形的邊長為 2 x -  公分,則此正三角形的周長為

公分。

23. 2(2 x - 5 ) - 3 (- x - 1 ) 可化簡為 。

2  5 
24. 9 æç y - 1 ö÷ - 2 æç + y ö÷ 可化簡為 。
è 3  ø è 2  ø
25. 哥哥有 100 元,弟弟有 50 元,媽媽各給哥哥與弟弟 x 元,請問

哥哥與弟弟共有 元。

26. 解一元一次方程式 5 x + 40 = 9 x - 24 

27. 解一元一次方程式 3(x - 5 ) - 2 (2 x + 1 ) = 1 

x  3  5 
28. 解一元一次方程式  -  = x + 1 
3 4  6 

2 x - 1  x - 2 
29. 解一元一次方程式  =
3  4 

30. 便利商店做促銷,買一個飯糰及二瓶豆漿只要 55 元,總共比原

價便宜 10 元,若已知一個飯糰原價 25 元,則豆漿一瓶原價 

______元。

31. 安安到蛋糕店訂購一個母親節蛋糕,已知蛋糕減價 100 元後,

再打七五折的價格為 600 元,請問一個母親節蛋糕原價______ 

元。

32. 爺爺今年 100 歲,三姊妹的年齡加起來和爺爺一樣,又知大姊


比二姊大 3 歲,二姊比小妹大 2 歲,則大姊今年______歲。

33. 快樂幼稚園的老師發汽球給小朋友,若一人 2 個還剩 10 個,若

一人 4 個則不夠 18 個,則快樂幼稚園共有______位小朋友。

2 1 
34. 解一元一次方程式  x +  x  = 7 
3  2 

3 - x 
35. 解一元一次方程式 2 x -  = 1 

36. 安安全班週末到東勢林場露營,36 人共租了 8 頂帳蓬,同學協

議每頂帳篷只睡 4 人或 5 人,請問這 8 頂帳篷中只睡 4 個人的共

有______頂。

x  x  x 
37. 解一元一次方程式 x = 1 + - +
3  6  12 

13 x  13  3 
38. 下列式子:-  =-  x  =- 2  x  是否正確?
5  5  5 

27  6 
39. 下列式子的化簡:  x  +  =3x 是否正確?
11  11 

40. 等腰三角形兩個等長的邊各為 a 公分,另外一邊長為 3 公分,

則其周長為  ______公分。
41. 長方形兩個鄰邊的長各為 a 公分、b 公分,則其周長為

公分。

42. 若 a=­4,則 3a - 8 = 。

43. 若 a=­4,則 a 2 = 。

44. 若 a=4r,則 a 2 = 。

45. 長方形兩個鄰邊的長各為 a 公分、b 公分,則其面積為

平方公分。

46. 新新每天零用錢 a 元,存了一星期以後,若想買一個 hello

kitty 的鉛筆盒,則尚不足 15 元,那麼一個 hello kitty 的鉛

筆盒需 元。

47. 竹竹百貨舉辦年終特賣會,全面八折優惠,若一件牛仔褲定價

為 a 元,打八折後需 元。

48. 小華原有 180 元,如果每週再存 30 元,那麼 a 週後,他共有

元。

49. 有一個算式為 7x+5,若文字符號所代表的數為 7,則算式 7x

+5= 。
50. 將一包糖果分給 x 個小朋友,若每人分得 10 個,則不夠 6 個,

因此這包糖果共有 個。


51. 已知光光的體重為 x 公斤,華華的體重是光光的  ,佳佳的體
10 

重比光光少 3.5 公斤,則光光、華華、佳佳三人共重 公

斤。

52. 兄比弟多 2 歲,若兄今年 x 歲,則 7 年後,兄弟二人的年齡和

為 。

53. 中華大哥大 240 元月租型的費率,通話 20 分鐘以上者,每秒加

收 0.2 元,若智勇這個月帳單繳了 a 元,且 a>240,則智勇這

個月的通話秒數為 秒。

54. 原子筆一打  x  元,買 6 枝原子筆共需  ______  元。

55. 已知父親今年  37  歲,兒子今年  11  歲,x 年後父子年齡和為 

______  歲。
(請化簡)

56. 彬彬買了  5  碗花生豆花,每碗是  x  元,他拿了一張千元的鈔

票付錢,應找回  ______  元。(請化簡)

1
57. 蚵仔麵線一碗  (a - 3 )  元,貢丸湯一碗  ( a + 1 )  元,小明各買  8

碗需付  ______  元。
(請化簡)

58. 求下列各算式所代表之數: 

(1)  x = -5 , - 20x - 3 = ______ 

(2)  y  = -3 , -  y 2  + 7 = ______ 

1
(3)  a =  , - 8a + 6 = ______ 

(4)  x =  -1 , x 5  + x 4  = ______ 


59. 若  x =  時,比較  x、  x 2  所代表的兩個數大小  ________。
17 

(請以  x  表示)

60. 問題:
『如果一個正方形的邊長是  x  公分,那麼它的面積為____ 

平方公分。』

甲說:空格中要填入  x 2 。

乙說:空格後已經有『平方』兩個字了,因此空格中只要填 入 

x  就可以了。

丙說:因為題目的意思不太清楚,其實甲、乙都有可能是對的。

你認為甲、乙、丙三人誰說的對?答:________。

61. 長方形的長為 6 公分,若其面積為 a 平方公分,則其寬= 

________公分。

62. 一年三班共有 30 位同學,參加校外教學活動,每位同學交通費
(x+30)元,餐飲費 90 元,問全班共需________元 。

63. 設 x=3 為方程式 mx+5=17 的解,則 m=________。

64. 已知小明第一次段考 5 科的平均為 a 分,則小明第一次段考的

總分為 分。

65. 有 n 張相同的紙張疊在一起,其厚度為 a 公分,則每張紙的厚

度為 公分。

66. 一平行四邊形底為 a、高為 h,則它的面積為 平方單

位。

67. 若符號 S 表是一正方形的周長,則該正方形的邊長

為 ,面積為 。

68. 有一個長方體長為 a、寬為 b、高為 c,則該長方體的體積為

立方單位,表面積為 平方單位。

69. 若正方體邊長= a,則正方體的體積= 立方單位,表

面積= 平方單位。

70. 以符號 r、h 分別表示圓柱體的底面半徑、高,以符號 π 表示圓

周率,則圓柱的體積可以表示成 立方單位,表面積可

以表示成 平方單位。
71. 以符號 a、b、h 分別表示一個的梯形的上底 、下底、高,則該

梯形的面積可以表示成 平方單位。

72. 已知圓錐體體積=底面積  ×  高  ÷ 3,如果有一圓錐底面半徑為 

r、錐體高 h,則體積為 立方單位。(以 π、h、r 表示,

其中 π 為圓周率) 

73. 化簡下列各式:

1
(1)  x + x + x = (2)  x + x + x =

1  1  4 1  1 
(3)  (-x ) ´ ´ = (4)  ( x ) ´  ¸ =
3  2  3  5  2 

74. 化簡下列各式?

(1)  (-x ) ´10 = (2)  5 x ¸ ( - ) =

3  5  11  24 
(3)  (-  a ) ¸ = (4)  ( -  a ) ¸ ( - ) =
11  22  3  25 

解答:

y 3 3 1  10
1. 7 - 2. -  x + 9  3. - 8a  4.8+6x 5.  x +  6. - y 
2  8  2  3  3 
13 x  x 
7.11 8. - 9.7x 10.  11.  12. 12x 2 
3  5  7 
5  3x 
13. 4 x - 4  14. 4 a  15.  x 16.  17.2500+1000 x
4  8 
3a + 50  x + 14 1
18.  19.  20. x + 100 21.  x + 10  22. 6 x - 2 
4  8  3 

23. 7 x - 7  24. 4 y - 14  25. 150 + 2 x  26.16 27. -18


7 2
28. - 29. - 30.20 31.900 32.36 33.14 34.x=6
2  5 
7 4
35. x  =  36.4 37. x  =  38.是 39.否 40. 2a+3
9  3 

41.2(a+b) 42.-20 43.16 44. 16r 2  45.ab 46.7a+15

47.0.8a 48.180+30a 49.54 50.10x-6 51.2.9x-3.5

52.2x+12 53.5a 54.6x 55. 2x+48 56.1000-5x

10  1 
57. 12a-16 58.(1)97 (2)-2 (3)-  或-3  (4)0
3  3 

61.  a 
2
59. x > x 60. 甲 62. 30(x+120) 63. 4 64. 5a 

a  ah 或 1  S  S 2 
65.  66.  ah  67.  ;  68. abc;2ab+2bc+2ac 
n  2  2  4  16 
(a + b)h 
69.  a 3 ; 6a 2  70.  pr 2 h ; 2prh + 2 pr 2  71. 

pr 2 h  5  1 8 
72.  73.(1) 3 x  (2)  x  (3) - x  (4)  x 
3  2  6  15 

74. (1) - 10 x (2) - 20 x (3) - 6 a (4) 275 a 


3  5  72 

三、 綜合、題組題

1.  (1)  如果 x = – 3,則 3x – 5 =______。 

(2) 如果 x = 5,則 3x – 5 =______。

2. 把下列各文字敘述改寫成一元一次式: 

(1) x 的  8 倍可寫成______。
(2)  比 y 的一半大 13 可寫成______。 


(3) z 的  倍減 4 可寫成______。

3. 簡記下列各式: 

(1) x × ( – 5 ) =______。 

(2) y ÷ ( – 7 ) – 13 =______。 

(3) z.z.3 =______。 

(4) w +w + w + w =______。

4. 求下列各式: 

(1)  當 x = – 2 時,–3 x + 9 =______。 

(2)  當 y = 6 時,– 8y – 6 =______。 

(3)  當 x = 0 時,– 5x – 5 =______。

5. 試求下列各代數式的值: 

(1) x = – 8,– 3x ÷ 2 =______。 


(2) x =  ,– 3x + 2 =______。 

(3) x = 0.9,– 3x – 2 =______。
6. 請完成附表:

算式 x 5 0 

5x-11 (1) (2) -9
7x+2 37 (5) (6) 

x -7 (7) (8) (9)

7. 若父親的年紀為兒子年紀的 4 倍多 5 歲: 

(1)若兒子為 x 歲,則父親為 歲。 

(2)若父親為 y 歲,則兒子為 歲。

8. 請在下列的空格中,填入各算式所代表的數: 


3
8  -
算式  2 

-5-2x  (1)  (3) 


3
x + 1  (2)  (4) 

9. 化簡下列各式: 

(1) 5 ( –2x + 3 ) = ________。 

(2) –2 ( 3x –  ) = ________。 


(3)  當  x =  時,


求  5 ( –2x + 3 ) – 2 ( 3x –  )  所代表的值為  ________。

10. 在下列空格中,填入各算式所代表的數:

x
5 7 (1)
算式

3x + 2 17 23 26 

3
x - 1  (2) (3) 11

11. 請在下列空格中填入適當的數值: 


式子 ③  3  59 

6 + 14 x  20  ④  832 

16 x - 9  7  39  ⑥ 

30 x - 3  27  ⑤  1767 

12. 兄弟現年分別為 10、8 歲,若

(1)經過 x 年後,兄為 歲。

(2)在 y 年前,弟為 歲。

(3)經過 x+3 年後,兄弟兩人年紀和為 歲。


13. 兄弟兩人相差 5 歲,

(1)若兄為 x 歲,則弟為 歲。

(2)若弟為 y 歲,則兄為 歲。

(3)經過 m 年,兄弟兩人相差 歲。

14. 已知阿明跑步速度為每秒 8 公尺,則

(1)20 秒後,阿明跑了 公尺。

(2)跑了 8000 公尺,阿明用了 秒。

(3)a 秒後,阿明跑了 公尺(用 a 來表示)

(4)跑了 b 公尺,阿明用了 秒(用 b 來表示)

15. 黃金市場上,市價每一兩 2000 元,則

(1)買 5 兩,需要 元。

(2)花 8000 元,可買 兩。

(3)買 a 兩,需要 元。

(4)花 b 元,可買 兩。

16. 請在下列空格中,填入各代數式在不同的 x 值所代表的數:


x 5  _

代數式 

3
­  x  (1) (2)

5x (3) (4)

1  2 
答:1.(1)-14;(2)11 2.(1)8x;(2)  y+13;(3)  z-4
2  3 
1  2
3.(1)-5x;(2)-  y-13;(3)3z ;(4)4w


4.(1)15;(2)-54;(3)-5 5.(1) 12;(2)  ;(3)-4.7

24  3 13 
6.(1)14;(2)-11;(5) 2;(6)  ;(7) - ;(8)-7;(9)- 
5  4  2 
x - 5 1
7. (1)4x+5 (2)  8. (1)-21 (2)7 (3)-2 (4) ­ 
4  8 

9. (1) –10x + 15 (2)–6x + 1 (3)14

13  19 
10. ③1 ④48 ⑤87 ⑥935 11. (1)8 (2)  (3) 
2  2 
12. (1)10+x  (2)8­y  (3)2x+24  13. (1)x­5 (2)y+5 (3)5 

14. (1)160 (2)1000 (3)8a (4) 

b  15  1 2
15. (1)10000  (2)4  (3)2000a  (4)  16.(1) ­  (2) ­  (3)26(4) ­ 
2000  2  6  3 

貳、 進階題

一、 選擇題
1. ( ) 下列敘述何者正確? 

4  1 
(A)當 x = 2 時,  x = 4  (B)當 y =2 時,8  y = 8 
5  2 

(C)當 z = 1 時,4z – 8 = – 4  (D)當 s = 2.3 時,11 – 5s = 1.5 

2. ( ) 阿明進入職棒隊,第一年的年薪是 A 元,因第一年表現

良好,所以第二年加薪 10%,但第二年因傷表現欠佳,於第三

年又減薪 10%,試問第三年年薪為多少元? 


(A) A 元  (B) ( A +  A )元 
10 
1  1  1  1 
(C) ( A +  A ) ×  元  (D) ( 1 +  )( 1 –  ) A 元
10  10  10  10 

3. ( ) 新新從一樓爬到二樓共走了 x 個階梯,若每層樓的樓梯

數都相同,那麼新新在一樓與十樓之間做往返一次的走樓梯運

動,共要走幾個階梯?(A)9x (B) 10x (C) 18x (D)19x  。

4. ( )『哈利波特 6-混血王子的背叛』全球熱賣中,新新書


店大降價每本售價 x 元,而竹竹書店每本售價比新新書店多 

倍,請問:竹竹書店每本售價多少錢? 

(A) x  (B) 7 x  (C) x  +  1 (D)以上皆非。


6  6  6 


5. ( ) 若方程式  = 1 . 5 等號左邊的分母化為整數後,此方程
0 . 2 
x  10 x 
式可變為下列哪一選項?  (A)  = 15  (B)  = 1 . 5 
2 2 
10 x 
(C)  = 15  (D)以上皆非

6. ( ) 光全和林鳳贏的 鮮奶每公升都賣 x 元,若光全先調漲價

格 20%後再調降 20%,而林鳳贏則是維持不變,則哪一家的鮮

奶較便宜?(A)光全(B)林鳳贏(C)一樣便宜(D)無法比較。

7. ( ) 蔚伶的班上有 38 人,在第一次段考英文及格的有 x 人,

已知數學及格的人數是英文及格的  ,則數學不及格的有幾

人? 
2  2 
(A) 38-x (B)  x (C) 38-  x (D)條件不足,無法求得答案。
3  3 

3 x - 2 
8. ( ) 在算式  中,當  x  為下列何數時,其算式值最大? 

2  2 
(A)  (B) –  (C)  1      (D)–2 
3  3 

9. ( ) 一年二班有女生 x 人,而女生比男生少的人數佔全班人


數的  ,則一年二班有多少人? 

5  8  9  4 
(A)  x  (B)  x  (C)  x  (D)  x 
2  3  4  5 


10. ( ) 若男女分班中,小明班上男生有 16 人,佔全班的  ,

則女生有多少人? (A)10 (B)12 (C)14 (D)16

11. ( ) 面積為 36 平方公分的正方形,則其周長為幾公分?


36 1  36 36 1  36 
(A)  æç ö÷ ´ (B)  æç ö÷ ´ 4  (C)  æç ö÷ ¸ 4  (D) ¸ æç ö÷
è 6  ø 4  è 6  ø è 6  ø 4  è 6  ø

12. ( ) 周長為 t 公分的正八邊形,則其每一邊長為幾公分?

1
(A)t ¸ 8 (B)t-8 (C)8 ¸ t (D)t ¸

13. ( ) 若 x=30,則下列選項何者正確?

(A) - ( -( - x ) 2 ) = 900  (B) - ( -( - x )) 2 = 900 

(C) - ( -( - x) 2 ) = - x 2  (D) - ( -( - x )) 2  = x 2 

14. ( ) 若 a=5,則下列選項何者正確?

(A) ( -a ) 2 = -25  (B) - ( a 2 ) = 25 

(C) ( -a) 2  = -( a 2 )  (D) - ( -a ) 2  = -( a 2 ) 

15. ( ) 若 3x+7 的值為 4,則 x 可以為多少?

(A)-1 (B)-2 (C)-3 (D)-4


2 7 
16. ( ) 若 a= - ,則 25 ( a 2 -  ) 的值為多少?
5  5 

(A)-30 (B)-31 (C)-32 (D)-33

解答:

1.C 2.D 3.C 4.B 5.B 6.A 7.C 8.B 9.A 10.B

11.B 12.A 13.A 14.D 15.A 16.B

二、 填充題

1. 若將一大正方形紙張對折 2 次以後所得之小正方形邊長為 a 公

分,則原大正方形面積為 平方公分。

2. 小華今年 x 歲,兩年前父親年齡是兩年前小華年齡的 7 倍,則

小華父親現年 歲。

3. 小華的錢是小明的 2 倍少 15 元,若小華有 y 元,則小明有

元。
4. 已知多啦 A 夢的體重為 Kitty 體重的 3 倍多 4 公斤,如果多啦 

A 夢的體重是 x 公斤,那麼 Kitty 的體重是______公斤。

5. 有五個連續奇數,其中最小的一個為 x,則最大的為______。


6. 某班的女學生有 t 人,占全班的  ,則全班有______人。

7.  x ÷ y × w ÷ z-s × t 可記為______。

8. 五個連續奇數依照由小而大的順序排列,第三個為 x,那麼最

大的數是______。


9. 化簡下列各式: (1 ) 13 - 7 ´ x = _________ ( 2 ) x ¸ ( - ) - 2 = _________ 

10. 若 x=5 是 ax + 3 = 23 的解,則 a=______。


11. 華華國中管樂團有若干人,已知男生人數比全團人數的  少 5


人,女生人數為全團人數的  多 11 人,則華華國中管樂團的女

生有______人。


12. 小光原有若干元,他用所有款的  買壹台數位相機,再用剩餘


的  買壹台 CD,結果剩下 3600 元,則數位相機一台______元。

3 - 2 x 2 - 3 x  5 


13. 解一元一次方程式:  - =  。
3  2  3 

2 1 1 4 
14. 解下列一元一次方程式:  + 3  y - ( -1  ) + y = 2 。
3  5  3  5 

15. 解下列一元一次方程式: 42x - ( -5 x ) + ( -9 ) = 38 。

16. 連續三奇數中,若最大數為 x,則最小數為 。

17. 竹竹唱片行,周小倫「七月半的蕭邦」專輯每片定價 a 元,若

照定價打九折出售,仍賺 50 元,則每片專輯的成本是

元。

18. 明星張惠惠新出了一張照片海報,這張海報按照成本加兩成出

售,若這張海報要賣  y  元,則成本價是  ______  元。


(請化簡)

19. 若  x = 3 + y – y 2 ,  y = 4m + 1,當  m = –3  時,  y = ________, 

x = ________。

20. 5  元硬幣和  10  元硬幣共  x  個,其中  5  元有  9  個,則 10 元

硬幣共有  ________  元。(請以  x  化簡表示)


3 2  3 
21. 若 x = 5 時  x - a  = (a 是一個數)
,那麼當  x = 5  時,a –  x 
4  7  4 

的值等於________。

9 15  9 15 
22. 若  x -  - 0 . 6 = 0 ,則 2 ( x -  ) + 0 . 7 的值為________。
4  7  4  7 

23. 若-5 與 21 兩數各加一個相同的數之後,所得新的兩數互為相

反數,則所加的數為________。

24. 若符號 S 表是一圓形的周長,則該圓形的半徑為 ,面

積為 。(圓周率用 π 來表示) 

r  a° 
25.  如圖,扇形的周長為 ,面積

為 。

答:

y + 15 x - 4 8 
1.  4a 2  2.  7 x - 12  3.  4.  5. x+8 6.  t
2  3  3 
xw 4 
7.  - st  8. x + 4 9.  (1 ) 13 - 7 x ( 2 ) - x - 2  10. a=4 11. 23
yz  3 
12. 7200 13. 2  14. 0  15. 1  16. x­4  17. 0.9a­50 

5  5y  2 
18.  y 或  19. y = –11,x = –129  20. 10x – 90  21.– 
6  6  7 
S  S 2 
22. 1.9 23. -8 24. ;
2 p  4 p 
a  a 
25.  r + r + r ´ 2 ´ p ´ ; r ´ r ´ p ´
360  360 

三、 綜合、題組題

1. 設一個正方體的邊長為 t 公分,則: 

(1) 表面積是______平方公分。 

(2) 體積是______立方公分。 
(3) 周長是______公分。

2. 算出下列各式所代表的值: 

2  5  3 
(1) x = - ,  x +  =______。 
3 12  2 

(2) y = 0.2,24 ( 4 – 15y ) =______。

3.  5 年前媽媽的年齡是女兒的 6 倍,請回答下列各題: 

(1)  若 5 年前女兒的年齡是 x 歲,則今年媽媽的年齡是______ 

歲。 

(2)  若今年媽媽為 y 歲,則女兒今年的年齡是______歲。
4. 紅茶每罐  x - 3 元,奶茶每罐  2 x  元,如果  x = 28,請問  32  罐

奶茶和  36  罐紅茶共 元。

5. 試根據下列各題的題意,列出代數式: 


(1)  荔枝一斤價格是蘋果一斤價格的  倍,設荔枝一斤價格為 x 

元,則蘋果一斤的價格為______元。 

(2)  龍山社區合作社向果菜市場以每箱 2200 元訂購橘子 x 箱,

除貨款外,交貨時需再支付運費 300 元,請問交貨時共需支付 

______元。 

(3)  已知長方形的周長為 44,如果長方形的寬是 x,則長方形的

面積是______。

6. 下列各計算式是新新與竹竹對同一個分式的約分過程,請各位

同學幫他們檢查看看喔!
3 x  -  48  3 x  -  48 
新新→  竹竹→ 
54  54 
x  -  48 3 x  48 
=  步驟一 =  -  步驟一
18  54  54 
x  -  8 x  8 
=  步驟二 =  -  步驟二
3  18  9 
若新新與竹竹的約分過程皆無錯誤,請回答:無;若有,請說明是

誰的過程出錯,且是從哪一個步驟開始發生錯誤呢?______
2 3 11 
答:1.(1) 6t ;(2) t ;(3)12t 2. (1)  ;(2) 24

y + 25
3.(1) 6x + 5;(2)  4.2692 

5. (1) 4x;(2) 2200x + 300;(3) x ( 22 – x )  6.步驟一

參、 挑戰題

一、 選擇題

1. ( ) 兩人在操場比跑步,已知甲速度 3m/s,乙速度 5m/s,則

兩人同時同向出發 20 秒後,則兩人相距多少公尺?

(A)10 (B)20 (C)30 (D)40 公尺

2. ( ) 龜兔賽跑!已知兔子速度每小時比烏龜多跑 5 公尺,則

龜兔同時同地同方向出發 p 秒後,龜兔相距多少公尺?

(A)5-p (B)5+p (C)5 ´ p (D)5 ¸ p

答:1. D  2. C
主題四 一元一次方程式 

4­2 式子的化簡

壹、 基本題

一、 選擇題 


1.  (      )  下列哪一個選項化簡的結果為 3  x ? 

1  1  1  1 
(A) 3 ´  ´ x (B)  3 +  + x (C)  3 +  ´ x (D)  ( 3 +  ) ´ x
4  4  4  4 

2.  (      )  下列哪一個選項化簡後,與其他三者不同? 

3  5 
(A) a + a + 4a (B) 6 a 2  - a (C)(­  ) ´ ( -8 a ) (D) 15a ¸ 
4  2 

x x 
3.  (      )  化簡  + 所得的結果為下列何者? 
3  4 
2  x 2  7 
(A) 7 x  (B)  x  (C)  (D)  x 
7  7  12 

2 x + 1  3 x - 1 
4.  (      )以下是莉文化簡  - 的步驟,請問哪一個步驟開始
3  4 

發生錯誤? 

2 x + 1  3 x - 1  8 x + 4  9 x - 3 


- =  - …………第一步 
3  4  12  12 
8 x + 4 - 9 x - 3 
=  …………第二步 
12 
- x + 1
=  …………第三步 
12 
- x  1 
=  + …………第四步 
12 12 

(A)第一步(B)第二步(C)第三步(D)第四步
5.  (      )  下列各算式的化簡,哪些是正確的呢?

甲: 3 x - x = 3  乙: 5 x + 2 = 7 x  丙: 4 x ´ 2 = 8 x 

(A)甲、乙  (B)甲  (C)丙  (D)乙、丙 

6.  (      )  下列何者為一元一次式? 

(A) 4x + 3y  (B) 5x + 2y + 5    (C) 7x 2 + 6    (D)    2y 

7.  (      )下列各個式子中,正確的有哪些?

1  4(-2x-3)=-8x-12 

2(5x-3)-(9x-4)=4x-7 

3(-4x+5)-3(x+2)=-7x-1 

4(7x+ 5 )-(x-1)=6x+ 2 

3  3 

5(2x-5)-3(-x-1)=5x-2。 

1○
(A)  ○ 35  2○
○ (B)  ○ 45  1○
○ (C)  ○45  13
○ (D)  ○ ○。 

8.  (      )由於生產成本降低,市場競爭激烈,PONY 的數位相機五

月份將售價調降 10%,六月份再度調降 10%,如果現在每臺數

位相機售價為 x 元,則五月份時每臺的售價為多少元? 

x  x 
(A) 0.81x 元  (B) 1.21x 元  (C)  元  (D)  元。 
1 . 21  0 . 81 

17 1 
9.  (      )  x ´  -  和下列哪個選項不相等?
3  3 
16 1  1 
(A) x ´  (B) (17 x - 1 ) ¸ 3  (C) x ´ 17 ¸ 3 -  (D) 17 ¸ 3 ´ x - 
3  3  3 

10.  (      )5x  與下列那一個式子是相等的? 

(A)5 + x  (B)x + x + x + x + x 


(C)x.x.x.x.x  (D) x 5 

11.  (      )若  x  不為  0,下列哪一個選項計算錯了? 


(A) 3 x ´ 5 = 15 x  (B) 6 x ¸  = 4 x 

1 1
(C) 1 x ´ 1  = 2 x  (D)  6 ¸ 3 x = 2 x 
2  3 

12.  (      )今年小楓爸爸的年齡是  36  歲,恰為小楓年齡的  3  倍,

則  x  年前小楓的年齡(設  x < 12)可用下列哪一個多項式表

示? 

(A)  12 – x(B)  (36 - x )  (C)  36 – 3x(D)  12 – 3x 

3 x - 5  2 x - 7 
13.  (      )化簡  - 的過程如下:
2  3 
3 x - 5  2 x - 7 
- = 3 (3 x - 5 ) - 2 (2 x - 7 ) = 9 x - 15 - 4 x - 14 = 9 x - 4 x - 15 - 14 = 5 x - 1 
2  3 

請問由左而右數,第幾個等號開始出現錯誤? 
(A)1    (B)2    (C)3    (D)4 


14.  (      )  一套成本為 x 元的書籍,定價為 æç x +  x ö÷ 元,現在書商舉
è 5  ø

辦優惠活動,依照定價打七折為促銷價,請問書商以促銷價銷

售一套書的利潤為何?
1  1  7  7 
(A)賺  x 元  (B)賠  x 元  (C)賺  x 元  (D)賠  x 元 
50  50  25  25 

15.  (      )正方體的邊長為 a,則其表面積為 

(A) a 2  (B) 4a 2  (C) a 3  (D) 6a 2 

16.  (      )下列何者錯誤? 


(A) 6x÷3=2x  (B) 6x÷  =4x 


(C) 6x÷(-2)=3x  (D) 6x÷(-  )=-9x 

17.  (      )下列哪一個式子化簡後為一元一次式? 

(A)  3 x + 2 y + 1 (B) 4 x - 3 y + 5 z + 4 (C) 4 x + y - y + 3 (D)  x  + x + 4

18.  (      )下列哪一個式子化簡後為一元一次式? 
2  3 
(A)  3x + 2 y - 1 (B)  3 x + 2 x 2 - 1 (C)  3 x +  - 1 (D)  3x - 2  x - 1 
x  5 

19.  (      )下列那一個數學式子化簡後的答案是 - 3 x ? 

1  1  1  1 
(A)  - 3 ´ ´ x (B)  - ( 3 ´ ´ x ) (C)  - (3 + ) x (D)  - (3 - ) x 
3  3  3  3 


20.  (      )  - 12 x 與下列哪一個式子代表的數恆相等? 

7  7  7  7 
(A)  - 12 ´ ´ x (B)  - (12 - ) x (C)  - 12 - x (D)  - (12 + ) x 
9  9  9  9 

5  5  5 
21.  (      )化簡代數式 -  x - x - =
3  2  4 
- 50 x - 15  - 50 x  50 x - 15  - 15
(A)  (B)  - 15 (C)  (D)  + 50 x 
12  12  12  12 
3  7  5 
22.  (      )化簡代數式 -  x + x - =
4  5  2 
13 x  50  13 x - 50  13 x  50 
(A)  - 50 (B) 13 x -  (C)  (D)  + 
20  20  20  20  20 

23.  (      )a,b 均為任意實數,則以下各式何者正確? 

(A)  23 a = 23 + a  (B) - 32 b = (- 32 ) ´ b 

(C)  4 a = a ´ a ´ a ´ a  (D)  b 4  = b + b + b + b 

24. (      )若 x 不為 0,下列哪一個選項計算錯了?(A) 20 x ´ 5 = 100 x 



(B) 9 x ´  = x + x + x (C) 4 x = x + x + 2 x (D) 6 ¸ 3 x = 2 x 

25. (  )以下等式,何者有誤?

(A) 2 x -  y + 5 x + 2 y + 1 = 7 x + y + 1 

4 1 
(B) -  (3 - 2 x ) + (7 x + 2 ) = 3 x - 2 
5  5 

(C) - (3 x + 8 ) - 32 x = -35 x + 8 

(D) 3 y + 100 x - 43 y = -40 y + 100 x 

26. (      )若 甲 = 5 . 6 ´ ( 1000 - a ) , 乙 = 6 . 3 ´ ( 1000 - a ) ,則甲 + 乙 不可能為

以下何者?

(A) (1000 - a )´ (5 . 6 + 6 . 3 )  (B) (5 . 6 + 6 . 3 ) ´ 1000 - a

(C) 11 . 9 ´ (1000 - a )  (D)  1000 ´ 11 . 9 - 11 . 9 ´ a


27. (      ) a , b , c 分別為三個相異的分數,以下等式何者有誤?

(A) a ´ b ´ c = (c ´ b ) ´ a  (B) ab + b = (a + 1 ) ´ b 

(C) c ´ (a + b ) = (a ´ b ) + c  (D)  a ´ c = c ´ a 

答案

1.D 2.B 3.D 4.B 5.C 6.D 7.A 8.D 9.A 10.B

11.D 12.A 13.A 14.B 15.D 16.C 17.C 18.D 19.C 20.D

21.A 22.C 23.B 24.D 25.C 26.B 27.D

二、 填充題 
8  1 
1.  化簡 5 æç x + 2 ö÷ - 3 æç 2 - x ö÷ = 
è 5  ø è 3  ø

2.  △ABC 中,底為 x 公分,對應高是底的 3 倍多 1,則高= 


3.  化簡 6 x - 2 ìí- x - [4 x + 2 (x - 1 )]üý = 
î 2  þ

5 甲  5 x 
4.  若 - x = = ,則甲 ´ 乙 = 
12  12  乙 

x  x  x  x 
5.  化簡 x +  + + + = 
2  4  8  16 

1  99  98 
6.  化簡  ´  ´ ´ 0 ´ x =
99  100  97 
7.  化簡 2 x + 3 x - x =

8.  化簡 0 . 2 x + 3 x - 0 . 5 x =

9.  化簡 x + 2 y + 3 x - y =

10.  化簡 x  +  y  - x  =
2  3  3 

11.  化簡 ( - 8 x + 4 x ) ´ 1  =


12.  化簡 3 ( x + 1 ) - 2 ( 4 - x ) =

13.  化簡 6x ¸ 4 =

14.  化簡 3 x + 2 - 2 x - 1 =


5  7 

15.  化簡 2 x - 3 - 2 x + 1 - 1 =


3  4 

16.  化簡 ( 2 x - 7 ) - ( 5 - x ) =

17.  化簡 - 7 x - 1 (2 x - 6 ) =


5  5 

18.  化簡 1 ´ 1 ´ 1 ´ 0 ´ x =


2  3  5 

19.  化簡 3 y ¸ æç - 1 ö÷ =


5  è 3 ø
20.  化簡 - 4 x - ( - 4 x ) =
3  3 

21.  化簡 x  - é- x + 3 ( - 5 x - 4 ) ù =


2  êë 6  úû

22.  化簡 4x ¸ ( -7 ) =

23.  化簡 ( - 3 a ) ´ ( -4 ) =

24.  化簡 ( - 3 )( 8 x - 6 ) =


25.  化簡 ( - 2 x + 5 - 1 ) ´ ( -6 ) =


26.  化簡 12 - 2 ( 6 y + 9 ) =

a - 3 
27.  化簡 3 - 2 a - =

28.  平行四邊形兩鄰邊分別為 (12 n - 7 ) 公分、 (5 n + 7 ) 公分,則平行四

邊形的周長為 公分。(以 n 表示) 


29.  一元一次式  1 –  的一次項為 :

一次項係數為: 常數項為: 

30.  a + a + a = 

31.  3x+4x+5x  =
32.  – 9x+( –3x ) = 

33.  – 5x – ( –7x ) = 

34.  4x – ( 3x ) = 

35.  14x – 8 x = 

36.  3 ( 8y ) = 

37.  3(-6x)= 

38.  (-3y)× 4= 

39.  (-5)×(-8x)= 

40.  – (2 x –7 ) = 


41.  36y ÷ ( –  ) = 

42.  2 ( 3s + 5t ) = 

43.  – 9 ( 2x + 7y ) – 5( –6x + 4y ) = 


44.  x ÷  = 

45.  ( –107x + 9 ) ÷ ( – 5 )  = 

46.  ( 2x + 1 ) + (-x+3 ) =

(1 )( -  x ) ´ ( -0 . 4 ) = _________ 

47.  化簡下列各式: 
21 
( 2 )( -7 x ) ¸ = _________ 

(1 ) - 4 - 7 x - 2 x + 3 = ________ 


48.  化簡下列各式:  2  8  4  3 
( 2 )( - x ) ¸ + ¸ ( - ) = ________ 
3  9  7  28 


(1 )  ( - 26 x - 91 ) = ________ 
13 
49.  化簡下列各式: 
2 4 
( 2 ) - 84 (  x - ) = ________ 
7  21 

50.  請在 內填上正確的數字:­(x-5)=( )×(x-5)。 

3 x  - 4  3x  4 
51.  ­  =­  □  。請在□內填上+或- 
5  5  5 

52.  五年前新新 a 歲、竹竹 a+3 歲,則五年後新新與竹竹的年齡相

差 歲。 
1  1 
53.  化簡  (4+x)+  (6x-3)= 
2  3 

1
54.  蚵仔麵線一碗  (a - 3 ) 元,貢丸湯一碗  ( a + 1 ) 元,小明各買 8  碗

需付 元。
(請化簡) 

55.  設原子筆每打  x  元,鉛筆每打  y  元,則買  4  枝原子筆、  6  枝


鉛筆共需_______  元,若用一張 500 元大鈔付帳,應找回 

________  元。 

56.  化簡下列各式:(1) - 3( 2 x - 4 ) + 4 ( 4 - 5 x ) - 7 = ________ 

1 4 
(2)  (3 x - 2 ) - 5 (  x - 1 ) - 1  = ________ 
4  15 

57.  化簡下列各式:(1) 7 x + 2 + 3 x + 15 - 7  = 。 


3  1 
(2) x - 4 + 2 æç x + 3 ö÷ = 。 
2  è 4  ø

1  1 
58.  化簡:(–  ) ( 2x – 4 ) ÷ (  ) = ________。 
2  3 

59.  職棒卡一包有  4  張,需要  x  元,則買  32  張需要多少元?答: 

________。(請以  x  表示) 

60.  如果  5000 = 18x + 518,請在以下空格中填入適當的數值: 

(1)36x =  。(2)9x +  = 2500。 

61.  父親年齡 39 歲,兒子  x  歲,  5  年之後此兩人年齡和是

歲。

答案: 

31 
1.  9 x + 4  2.  3 x + 1  3.  14 x - 2  4.  60 x  5.  x  6. 0  7.  4 x  8.  2 . 7 x 
16 
x y  11x + 19 
9.  4 x +  y  10.  +  11.  - x 12.  5 x - 5  13.  1 . 5 x  14. 
6  3  35 
2 x - 27  9 6  9
15.  16.  3 x - 12  17.  -  x + 18. 0  19.  -  y  20. 0 
12  5  5  5 
4x 9 
21.  4 x + 12  22.  -  23.  12 a  24.  - 6 x + 25.  4 x - 4  26.  - 12 y - 6 
7  2 
9 - 5 a x  1 
27.  28.  34 n  29. –  ;-  ;1  30. 3a  31. 12x  32. ­12x  33. 2x 
2  3  3 
34. x  35. 6x  36. 24y  37. ­18x  38. ­12y  39. 40x  40.  -2x+7 
5  107 9 
41.  -81y  42. 6s+10t  43. 12x-83y  44.  x  45.  x+  46. x+4 
4  5  5 
(1 ) - 10 x - 35 
47.  (1 ) 3 x ( 2 ) - 5 x  48.  (1 ) - 9 x - 1 ( 2 ) - 3 x - 16  49.  50.  ­1 
5  3  4  3  ( 2 ) - 24 x + 16 
- 3 x  4  5 x + 2 
51.  + 52. 3  53.  54. 12a – 16  55. (1)4x + 6y(2)500 – 4x – 6y 
5  5  2 
7  1 
56. (1) –26x + 21(2)–  x + 3  57. (1) 10x + 10(2)2x + 2  58. –3x + 6 
12  2 
59. 8x  60. (1)8964(2)259  61. x + 49 

三、 綜合、題組題 

1.  化簡下列各式: 

(1) ( - 2 y ) × ( - ) = _______ 


(2) ( - 3 x ) ¸ = _______ 

3  2 
(3) a -  a = _______ 
2  3 

(4) - 7 x - ( -2 x ) = _______ 

(5) 3 ( 8 x - 2 ) + [ 2 - ( 3 x + 1 )] = _______ 

x  5  3 2 
(6) 6 (  -  ) - 15 (  + x )  = _______
3  2  5  3 
2.  是非題:(下列各題計算結果是否正確,請填「○」或「×」) 

3  2 
_______ (1)  a ¸ ( - ) = a 
2  3 

_______ (2)  3 y +  y = 3 y 2 


_______ (3)  y ¸ 20 ¸ 5 =

3.  一件衣服將成本加二成做為定價,照定價再打七五折出售,若

定價為  x  元,則: 

(A)  成本是多少元?答:________ 

(B)  售價是多少元?答:________ 

(C)出售後,賺或賠多少元?答:________ 

4.  化簡下列各式: 
(A)  –3x + 2x = ________ 

(B)  y – y = ________ 

10 
(C)  5x ¸ (–  ) = ________ 

(D)  3x + [ 2x – ( 4x – 6x ) ] = ________ 
1  1  1  3 
(E)  x –  –  x –  = ________ 
4  3  2  4 

5.  是非題: 

_______ (1)可以將  2 ( x + 5 )  寫成  2x + 5。 

_______ (2)可以將  ( 3 + 4x )( –5 )  寫成  ( 3 + 4x ) – 5。


x  - 1 - x  - 1
_______ (3)可以將  –  寫成  。 
2  2 

_______ (4)有一正方體的邊長為  x,則其表面積為  x 2 。 

6.  化簡下列各式: 


(1) y ´ 4 =


(2) 5x ¸  =

(3)  1.5b + 8.5b = 
(4)  7x – 8 + 5x + 2 = 

7.  化簡下列各題: 
(1)  7y – 4y = 
(2)  ( 4x – 7 ) + ( 5x + 6 ) = 
(3)  3 ( y – 6 ) – ( y + 1 ) = 
3  5 + 3 x 
(4)  x -  = 
2  2 

8.  化簡以下各式: 


(1)  3 x + 7 y - x = ______ 


(2) 10a + 2 b - ´ (10 a + 15 b ) = ______ 

(3)  - (3 x - 2 y ) - 7 x = ________ 

(4)  - (3 a - 2 b ) - 7 a + 2 b = ________ 

9.  化簡下列各式: 

(1) 5 x - 2 x + [2 ´ (6 x - 4 ) - 1 ] = ________ 


(2) 4 ´ (2 y - 1 ) - (1 + 2 y ) = _______ 
1
(3) æç x - 3 y ö÷ ´ 2 - (4 x - y ) = __________ 
è 2  ø
1 5  1 
(4) (x - y ) ´ + ¸ = _______ 
3  3  y 
2  æ 6 x  3 ö
(5) 2 x -  ´ ç ¸ ÷ = _______ 
3 x  çè 4  y ÷ø

10. 化簡下列各式:

4 x - 5 y 
(1)  - 32 x 

1  5 y  3  ù
(2) 3 ´ éê (3 x - 2 y ) - ´ (- 2 ) - x ú
ë 6  12  2  û

答案: 

y  7  5 
1.(1)  (2)–  x(3)  a(4)–5x(5)21x – 7(6)–8x – 24  2. (1) ×(2) ×(3) × 
3  2  6 
5  3  1  3  1  1 
3. (A)  x(B)  x(C) –  x  4. (A)–x(B)–  y(C)–3x(D)7(E)–  x – 3 
6  4  12  4  4  4 
28  35 
5. (1)×(2)×(3)×(4)×  6. (1)  y(2)  x(3)10b(4)12x – 6 
3  4 

7. (1)3y(2)9x – 1(3)2y – 19(4)– 

12
8.  (1)  x + 7 y (2) 8 a - b (3) - 10 x + 2 y (4) - 10a + 4 b 

1 4  y 
9.(1) 15 x - 9  (2) 6 y - 5  (3) - 3 x - 5 y  (4)  x + y  (5) 2 x -
3  3  3 
5  3 
10. (1) - 30 x - y  (2)  - 3 x + y 
2  2 
貳、 進階題

一、 選擇題 

1.  (      )若 a、b、c 為三個相異的正整數,則下列選項何者「不正

確」? 

a  b 
(A) ( - a + b ) ¸ c = - +
c  c 

(B) ( - a) ¸ ( b - c ) = -a ´
- b + c 

(C) - a ¸ ( b - c ) = a ´
c - b 

(D) ( - a - b ) ¸ c = -a ¸ c - b ¸ c 

2.  (      )將四張 100 元的鈔票和 x 個 10 元的硬幣平均分給 5 個人,

則每個人分得多少元? 

1 x + 4 4
(A)  x + 400  (B)  (C)  x + 2  (D) 2 x + 80 
5  5  5 

3.  (      )已知媽媽年齡恰好是兒子的  2  倍多  16  歲,且兩人的年

齡和為  52  歲,請問媽媽現在幾歲? 


(A)39  (B)40  (C)41  (D)42 

3 x - 5  2 + x 
4.  (      )化簡  - 為下列何式? 
6  5 
9 x - 37  21x - 37 
(A)  9x – 37(B)  30  (C)  21x – 37(D)  30 

5.  (      )下列選項何者正確?
1  1  x x 
(A) 2 x - x 可化簡為 2    (B)  + 可化簡為 3 x + 2 x = 5 x 
2  2  2  3 
æ 1 ö
(C) (5 x ) ¸ (7 x ) 可化簡為 5 x  (D) 3
3x × ç - ÷ 可化簡為 -  x 
7  è 2 ø 2 

6.  (      )化簡代數式 12 [- 4 x - 3 (x + 1 )] =

(A)  - 84 x - 36 (B)  - 21x - 9 (C)  - 120 x (D)  - 42 x - 18 


7.  (      )化簡代數式 - 16 ( - - 5 x )  =

(A)  14 - 80 x (B)  14 + 80 x (C)  - 14 - 80 x (D)  - 14 + 80 x 

x  1  1  1  1  1 
8.  (      )化簡 +  x - x = (A)  x (B)  x (C)  x (D)  x 
3  4  12  4  3  2 

3  1  3  7 x 
9.  (      )化簡 -  x - x + x - =
5  25  50  10 
32x 32 64 33
(A)  -  (B)  -  x (C)  -  x (D)  - x 
50  25  25  25 

答案:1. B  2. D  3. B  4. B  5. D  6. A  7. B  8. C  9. B 

二、 填充題 

2 x - 1  3 - x 


1.  化簡  - = ________ 
3  2 

x - 1
2.  化簡  - 2 x + 1 = ________ 

3.  將一張長為 10 公分,寬為( 5 x - 1 )公分的長方形紙片四個角都減

去邊長為 3 公分的小正方形,然後折成一個長方體,問長方體

容積為_______。 
10 


5x­1
4.  △ABC 為一等腰三角形,已知∠A= x o ,則∠B 有可能為_______ 

度。 

5.  五個連續偶數由小到大排列,若第二個數為 x ,則此五個數的

和為_______  。(以 x 表示) 

4 13 27 
6.  化簡  y - 4 +  y - = 
3  2  4 


7.  化簡–[(    2a + 8 ) ×  ]+9a  = 

8.  如圖將相鄰兩數式子相加的結果填入下方的空格中,則圖(3)代

表的式子為  (以 x 表示) 
(例)  x - 1 2  - 5 4 x - 1  2 x + 1 

x + 1
(1)  (2) 

(3) 

9.  將長方形 ABCD 摺出四小塊長方形,摺痕 EF 、 GH ,相交於一

點 I  ,再摺出△ IBD (如圖),則△ IBD 的面積為  (以 

x 表示)  A  8  G  2x  D 


E  I  F 

B  H C 
10.  老師生日,買了一些糖果分給全班 a  位同學,若每人分 3 顆,

還剩下 7 顆,如果糖果每顆 2 元,老師花多少錢買糖果? 

11.  4 個連續奇數,若最小的奇數為 y,請從小到大寫出此 4 個奇

數: 、 、 、 ;此 4 個數的和

為 ,平均為 

12.  一籃梨子平均分給 y 位同學,若每人 7 個,則不夠 3 個,則原

來這一籃梨子的一半有多少個 。 

13.  合作社一本作業本 a 元,一支原子筆比作業本貴 5 元,小櫻買

了 5 本作業本和 4 枝原子筆,共花了 元。 

3 5  1  2 
14.  化簡 10 (  x -  ) - 6 ( - x - ) =
5  2  2  3 

5
15.  化簡 ( - 4 x - 10 )  - (  - 8 x ) =  。 

3 - 2 x 2 - 3 x 


16.  化簡  - = 。 
3  2 

2 1  1 4 
17.  化簡  + 3  y - ( -1  ) + y = 。
3  5  3  5 

18.  這學期新新國中音樂班舉辦音樂會,其中有  的學生參加管樂

3  5 
表演,有  的學生參加弦樂表演,有  的學生前兩項活動都有
4  12 

參加,試問:若全音樂班有 x 個學生,則在這場音樂會中「參

加管樂表演卻沒有參加弦樂表演」的學生有 人。 

19.  已知長方形周長為  x  公分,且長比寬的兩倍多  2  公分,則寬

為  ______  公分。(請化簡) 

20.  四個連續奇數中,設最大的數是  y,則最小的數是  ________  ,

且此四個數的平均是  ________  。(請化簡) 

1  5 
21.  化簡:  ( 3x + 6 ) –  ( 2x + 3 ) = ________。 
9  6 

22.  當 x = 75 時,則  ( 237x + 450 ) – ( 236x + 449 )  的值為 

________。 

23.  化簡:  –3 ( 3x – 1 ) – (–4) ( 5x – 2 ) =  。

答案: 

7 x - 11  - 3 x + 1  180 - x


1.  2.  3.  60 x - 84  4. x o 或(  )度  5. 5 x + 10 
6  2  2 
47 43  15 
6.  y -  7.  a-6  8.  10 x - 6  9.  - x 2 + 24  10. 6a+14 
6 4  2 
7 y - 3 
11. y、y+2、y+4、y+6;4y+12;y+3  12.  13. 9a+20

25  5  1 
14.  9 x - 21  15.  4 x  -  16.  x  17.  4 y + 2  18.  x 
2  6  4 
x  2  x - 4 4  11 
19.  –  或  20. y – 6;y – 3  21. –  x –  22. 76 
6  3  6  3  6 
23. –29x – 5 
三、 綜合、題組題 

1.  一條繩子折成等長的 3 段後,比柯南的身高多出 2 公分: 

(1)假設柯南的身高為 x 公分,則繩子的長度為 公分。 

(以 x 表示) 

(2)假設繩長共 y 公分,則柯南的身高為 公分。(以 y 表示) 

2.  一個三位數字,其個位數字是百位數字的 2 倍少 1,十位數字

是個位數字的 2 倍多 1,假設百位數字為 x ,則: 

(1)  個位數字可以 x 表示為 。 

(2)  十位數字可以 x 表示為 。 

(3)  這個三位數字可以 x 表示為 。 

3.  如下圖, 
x+3

(1)長方形的周長為 。(以 x 表示) 

(2)  長方形的面積為 。(以 x 表示) 


4.  設一個 MP3  定價  x  元,則 

(1)打八五折的特價為 元 (以 x 表示) 

(2)手機定價比 MP3 貴 3 倍,手機定價為 元(以 x 表示) 

5.  火影忍者漫畫每本 x 元, 

(1)要買全集  45 本共需 元。 

(2)若大邦每月存 9 x 元,需 個月才能買齊全集 45 本。 

6.  哥哥的身高是妹妹身高的 2 倍多 5 公分 

(1)若妹妹身高為 x  公分,則哥哥身高為 公分。 

(2)若哥哥身高為 y 公分,則妹妹身高為 公分。 

7.  竹竹國中本學期七年忠班第三次定期考數學成績結果如下: 

(1)若七年忠班學生共 a 人,其中男生 20 人,則女生有 人。 

(2)承上題,若男生數學成績平均 65 分、女生數學成績平均 61 

分,則全班數學成績總分是 分。 

(3)承上題,全班數學成績平均是 分。 

8.  將下列各式的乘積展開: 

(1) - 5 ( -3 x - 20 ) =


5  5 
(2) - 12 (  x - ) =
11  121 

(3) - 7 ( -4 x + 3 y - 5 ) =

9.  將下列各式的乘積展開: 

(1) - 3 ( 5 x - 4 )  =


(2) - 33 (  x - 4 )  =
11 

(3) - 5 ( -4 - 4 x ) =

(4) - 3 ( 5 x + 4 y - 4 ) =

10.  請化簡以下各代數式: 

(1) 2 x - ( -4 x - 3 ) =

(2) - 3 [- 3 x - 3 ( -7 x - 2 ) ] =

(3) - 3 x - [- 2 ( -2 x - 2 ) ] =

11.  請化簡以下各代數式: 

(1) - 3 x - {5 x - 2 [- 3 x + 2 (- 5 x + 2 )]} =

(2) x + {- 2 x - 5 [x - 2 (11 x - 2 )]} =


ì1  2  ü
(3) - 3 x - í x - 2 éê- x + 2 (- x + 2 )ùú ý =
î 3  ë 3  ûþ

12.  請化簡以下各代數式: 

(1) 5 ( x + 3 ) - 6 ( x - 7 ) =


(2) 3 ( 2 x - 2 ) + 6 ( 2 x - 2 ) =

1  3 4  5 
(3) ( x + 2 ) - ( x - ) =
3  5  5  3 

答案: 


1. (1) 3 x + 6  (2)  - 2  2. (1) 2 x - 1  (2) 4 x - 1  (3) 142 x - 11 
3
y - 5 
3. (1)2x+18(2)6x+18  4. 0.85x、3x  5. 45x;5  6. 2x+5; 

61 a  +  80 
7. (1)a-20(2)61a+80 (3)  8. (1)  15 x + 100 (2)  - 55 x + 5 

(3)  8 x - 21 y + 35  9. (1)  - 15 x + 12 (2)  - 9 x + 132 (3)  20 x + 20 

(4)  - 15 x - 12 y + 12  10. (1)  6 x + 3 (2)  - 54 x - 18 (3)  - 7 x - 4 

11. (1)  - 34x + 8 (2)  104x - 20 (3)  - 26 x + 8  12. (1)  - x + 57



3
(2)  18 x - 18 (3)  -  x + 2 

參、 挑戰題

一、 選擇題 
7 x - 3  10 x + 3 
1.  (      )  用分式通分方法化簡 - =
5  7 
x - 36 x + 36 - x - 36 - x + 36
(A)  (B)  (C)  (D) 
35  35  35  35 

- 3 x + 11  - 5 - 4 x 


2.  (      )化簡 - =
4  3 
7 x - 53  7 x + 53 
(A)  x + 16 (B)  (C)  (D)  x - 16
12  12 

答案:1.C  2. C 

二、 填充題 


1.  化簡  ( 3a – 18 ) –[36 + 4 ( a – 9 ) – 4a]= 

2.  鳴人早餐吃了飯糰和豆漿,熱量為 b 大卡。午餐吃的便當熱量

是早餐的 2 倍少 10 大卡。如果一碗拉麵的熱量是早餐的 2 倍多 

20 大卡,而鳴人晚餐吃了兩碗拉麵,則鳴人三餐共攝取了多少

大卡? 

3.  一打雞蛋 4a 元,則一粒雞蛋 元,一箱雞蛋有 12 打,

買 2 箱共花 元。 

4.  一粒水煎包 y 元,買 10 粒送一粒,則買 20 粒共 元,

平均每粒 元。
5.  光明高中二年級共有學生 4x + 26  人,一年級比二年級多 x-15 

人,三年級比二年級少了 2x+7 人,則光明高中三個年級共有

學生 人。 

6.  三角形一內角為 x 度,,另一內角是其 2 倍多 10 度,則第三個

內角的度數為 。

答案: 

4  a  19
1.  a-8  2. 7b+30  3.  ;96a  4. 19y;  y  5. 11x+56 
3  3  20 

6. 170-3x 

三、 綜合、題組題 

1.  設一個 MP3  定價  x  元, 

(1)若 MP3 定價為成本加 3 成,則 MP3 成本為 元(以 

x 表示) 

(2)MP3 打八五折後,賺或賠 元 (以 x 表示) 

2.  某個二位數,其十位數字與個位數字和為 x, 

(1)若個位數字為 3,則十位數字為 。 

(2)此二位數用 x 表示為 。
(3)若個位數字與十位數字對調,此新的數用 x 表示為 。 

(4)新數和舊數的和為 。 

(5)新數和舊數的差為 。 

3.  小寶去內灣玩,從火車站到內灣相距  d  公里,坐公車去時平均

時速 40 公里,回程時公車時速 50 公里,則來回交通共花

小時。 

4.  阿福口袋中有 50 元硬幣與 10 元硬幣共 30 個, 

(1)若 50 元硬幣有 x 個,則 10 元硬幣有多少個 。 

(2)阿福身上共有 元。 

(3)若買 CD 花了 250 元,則阿福身上還剩 元。 

5.  大雄目前存了 1680 元,而且每週繼續存 15 元,但是每週花 x 

元買 6 個銅鑼燒和小叮噹一起吃,請問 

(1)每個銅鑼燒的單價是 。 

(2)八週後大雄還剩 元。 

6.  化簡下列各式  : 

(1) 5 ( x - 1 ) - 3 ( 2 x + 1 ) =

2 x - 1  2 ( 2 x + 1 ) 


(2) - =
7  3 
5 x  7 x  - 13 x 
(3) x +  + - =
3  15  30 

7.  化簡下列各式  : 

(1) - ( -4 x + 11 ) + 4 ( -3 - 22 x ) =

- 20 x - 1  4 ( - x + 12 ) 


(2) + =
3  5 
2 x  - 3 x  - 4 x  - 5 x 
(3) x +  + - - =
3  4  6  12 
2 x  - 3 x  2 x 
(4) - (  - ) =
15  - 5  - 3 

答案: 

10 21 
1.  x  ;賺  x  2.(1) x-3  ;  (2)10x-30+3  ;(3)x+27; 
13  260 
d  d 
(4)11x;(5)54-9x  3.  +  4. (1)30-x(2)4x+300(3)40x+50 
40  50 
x  - 22 x - 17  107 x 
5. (1)  (2)1800-8x  6. (1) - x - 8 (2)  (3) 
6  21  30 
- 112 x + 139  - 17x
7.(1) - 84 x - 23 (2)  (3) 2 x (4) 
15  15 
主題四 一元一次方程式 

4­3 解一元一次方程式

壹、 基本題

一、 選擇題 

1.  (      )  小玉把裝滿一塊錢硬幣的猪公撲滿打開,發現若拿走 39 

個 1 元硬幣後,全部兌換成五元硬幣的個數,是加入 11 個 1 元


硬幣後,全部兌換成 10 元硬幣個數的  倍,若假設小玉的猪公

x  x  3 
撲滿有 x 元,下列哪一個列式是對的?
(A)  -39=(  +11)× 
5  10  2 
3  ( x - 39 )  ( x + 11 )  3 
(B) 
5(x-39)=[10(x+11)] 
×  (C)  =  ×  (D) 
2  5  10  2 

5x-39=(10x+11)×  。 

2.  (      ) 3x +5 = 20 可根據何種等量公理得 3x = 20 – 5?(A) 等量

加法公理  (B) 等量減法公理(C) 等量乘法公理  (D) 等量除法公

理 
5  1  1 
3.  (      ) x =  為下列哪一個方程式的解?(A) 5x =  (B) – 5x = 
2  2  2 
x  1  2 
(C)  =  (D) –  x = 1 
5  2  5 

4.  (      )明星專櫃店裡,T 恤每件售價 x 元,蓉萱在其消費總額中

有 80 %刷卡,付現 400 元,蓉萱共買了 5 件 T shirt,則下列式

子何者正確?(A) 5x×80 %=400(B) 5x÷80 %=400(C) 400÷( 1 – 
80 %)=5x(D) 400×( 1 – 80 % )=5x
5.  (      )鈴茹與朋友兩個家庭到游泳池游泳,買了 6 張全票及 4 張

半票,共付了 840 元,已知全票比半票每張貴 40 元,設全票票

價每張為 x 元,則下列何式與題意符合?(A) 6 x + 4 ( x + 40 ) = 

840(B) 6 x + 4 ( x –40 ) = 840(C) 4 x + 6 ( x + 40 ) = 840(D) 4 x + 
6 ( x –40 ) = 840 
6.  (  )  下列何者是方程式 34–x ÷ 4 = 6 的解?(A) x = 6 × 4 + 34 

(B) x = ( 34 – 6 ) × 4  (C) x = ( 34 ­ 6 ) ÷ 4  (D) x = 6 × ( 34 – 4 ) 


7.  (      )  在下列的運算式中,何者正確?(A)-5x=7,則 x=7+5(B) 
5
-5x=7,則 x=7÷(­5)(C)-5x=7,則 x=7×5(D)-5x=7,則 x= ­ 


8.  (      )松英原有 x 元,買一本參考書花了所有錢的  ,結果還剩


下 360 元,依照題意,可以如何列等式?(A)  x = 360  (B) x = 

3  3 
360  (C) x × ( 1-  ) = 360  (D) x-  = 360 
5  5 

9.  (      )  下列哪一個方程式的解與 0.6 x+5=0.3 x-0.1 的解相

同?(A)6 x+5=3 x-1(B)6 x+50=3 x-0.1(C)6 x+50=3 x- 

1(D)6 x+5=3 x-0.1 

10.  (      )  客運公司因物價上升調漲售票,新竹到台北的票價漲了 

10%,若原票價為 x 元,漲價後為 110 元,則可列式為: 

(A) x‧( 1 + 10%  ) = 110 

(B) x‧( 1 – 10%  ) = 110 

(C) 110‧( 1 + 10%) = x 

(D) 110‧( 1 – 10%  ) = x
5 1 
11.  (      )  解方程式 ­ x + 4 = x ­ 6 的過程中,下列哪一個步驟先發
3  4 

生錯誤? 

1  5 
步驟一: 
4= x + x ­ 6 
4  3 
1  5 
步驟二:4 + 6  = x + x 
4  3 
23 
步驟三:10 = x 
12 
230 
步驟四:x =
12 

(A)步驟一  (B)步驟二  (C)步驟三  (D)步驟四 

12.  (      )志箐、美芳兩人共有 480 元,若志箐給美芳 60 元後,美

芳比志箐多了 40 元,則原來志箐有多少錢?(A) 320  (B) 280 

(C) 260  (D) 240 元 

『某數加 7 後,再除 5,再
13.  (      )小超超的老師出了一道數學題:

加 4,最後得到 8』
,請你幫小超超算算看,某數是多少?(A) 13 
(B) 49  (C) 15  (D) – 3 
14.  (      )  請問 x=­2 是下列哪一個一元一次方程式的解?(A)-2+ 

x=0 (B)3x-6=12 (C)4x+8=0 (D)6-2x=2 

15.  (      )  下列四個選項中,何者才是一元一次方程式? 

(A) - 3 + x = x 2 + 2  (B) - 3 + x = y + 2  (C) 2x - x = 0  (D) 2 x - x 。 

x + 2  x - 1 
16.  (      )  -  =1,則 x=?  (A) 0 (B) 1 (C) 2 (D) 3。
3  2 
2  2  1
步驟○
17.  (      )小馬哥解方程式  x-1=5 的步驟如下:  x-1=5  ──→ 
3  3 

步驟○ 3
步驟○
2x-3=5 ──→ 2x=8 ──→ x=4,請問下列敘述哪一個是正

1就錯了(C)步驟○
確的?  (A)他得到正確的結果(B)步驟○ 2錯

3錯了。 
了(D)步驟○
4
18.  (      )  某百貨公司歲末出清,將原價 x 元的衣服,以  ( x - 50 ) 售

出,試問下列哪一個選項,可以作為此百貨公司的優惠標語? 

(A)原價打 8 折再減 50 元(B)原價打 4 折再減 50 元(C)原價減 50 

元再打 8 折(D)原價減 50 元再打 4 折。 

19.  (      )小光去路邊攤買衣服,老闆說每件褲子比每件衣服貴 60 

元,小光買了 4 件衣服,2 件褲子,拿了一張千元紙鈔給老闆,

老闆找了小明 100 元,則褲子每件多少元?(A) 100 (B) 130 

(C) 150 (D) 190 元。 

20.  (      )佳佳到唱片行花了 169 元買了一片 CD,這片 CD 是標價

打九折後,再用折價卷折價 20 元買到的,那麼這片 CD 標價為

多少元?  (A) 200 元(B) 210 元(C) 240 元(D) 250 元。 

21.  (      )  慶生會當天準備了若干顆糖果,準備平均分給與會來

賓,如果每人分 8 顆,還剩下 23 顆糖果;如果每人分 11 顆,

則有 2 個人沒有分到糖果,請問共分給多少人? 

(A) 12 (B) 13(C) 14 (D) 15 人。
22.  (      )解方程式  4x – 6 = 2x + 8  ,得  x  是多少? 

(A)4  (B)5  (C)6  (D)7 

23.  (      ) x  的一元一次方程式:  6x = 2x,其中  x  的解為 


(A) x = 4  (B) x =  (C) x = –4    (D) x = 0 

24.  (      )  下列哪個選項,是  36 – x ¸ 5 = 6  中  x  的值?(A) x = 6 ´ 

5 + 36(B) x = ( 36 – 6 ) ´ 5(C) x = ( 36 + 6 ) ´ 5(D) x = 6 ´ ( 36 – 


5 ) 
25.  (      )解方程式 2(3 x - 1 ) - 5 (- x + 3 ) = 5 ,得 x =? 

4  18 
(A)2  (B)22  (C)  (D)  。 
11  11 

26.  (      )  下列各式中,何式不是一元一次方程式? 

(A)  - x = 3 (B)  1 - x = 3 (C)  - x + 3 = 7 (D)  x · x - 3 = 7 。 

27.  (      )  下列各式中,何式是一元一次方程式? 
3 3 
(A)  - 3x + 2 y = 12 (B)  5 x  + 10 = 22 (C)  x - 2 = 45 x + 11 (D)  -  = 1 
x  4 

28.  (      )  當 m 為多少時,會滿足方程式  5m+11=121? 


(A) 22(B) 33(C) 44(D) 55 
29.  (      )  試求出方程式  3 ( 3 x  -  4 )  - 3  = 75  ? 

22 22 22 22
(A)  x =  (B)  x =  (C)  x =  (D)  x = 
3  9  7  13 

30.  (      ) ­32.5 是哪一個方程式的解?
2 1 
(A)  2 x - 1 = 11 (B)  -  x - 12 = 1 (C)  8 x -  = 2 (D)  2 x - 3 2 = 4 
5  11 

31.  (      )  下列哪一個一元一次方程式無解? 

(A) 3 x + 7 = 11 (B) 5 x - 3 = 5 x + 3 (C) 2 x - 1 = 2 (D) 5 x - 3 = 2 x + 3 x - 3 

32.  (      )哪一個一元一次方程式的解與其它三個不同? 

9 2  1  32  7 
(A) 4 x - 4 = 8 (B)  x +  = x + (C) 0. 5 x +  = 1 . 23 x - 0 . 44 (D) 
4  5  4  5  4 

4 x - 3 = 7 x + 15 。 

33.  (      )  若 5 為此 x 的一元一次方程式  3 x + 2 - 4 b = 2 b - 1 之解,則 

1  3  7  5 
b=  (A)  (B)  (C)  (D)  。 
3  2  2  2 

34.  (      )  若 3 為此 x 的一元一次方程式  7 x + 1 - 5 a = 12 之解,則 a= 

(A)2  (B)3  (C)4  (D)5  。 

35.  (      )  數學老師公布第一次段考成績,最高分為 80 分,阿呆分

數的 3 倍再減 20 分就是全班最高分的 2 倍,求阿呆考幾分? 

(A) 60(B) 90(C) 30 (D) 70 

36.  (      )  數學老師公布第一次段考成績,最高分為 100 分,阿呆

分數的 3 倍再加 10 分就是全班最高分,求阿呆考幾分? 

(A) 60(B) 90(C) 30 (D) 70。 

37.  (      ) a、b 都不等於 0,a、b 兩數互為相反數、下列何者正確? 

(A)A=B  (B)A=  B  (C)A+B=0  (D)–A=­B


38.  (      ) x 是何值時,(2 x ­ 5)與(9 x +16)互為相反數? 

(A)­2  (B)­1  (C)0  (D)1  。 

39.  (      )父子的年齡和是 80 歲,且父的年齡恰是子的年齡的 3 倍,

求父子相差幾歲?  (A)30(B)40(C)50(D)60 

40.  (      )父子的年齡和是 120 歲,且父的年齡恰是子的年齡的 2 

倍,求父子相差幾歲?  (A)30(B)40(C)50(D)60 

41.  (      )小呆帶 500 元,以原價的八折購買一雙皮鞋,不夠 100 元,

則皮鞋原價是幾元?(A)450(B)550(C)650(D)750 

答案: 
1.C  2.B  3.B  4.C  5.B  6.B  7.B  8.C  9.A  10.A 
11.D  12.B  13.A  14.C  15.C  16.B  17.B  18.C  19.D  20.C 
21.D  22.C  23.D  24.B  25.A  26.D  27.C  28.A  29.B  30.B 
31.B  32.D  33.D  34.A  35.A  36.C  37.C  38.B  39.B  40.B 
41.D 

二、 填充題 

1.  某數的四分之一减去某數的七分之一,比某數的八分之一少 

15,則某數為 。 

2.  解 2{3 [4 (5 x + 1 ) + 8 ] + 20 } - 7 = 225 ,x= 。


x + 3 2 x + a  7 
3.  若方程式  - = 的解為 x = 2 ,求 a =  。 
3  2  6 
3  4
4.  若 1 - 2 x = ,則  x + 1 的值為 。 
4  3 

5.  有一個矩形的周長為 68 公分,如果長比寬的 2 倍少 5 公分,那

麼此矩形的面積為 平方公分 。 

1  1 
6.  小玉原有若干元,他用原有錢的  買筆記本,再用剩下錢的  買
4  5 

文具用品,共計花去 200 元,則他原有 元。 

7.  一個三角形面積為 15 平方公分,若底邊長為 (2 x - 2 ) 公分,對應

高長 3 公分,則 x 的值為 。 

8.  一支棒球隊在已經比過的 60 場比賽中,勝率為 5 成,從現在開

始,該球隊需要連贏 場,才能使勝率提高為 6 成。 

9.  若 a 為一元一次方程式 8a - 3 - {- 5 + 4 [3 a - (6 a - 4 )]} = 5 的解,則 a 的

值為 。 

4 x - 6  - 2 x - 3  3 
10.  解一元一次方程式:  - = ,x= 。 
8  4  4 

11.  若  x + 1 + y - 2  + z + 3  = 0 ,則 x +  y + z  = 。 

x  x  x  x 
12.  若 x = + + + ,則 x =  。 
2  2 2  2 3  2 4 

13.  一個二位數,其個位數字與十位數字的和為 8,如果原數減 10,

等於十位數字與個位數字互換後所得新數的 2 倍,則此二位數
為 x =  。 

0 . 36  0 . 09 x - 0 . 18 


14.  解一元一次方程式 1. 5 =  + ,則 x =  。 
0 . 2  0 . 9 

15.  妙麗在魔法學校新學了一種「猜心術」的魔法,她對容恩說:「

你想一個數,將此數乘以 5,再加 4,再把整個答案乘以 2,最

後減去 8,告訴我你的答案,我就可以猜出你心裡想的數!」若

容恩告訴妙麗的答案是 160,則容恩心中想的數字

為 。 

16.  解一元一次方程式 0. 9 x - 3 = 2 . 4 x ,則 x =  。 

2 1 
17.  解一元一次方程式  x + 6 = x ,則 x =  。 
5  3 

2 x + 5  - x - 1 
18.  解一元一次方程式  = ,則 x =  。 
3  2 

4  6  x - 1 
19.  解一元一次方程式 2 x -  = - ,則 x =  。 
5  5  4 

3 - 1 
20.  解一元一次方程式  (2 x - 5 ) = ( x - 11 ) ,則 x =  。 
5  2 

21.  解一元一次方程式 4[3 (2 x - 1 ) + k ] = -24 ,得 x = -3 ,則

k  =  。 

22.  有一四邊形 ABCD,其四個內角分別為 

x o , ( x + 40 ) o , ( x + 10 ) o , ( x - 10 ) o  ,則四個內角分別

為 , , , 。
23.  英明最喜歡和數學老師比身高,有一次老師略帶玄機告訴他:

14 
老師身高的  倍比 170 公分少 2 公分,則老師的身高
13 

為 。 

24.  若三個連續奇數總和為 69,則這三個連續奇數中最大數

為 。 

25.  家銘和朋友到悠活游泳池游泳,買了 15 張優待票和 10 張普通

票,總共花了 4050 元,已知一張優待票比一張普通票便宜 30 

元,則一張普通票要 元。 

26.  已知大寶、二寶、三寶共有 157 元,若大寶的錢變為一半、二

寶的錢少 7 元、三寶的錢變成 3 倍之後,三人錢數就相等了,

則原來大寶有 元,二寶有 元,三寶有

元。 

27.  戶外教學時,法拉利教官被指派購買電池與底片,已知一卷底

片與一顆電池的總價為 220 元,又底片的價錢是電池的 3 倍多 

20 元,若小美教官拿了 1000 元給法拉利教官,要買 2 卷底片、 

3 顆電池,則法拉利教官應找回 元。 

28.  某次信樂團演唱會預售一張門票 500 元,但因當天到場人數太

少,所以現場有黃牛以 7 折的價錢販賣門票,已知今天小丸子

和朋友共 6 人一同去欣賞演唱會共花了 2700 元,則他們買了
張黃牛票。 

29.  某日營養午餐有麥克雞塊若干個,若每個人拿 2 塊,會剩下 13 

塊;若每個人拿 3 塊,會不夠 20 塊,則雞塊總共有 塊。 

30.  二年某班在開學時測量 BMI 指數,若 BMI 指數大於 25 必須參


加享瘦班:已知 BMI 大於 25 的人數佔了全班的  ,BMI 介於 

20~25 的人數佔了全班的一半少 1 人,而 BMI 小於 25 的有 7 

人, 則 BMI 大於 25 必須參加享瘦班的有 人。 

31.  某次數學測驗共 50 題,答對一題給 2 分,答錯一題倒扣 1 分,

不做答則不計分,若小黃做答 43 題,得分 68 分,則小黃答對

題。 

32.  請利用等量公理或移項法則解下列各方程式: 

(1) x + 80 = 60,x =______。(2) x ­ 60 = 80,y =______。 

(3) 7 z = 84,z =______。(4) w ÷ 3 = 12,w =______。 

33.  請利用等量公理或移項法則解下列各方程式: 

(1) 35 + x = 25,x =______。(2) 48 ­ x = 80,y =______。 

(3) z × 15 = 90,z =______(4) 65 ÷ w = 13,w =______。 

34.  請利用等量公理或移項法則解下列各方程式: 

(1) 4x + 800 = 600,x =______。(2) 2( y + 4 ) = 16,y =______。
(3) ( z – 20) ÷ 2 =30,z =______。(4) 7 ( w + 20 ) = 210,w 

=______。 

35.  一個蛋糕 x 元,切成同樣大小的 8 小塊,若小明買了 3 小塊,

付了 60 元,求  x =  。 

36.  若中性筆的單價比自動鉛筆的單價貴 5 元,且 8 枝中性筆和 6 

枝自動鉛筆共需花費 390 元,則每枝中性筆需 元,

每枝自動鉛筆需 元。 

37.  某電信業者推出促銷專案,月租費 150 元,可免費通話 2000 

秒,超出 2000 秒的部分以每秒 0.08 元計費,阿寶九月分的電

話費為 550 元,則阿寶這個月通話時間為 秒。 

38.  學生分配宿舍,若 5 人住一間,則有 2 人無宿舍可住;若 6 人

住一間,則剩餘宿舍 2 間,問學生共有 人。宿舍有

間。 
1  1 
39.  小超超有一筆存款,他花了存款的  買一雙球鞋、  買一部電子
5  4 

辭典,還剩下 6600 元,則小佑原有存款 元。 】 

40.  小雄雄有收藏拾元及伍元硬幣的習慣,已知他已收藏了 203 個

硬幣,且這些硬幣一共值 1890 元,則他收藏了 個拾

元硬幣, 個伍元硬幣。 

41.  優質農場裡所生產的牛乳製品,牛奶糖一包 50 元,鮮乳一瓶原
價 35 元,今天進行鮮乳大特賣,6 瓶鮮奶裝成 1 袋,若堯哥買 

2 袋鮮奶及 2 包牛奶糖,他拿 1000 元鈔票付錢,找回了 600 元,

則平均 1 瓶牛乳特價後是 元。 

42.  小茹茹做科展,將一特殊材質的球由某高度落下,每次反彈的

高度均是原來的 3 ,反彈兩次之後,離地面的高度是 9 公尺,

則原來的高度是 公尺。 

43.  解方程式 3(x+1)-2=2(x+1)-1,則 x =  。 



44.  若 3 x - 5  = 7  - ( - 2 x ) ,且  y = 8  - y ,則 x +  y  =  。 

45.  請解以下各方程式: 

x  x 
(1)  -  = 1 ,則 x =  。 
3 5 
x  x  x 
(2)  -  - = 1 ,則 x =  。 
3 5  7 
x  x  x  x 
(3)  -  - - = 1 ,則 x =  。 
3 5  7  105 

46.  如果-2x=2x,則 x =  。 

47.  若一班級 5 人,第二次段考完後,有 1 人排名進步 1 名,有 2 

人排名退步 2 名,有 1 人維持原排名,則另一名的排名是進步

或退步 名。 

48.  花媽跟柚子到學校操場去慢跑。半個小時,花媽跑了三圈又 24 

公尺,是柚子跑的距離的四分之一倍。已知柚子半小時跑 2.4 

公里,則操場一圈 公尺。
49.  已知花媽比他的兒子柚子大 29 歲。五年後,花媽的年齡是柚子

年齡的 3 倍少 5 歲,則柚子今年 歲。 

50.  是非題:(下列各題計算結果是否正確,請填「○」或「×」) 

1 1 
(1)  x -  x = 1 ,其解為  x = 1 。 
2  3 

(2) - 8 = 24 y ,其解為  y  = -3 。 

51.  解下列各一元一次方程式: 

(1)4x – 8 = 6x,則 x =  。 

(2)8 + x = 9,則 x =  。 


(3)  x = 0,則 x =  。 

(4)1.2x = 6,則 x =  。 

1  1 
(5)  x –  x = 1,則 x =  。 
6  8 

52.  是非題: 


(1)2x + 1 =  + 5  是一個一元一次方程式。 

(2)當等號左右兩邊相等時,在等號左右兩邊各除

以同一個數時,等號兩邊仍維持相等,以上說的是「等量(除

法)公理」。 

53.  解一元一次方程式 17 ( 12 – 5x ) = –34 ( 7x – 6 ), x = ________。


- x + 1 x - 2
54.  解一元一次方程式  =  ,x = ________。 
2  3 

55.  解下列方程式: 

(1)–( x – 3 ) + 2x × 3 = –2,x = ________。 
2 x + 1  x - 5
(2)  – 1 =  ,x = ________。 
3  2 

56.  利用「等量公理」,求出下列各題中未知數所代表的數: 

(1)  3 x + 700 = 8 x + 200 ,x = ________。 

2
(2)  y - 6 = 18 ,x = ________。 

(3)  24 - 4 z - 3 z  = 3 z + 4 ,x = ________。 

57.  解一元一次方程式: 

(1)  2x – 14 = 8,x = ________。 

(2)  4x – 5 = 12 – 2x,x = ________。 

(3)  3x + 15 = 21,x = ________。 

(4)  4x – 5 = 10 + x,x = ________。 

58.  一年甲班有一次聚餐,買了  8  杯綿綿冰,  6  杯四果冰,共付

了  450  元,如果每杯綿綿冰  x  元,四果冰比綿綿冰每杯多  5 

元,依題意列出等式: 。 

59.  解下列一元一次方程式:
(1)若 x – 2 = 7 的解為:x = ________。 

(2)若 4y – 2 = 3(y – 5)的解為:y = ________。 

2  4 
(3)若  x =  的解為:x = ________。 
3  5 
4
(4)若 y ¸  = 4 的解為:y = ________。 

60.  解下列一元一次方程式: 

(1)  –1 = 6x + 2,x =______。 

10 
(2)  x =  x – 70,x = ______。 


(3)  17x =  x,x =______。 
14 

(4)  25 + 0.3x = –2x + 2,x = ______。 

(5)  7x + (–x) + 19 = 14x – ( 15 – 9x ),x =______。 

61.  設 x = 5 為  x  的一次方程式 ax – 5 = 6a – 3x 的解,則 a =______ 


。 

62.  求出下列各式的解: 

(1)  x + 1 = 3 ,x = ______。 

(2)  2 x + 1 = 3 ,x = ______。 

(3)  3 x + 7 = 5 ,x = ______。 

2 1 
(4)  x +  = 5 ,x = ______。 
3  3 

63.  求出下列各式的解:
(1)  3 x = 15 ,x = ______。 

(2)  - 3 x = 18 ,x = ______。 

1
(3)  x = 2 ,x = ______。 

3 2 
(4)  x =  ,x = ______。 
7  3 

64.  相反數的應用,回答下列各題:(1) ­3 的相反數為 

(2)  兩數互為相反數,則兩數之和為  (3) 0 的相

反數為 。 

65.  回答下列各題:(1) x 的相反數為 y,若 2 x - 1 = 11 ,則 y=________。 

(2)  6 x - 12 的相反數為 48 ,則 x =  ________。  (3)  當 x = y 時, 

7 x - 10 的相反數為 - 4 ,則 y 的相反數為________。 

66.  利用等量公理完成以下各題求解: 

3
(1)  x = 7  ,x = ______。 

(2) - 7 = 2 x  ,x = ______。 

(3) 0. 23 x = 0 . 46  ,x = ______。 

67.  利用等量公理完成以下各題求解: 

(1) 5 x - 3 = 4 x + 2  ,x = ______。 

(2) 4 x - 1 = -3 x + 20  ,x = ______。


1 3 
(3)  x + 11 = x - 1  ,x = ______。 
3  4 

68.  請解以下各方程式: 

2 x - 1 
(1)  = 3 ,x = ______。 
15 
2 x - 1 
(2)  = 5 x ,x = ______。 
15 
2 x - 1  1 - 3 x 
(3)  = ,x = ______。
3  5 

答案: 


1.840  2.  x = 5 3.  a = -3 4.  5. 273  6. 500  7.  x = 6 8.15 

19  3  16 
9.  10.  11.  - 2 12.  13.62  14.  x = -1 15.16 
20  4  15 
13
16.  x = -2 17.  x = -90 18.  x = - 19.  x  = 1 20.  x = 5

21.  k  = 27 22. 80 o , 120 o , 90 o , 70 o  23. 156  24. 25  25. 180 

26. 90;52;15  27. 510  28. 2  29. 79  30. 12  31. 37 


32. (1)­20 (2) 140 (3) 12 (4) 36  33. (1)­10 (2) ­32 (3) 6 (4) 5 
34. (1) –50 (2) 4 (3) 80 (4) 10  35.160  36. 30;25  37. 7000 

38. 72;14  39. 12000  40. 175;28  41. 25  42. 16  43. 0  44. 15 
105
45. (1)  15  (2) ­105(3)  -  46. 0  47. 3  48. 192  49. 12 
2  2 

50. (1)×(2)○  51. (1) –4 (2) 1 (3) 0 (4) 5 (5) 24  52. (1)○(2)×



53. 0  54.  55. (1)–1(2)–11  56. (1) 500 (2)36 (3) 10 

17 
57. (1)11 (2)  (3)  2 (4)5  58. 8x + 6 ( x + 5 ) = 450 

3  1 
59. (1)9(2)–13(3)  (4)5  60. (1)–  (2)30(3)0(4)–10(5) 2  61. 10 
5  2 
2 14 
62. (1) 2 (2) 1 (3) -  (4)7  63. (1) 5(2) ­6(3) 6(4) 
64. (1) 3(2) 0(3) 0 
3  9 
14  7 144 
65. (1)­6 (2) ­6  (3) ­2  66. (1)  (2) -  (3)2  67. (1)5 (2)3 (3) 
3  2  5 
1 8 
68. (1) 23(2)  -  (3) 
73  19 

三、 綜合、題組題 

1.  某次籃球比賽因為下雨的關係,所以順延兩週舉行,已知原訂

兩天的日期與順延後兩天的日期 4 數總和為 86,請你說出原訂

第一天籃球比賽的日期(即問 A 的日期)。(可參考底下圖表) 

日 一 二 三 四 五 六 
A  B 

C  D

2.  (1)段考後大雄發現他的數學成績和靜香的成績和剛好 100 分,

倘若靜香給大雄 10 分,靜香的成績剛好是大雄的 4 倍,假設大

雄考了 x 分,

請你列出一元一次方程式:_______________________________ 
(2)解出可得大雄考_____________分。 

3.  四個連續奇數,其中最小的數為 x,請問: 

(1)若  x= 11,則四個連續奇數分別為 。 

(2)這四個連續奇數中,最大的數為 。(以 x 表示) 

(3)若這四個連續奇數的總和是 144,則這四個連續奇數分別

為 。 

4.  一雙運動鞋以定價九折售出,可賺 240 元;若以定價七折售出,

賠本 80 元,假設定價為 x 元,則: 

(1)可列得 x 的一次方程式為_____________________。 

(2)成本為 元。 

5.  請利用「等量公理」解下列方程式,並且驗算:–8x + 5 = –3x + 
15 

6.  利用「等量公理」解方程式 x + 7 = 4。 

7.  請判斷下列解一元一次方程式的過程是否正確?並且加以說

明。若­8x=16,則 x=16+8=24。 

8.  以下是 A、B、C 三位學生解方程式的步驟,請分別寫出哪些步

驟錯誤,加以訂正: 

A 學生: 

2 3  5
題目  x -  = 1 =>步驟 1  8 x - 9 = 1 =>步驟 2  8 x = 10 =>步驟 3  x = 
3  4  4 
B 學生: 

題目 3 x = -4 x =>步驟 1  3 x + 4 x = 0 =>步驟 2  7 x = 0 =>步驟 3  x=無解 

C 學生: 

題目 9 x + 11 x = 3 x =>步驟 1  9 + 11 = 3 =>步驟 2  20 = 3 無解

答案: 

1.14  2.(1) (100­x)­10=4(x+10) (2) 10 分  3. (1) 11,13,15,17 

(2) x + 6(3) 33,35,37,39  4.(1) 0.7x+80=0.9x-240(2)1200 

5. x = –2 
【驗算】左式= –8 ´ (–2) + 5 =16 + 5 = 21 

右式= –3 ´ (–2) + 15 = 6 + 15 = 21 

左式=右式

所以  x = –2  是原方程式的解。 

6. x + 7 – 7 = 4 – 7,x = –3 

7.  錯誤。此題過程應改為­8x=(­8)× x=16→x=16 ÷(­8)=  ­2 

8. A:  步驟 1  8 x - 9 = 12 

B:  步驟 3 x=0 

C:  步驟 1  20 x = 3 x Þ 17 x = 0 Þ x = 0 


貳、 進階題

一、 選擇題 

1.  (      )  凱哥進入職棒隊,第一年的年薪是 x 元,因第一年表現良

好,所以第二年加薪 10%,但第二年因傷表現欠佳,於第三年

又減薪 10%,已經知道他第三年年薪為 100000 元,依照題意

1  1 
列出一元一次方程式為下列何者?(A) ( 1 +  –  ) x = 100000 
10  10 
1  1  1  1 
(B) x  +  –  = 100000 (C) (x +  )( x –  ) = 100000  (D) 
10  10  10  10 
1  1 
( 1 +  )( 1 –  ) x = 100000 
10  10 

2.  (      )  已知  a=5  是一元一次方程式的解,-9 (3-2a) = 15-8k 

的解,求 k=?(A)6(B)–6(C)11(D)–11 

3.  (      )  請問下列選項何者係以移項法則解方程式 

36 -  x  ¸ 7  = 6  ?(A)36-x÷7-36=6-36(B) 36-x=6×7(C)36 

-x÷7=6+x÷7(D)36-6=x÷7 
4 x 
4.  (      )  竹竹解一元一次方程式  -  4  = 8 的步驟如下: 

4 x 
- 4  = 8  ¾ 步驟 
¾¾1 ® 4 x  - 4  = 24  ¾ 步驟
¾¾2 ® 4 x  = 28  ¾ 步驟
¾¾3 ® x  = 7 

1
請問下列敘述哪一個是正確的?(A)她得到正確結果(B)步驟○

2 錯了(D)步驟○
就錯了  (C)步驟○ 3 錯了。
5.  (      )  -12 與 22 兩數各減一個相同的數之後,所得新的兩數互

為相反數,若所減的數為 x,則依題意可列出下列哪一個算式? 

(22- x)=0  (B)(-12- x)-(22- x)=0 


(A)(-12- x)

- 12 -x 
(C)(-12- x)+(22- x)=0  (D)  =1。 
22 -x 

6.  (      )康康解出方程式 (x-3)-4(x+1)=-3 的 x 值後,將 

x 值標示在數線上,則下列哪一個圖示是 x 值的大略位置? 

(A)  (B) 

(C)  (D)  。 

7.  (      )  以下敘述,何者正確?(A)  今年姊姊的年齡是妹妹的 2 

倍,則 3 年後姊姊的年齡仍然是妹妹的 2 倍。(B) 天平左邊放

一顆橘子、一顆蘋果,右邊放一顆橘子、二顆檸檬,則一顆蘋

果的重量等於二顆檸檬的重量。(C)  若  xz = yz ,則  x = y 。(D) 

若  2 x + 4 = 4 y + 6 ,則  x + 2 = 2 y + 3 。


8.  (      )  如下圖所示,將重量不同的三種積木置於等臂天平兩側

(同一種積木重量相同)
,且天平呈平衡狀態,則在以下那一種

情形中,天平不會平衡? 

選項 左盤 右盤 

(A) 
(B) 
(C) 
(D)

9.  (      )在建華文具店裡,4 塊橡皮擦的價錢等於 2 枝鉛筆的價錢 

(每塊橡皮擦價錢一樣,每枝鉛筆價錢一樣),則下列東西價錢

的大小關係,那一個是錯誤的?(A)  1 塊橡皮擦  < 1 枝鉛筆(B) 

2 塊橡皮擦  < 3 枝鉛筆(C)  6 塊橡皮擦  < 4 枝鉛筆(D)  2 塊

橡皮擦  + 6 枝鉛筆  < 6 塊橡皮擦  + 2 塊橡皮擦。 

10.  (      )  以下是巴斯向音速說明證得  4 = 3  的步驟。

已知: 

x  =  2 

(第
(第一
一步
步驟
驟)
) (等
ò(等量
量加
加法
法公
公理
理)


x + 3x =  2  + 3x 
(第
(第二
二步
步驟
驟)
) (等
ò(等量
量減
減法
法公
公理
理)
) 

4x – 2 = 3x –  2 

(第
(第三
三步
步驟
驟)
) (等
ò(等量
量乘
乘法
法公
公理
理)
) 

8x – 4 = 6x – 3 
(提
ò(提出
出公
公因
因式
式)
) 
4 ( 2x – 1 )= 3 ( 2x – 1 ) 
(第
(第四
四步
步驟
驟)
) (等
ò(等量
量除
除法
法公
公理
理)
) 
4  = 3 
試問:音速指出巴斯錯誤的地方為何?(A)  第一
第一步
步驟
驟。(B) 

第二
第二步
步驟
驟。(C)  第三
第三步
步驟
驟。(D)  第四
第四步
步驟
驟。 

11.  (      )已知現年父、子年齡和為 58 歲,且 5 年前父親年齡是孩

子年齡的三倍,請問父、子相差多少歲?(A) 17  (B) 18    (C) 24 

(D) 41。 

12.  (      )某測驗有 10 道題目,答對一題得 3 分,答錯一題扣 2 分,

若沒有作答則不計分。如果小玉自行對完答案後,知道自己只

答對一題,但總分為-9 分,那麼她答錯了幾題?(A) 4 題  (B) 

5 題  (C) 6 題  (D) 7 題。 

13.  (      )小新過年收到的壓歲錢共 x 元,他的分配使用方式如下:

其中一半的錢拿來做為儲蓄存款,另一半的錢則支出在添購一

些用品上,在小新添購的用品當中買新衣服用了  x ,買文具用

1  1 
品用了  x ,買遊戲卡用了  x ,請問下列何者正確?(A)小新買
6  12 

新衣服占所有支出的  (B)小新買文具用品和遊戲卡所花掉的

錢比買新衣服的錢多(C)小新對於壓歲錢的分配使用當中,買新

衣服所分配的金額最高(D)以上皆非。 

14.  (      )  下列各式中,何式是一元一次方程式?(A)  - 3 x + 12 (B) 

x 2 + 10 = 22 (C)  - 23 x - 2 3  = 3 2 (D)  x = y + 5 。 

15.  (      )  已知 2 x - 1 = 9 則  x = (A)  只有 5(B)  只有­4(C) 5  或  ­4(D) 

無解。 

16.  (      )  若  5 x + 4 = 4 x - 1 , 2 y + 11 = 3 y - 3 則下列何者正確? 

(A)  xy = -80 (B)  x + y = -9 (C)  x 為質數(D)  y 為偶數。 

17.  (      )  3 x - 4 = 4 x + 2 的解 x = m, 8 x + 12 = 3 x + 22 的解 x = n  則下列何

者正確?(A) m÷n=2 (B) m×n=10(C) m 是奇數(D) n 是質數。 

18.  (      )  某數的 3 倍加 5 再 4 倍為­4,則某數=  (A)­1(B)­2 (C)­3 

(D)­4。 

11  13 
19.  (      )  某數的 2 倍減 3 是 5,則某數=(A)4(B)  (C)  (D)3。 
5  5 

20.  (      ) b 是 a 的 2 倍少 7,且 a、b 的和是 23,則 a-b=?(A) 2(B) 

3 (C) 4 (D)5。 

21.  (      )  甲是乙的 9 倍 且甲、乙的和是 50,則甲-乙=?(A) 35


(B)40(C)45 (D)50。 

22.  (      )  已知三個連續偶數的和是 60,則  (A)最小的數是 20(B) 

中間的數是 22(C)最大的數是 24(D)最大的數與最小的數之和為

中間的數的 2 倍。 

23.  (      )  已知三個連續奇數的和是 3003,則  (A)最小的數是 

997(B)中間的數是 999(C)最小的數是 999(D)最大的數是 1001。 

1  1 
24.  (      )  某數扣掉某數的  ,再扣掉某數的  ,最後剩下 7,則某
3  9 

數是  (A)13(B)14(C)15(D)16。 

1  1  1 
25.  (      )  某數的  減去它的  的差是 20,則某數的  是? 
3  9  2 
(A)30(B)35(C)40(D)45 
26.  (      )  -38 與 10 同加多少後會成為相反數? 
(A)13(B)14(C)15(D)16 
27.  (      )  -40 與 6 同減去多少後會成為相反數? 
(A)­16(B)­17(C)­18(D)­19 
28.  (      ) 0 的相反數為(A) 1(B) ­1(C) 0(D)不存在。 

29.  (      ) 4+x 後所得新數與 15 互為相反數,則 x=(A) ­15(B) 


­16(C) ­18(D) ­19 
30.  (      )資優班入學測驗數學科共 20 題,答對一題 5 分,答錯一

題倒扣 1 分,沒有作答則不予計分,若小立作答 18 題,得 66 

分,則小立答錯幾題?  (A)2  題(B)3 題(C)4 題(D)5 題。 

1  1 
31.  (      )把 100 分解成 a、b 兩數,a 數的  與 b 數的  的和是 10,
15  5 
b=?(A)25(B)26(C)27(D)28。 

32.  (      )五個連續奇數的和是 95,中間數是(A)15(B)17(C)19(D)21 

33.  (      )五個連續偶數的和是 90,中間的數是多少? 

(A)16(B)18(C)20(D)22 

答案: 

1.D  2.B  3.D  4.B  5.C  6.D  7.D  8.D  9.D  10.D 
11.C  12.C  13.C  14.C  15.C  16.D  17.D  18.B  19.A  20.B 
21.B  22.D  23.C  24.C  25.D  26.B  27.B  28.C  29.D  30.C 
31.A  32.C  33.B 

二、 填充題 

1.  某年的五月有 4 個星期日,已知這四個星期日的日期數和是 

66,則該年的五月二日是星期 。 

2.  某人沿相同的路徑上、下山一趟,共需 4 小時 40 分,已知上山

每小時可走 1.5 公里,下山每小時可走 2 公里,則山路長(僅上

山或下山一趟)為 公里。 

3.  一段路程甲、乙兩人同地同向出發,甲讓乙先走三分鐘,當乙

到達終點時,甲也同時抵達終點,已知甲分速為 15 公尺,乙分

速為 12 公尺,則此段路程全長為 公尺。 

4.  如下圖,四邊形的面積為 12 平方單位,則 x =  。



x+2 
2

x­1 

5.  糖果若干個,小黃取一半又多一個,小偉取剩餘的一半又多兩

個,小霖再取剩下的一半又少三個,最後還剩下 5 個,則原有

個糖果。 

6.  魔法學校 50 週年校慶,為迎接這次校慶來到,總務處整修校

園,工程原定 10 名工人,連續 12 天完成,現工作 6 天後,因

雨停工 4 天,則至少需加派 名工人,才能如期完工,

不致影響校慶活動。 

7.  有兩枝等高但不等粗的香,第一枝香 3 小時可以燒完,第二枝

香 2 小時可以燒完,若同時點燃且以一定的速率燃燒,則在燃

燒 小時後,第一枝香的高度會等於第二枝香高度的 3 

倍。 

8.  魔法學校某班共 50 名學生,報名參加數學和天文研習營,已知 

30 名學生參加數學研習營,參加天文研習營的同學比參加數學

研習營的同學多 3 人,兩組都不參加的人數比兩組都參加人數

的三分之一多 1 人,則兩組都參加的人數有 人。 

9.  有一堆放在一起且分別標有 1 至 10 號碼的 10 捆鋼條,每捆有 
100 根,已知其中 9 捆是同一規格,每根重 10 公斤,另一捆是

另一規格,每根重 13 公斤,現在從 1 號捆中取 1 根,2 號捆中

取 2 根,以此類推直至 10 號捆中取 10 根,一起過磅秤出總重

量為 571 公斤,則與其他鋼條不同規格的標號是 號。 

10.  某次數學課秩序不佳,林老師將全班 30 人的椅子搬走部份只剩

下 18 張,讓大家輪流站著上完一節 40 分鐘的課程,如果一張

椅子只能坐一個人,則平均每人須站 分鐘。 

11.  解下列各方程式: 

(1) 0.4x + 8 = 6,x =______。 


(2)  ( y + 14 ) + 10 = 16,y =______。 


(3) ( z – 27 ) ÷  + 25 = 30,z =______。 


(4)  ( 2w + 18 ) = 32,w =______。 

12.  解下列各方程式: 

(1) ( 4x – 3 )  -  ( 2x + 1 ) = 3x + 2,x =______。 

1  1 
(2)  ( x – 2 ) =  ( x + 6 ) +2,x =______。 
3  4 

13.  已知多啦 A 夢的體重為 Kitty 體重的 3 倍多 4 公斤,如果多啦 

A 夢的體重是 x 公斤,而且多啦 A 夢比 Kitty 重 120 公斤,則

多啦 A 夢的體重是 公斤。


14.  一年 12 班全班有 32 人,第二次段考數學及格的有 20 人,數學

及格、英文也及格的有 12 人,至少有一科及格的人有 23 人,

則英文考不及格的有 人。 

15.  已知超哥、凱哥、憲哥三個人的年齡和是 93,如果超哥的年齡

加 8 歲比憲哥年齡的兩倍少 20 歲,凱哥比憲哥大一歲,求三人

的年齡分別是超哥: 歲、凱哥: 歲、憲哥:

歲。 

16.  小貞貞手邊有 5%的食鹽水和 10%的食鹽水若干,想要調出 8% 

的食鹽水,已知她使用了 5%的食鹽水 200 公克,則她必須再用 

10%的食鹽水 公克才可以調出她想要的濃度。 

5 + x  1 
17.  解方程式  - (90 - x )  = -6 ,x =  。 
4  3 

3 x  - 1 
18.  解方程式 5 ( 1 -  x )  + 2 x  = - ,  x =  。 

19.  若方程式 2x+3a=5x-6 與 3(x-2)=x+a 有相同的解,則 a 

之值為 ;x= 。 

20.  五年前新新 a 歲、竹竹 a+3 歲,若五年後竹竹年齡是新新年齡

的  倍少 7 歲,則竹竹今年 歲。 

21.  解下列各一元一次方程式:
6 - x 
(1) x + 8 - = 2  ,  x =  。 

1 1 
(2)  (2 x - 5 ) - ( x - 4 ) = 1  ,  x =  。 
4  6 

22.  小明到麥當勞,用去原有的三分之一少 4 元買漢堡,再用剩下

的四分之一多 5 元買可樂,後來再用剩下的一半少 1 元買薯條,

最後剩下 45 元,則小明原有 元。 

23.  求出下列各式中  x  所代表的數? 

(1)  3 x + ( -8 ) = -53 , x =  。 

(2)  15 x  = 2 x + 9 , x =  。 

1
(3)  x - 3 = -2 , x =  。 

(4)  3( x + 5 ) - 4 = 11 , x =  。 

(5)  - 4 x + 7  = -6 x + 13 , x =  。 

24.  有一個長方形的長比寬的  2  倍少  3  公分,若這個長方形的周

長是 42 公分,則此長方形的面積為 平方公分。 

25.  小丸子全家去吃海鮮,看到餐廳的菜單如下,他們於是抓了兩

隻石斑魚,分別用清蒸和三杯的口味 石斑活魚三吃
以兩計重 時價計算
烹飪,在結帳的時候這兩道魚總共的 今日時價 每兩 25 元

價錢是 1250 元,則這兩條魚總共重 清蒸 酌加收 25 元
紅燒 酌加收 40 元
兩。 三杯 酌加收 50 元
26.  小丸子的爸爸發一個星期的零用錢給小丸子和姊姊,小丸子先


分到了  200  元,又分到剩下錢數的  ,姊姊分到最後的剩下

錢數,結果小丸子和姊姊得到一樣多的錢,則小丸子分到了

元。 

27.  小梅與家人一起布置慶生會場時,發現  3  條童軍繩與  150  公

分的繩子排成一直線連成的長度,比  4  條半童軍繩排成一直線

連成的繩子少  30  公分,則  1  條童軍繩的長度是 公

分。(繩子不打結)

童軍繩 童軍繩 童軍繩  150 公分

半條童軍繩 童軍繩 童軍繩 童軍繩 童軍繩 

30cm


28.  周星星原有若干元,若他用原有的錢的  買筆記本,原有的


錢的  買文具用品,總共花去  168  元,則他原有

元。 

29.  如果 x – 1 = 7 的解比 x –1 = k  (  k  是一個數)的解大 2,那麼 

k =  。 


30.  甲、乙二生打工所得收入相同。甲用去了打工收入的  ,乙用 


去了打工收入的  :(1)甲、乙二人的支出總和佔他們兩人打工

收入總和的 。(2)設兩人共用去 1800 元,利用解方程

式求出他們二人打工收入為甲: 元、乙: 元。 

31.  父子兩人的年齡各為 44 歲與 18 歲,________年後,父親年齡

為子年齡的 2 倍。 

32.  兄弟三人量體重,已知大哥比二哥重 12 公斤,二哥的體重為小

弟的 2 倍,設小弟的體重為 x 公斤,則(1)大哥體重可列式為 

_______公斤;(2)若兄弟三人體重和為 72 公斤,則可列一元一

次方程式為 ,可解得  x =______,即大

哥體重是__________公斤。 


33.  三個連續偶數,最大數是最小數的  倍,則三偶數為 

________、________、________。 

14 - x
34.  解一元一次方程式:x–  = 8,x=________。 

35.  小明想將濃度  10%  的糖水  180  公克,再加________公克的水

可變成濃度  8%  的糖水。 

36.  一杯由糖  50  公克、水  450  公克混合的糖水,其重量的百分甜

50
度為  ´ 100% = 10%, 若加了  a  公克的水後,重量百
50 + 450 

分甜度為  8%,試問  a =  。 

37.  解下列各一元一次方程式:
x  1 
(1)  -  x = 0 ,x=________。 
3 3 
x  1 
(2)  -  x = 1 ,x=________。 
3 3 
x  1 
(3)  - x + 2 = 5 x ,x=________。 
3 3 

(4)  5 x - 2 × 2 x = x ,x=________。 

38.  解下列各一元一次方程式: 

(1)  3 x = -5 x ,x=________。 

(2)  8 x - 10 x = 10 ,x=________。 


(3)  = 0 ,x=________。 
3

(4)  3 x - 1 = 5 + 3 x ,x=________。 

39.  解下列各方程式: 

(1)  x  + 2 = 11 ,x=________。 

(2)  2 x  + 1 = 15 ,x=________。 

(3)  x - 4 = 12 ,x=________。 

(4)  2 x - 4  = 12 ,x=________。 

40.  若 x 是正整數,試求以下各方程式的解: 

(1)  x + 3 = 5 ,x=________。 

1
(2)  x + 2 = 11 ,x=________。

3 4 x 
(3)  x - 5 = + 1 ,x=________。 
2  3 

41.  若 x 是正整數,試求以下各方程式的解: 

(1)  x - 8 = 10 ,x=________。 

2
(2)  x - 5 = -9 ,x=________。 

3 2 
(3)  x - 4 = x + 2 ,x=________。
7  5 

答案: 

12 
1.  星期三  2. 4  3. 180  4.4  5.26  6.20  7.  8.21 

9.7  10.24  11. (1) – 5(2) 2(3) 34(4) 11  12. (1) ­6 (2) 50  13. 178 
14.17  15.超哥 32 歲、凱哥 31 歲、憲哥 30 歲  16.300  17.39 
7  19 
18.  19. 2;4  20.18  21.(1) –3 (2) –  22.180 
3  4 

23. (1) –15(2)  (3) 5(4) 0(5) 3  24.104  25.47  26.300  27.120 
13 

28.336  29.5  30.  ;4000  31.8 
40 
32. (1)2x + 12(2)2x + 12 + 2x + x = 72(3)12(4)36  33. 16、18、20 

2
34.10  35.45  36.125  37. (1) x = 0 (2)無解(3) x =  (4) x 為任意數 

10
38. (1) x = 0 (2) x =  (3) x = 0 (4) ­1=5\ 無解  39. (1) x = ±9 (2) x = ±7

(3) x = 16 , - 8 (4) x = 10 , - 2  40. (1) x = 2 (2) x = 27 (3) x = 36

41. (1) x = 18 , - 2 Þ 因為是正整數 \ x = 18


(2) x = -6 Þ 無解,因為 x 是正整數(3)  x = 210

三、 綜合、題組題 

1.  父親對孩子說:「我在你這個年齡的時候,你只有 5 歲,等你到

了我這個年紀的時候,我就 44 歲了。」請問父親 3 年後是幾歲?

答案:1.34 

參、 挑戰題

一、 選擇題 
3x ­ 2  7x + m  6x + 11 
1.  (      )  解方程式  ­  ­ 3 = ,得  x = 3,則  m =? 
5  2  10 
(A)­16  (B)­18  (C)­20  (D)­22。 


2.  (      )  佩佩原有 x 元,先用去原有錢的  買 2 本書,再用去原有


錢的  買 5 枝筆,最後剩下 300 元,試問一枝筆多少錢? 

(A)100  (B)150  (C) 180  (D) 210。 

3.  (      )  承上題,買 3 本書,買 10 枝筆,要花多少元?  (A) 1100 

(B)1200  (C)  1300  (D) 1400。 

4.  (      )  雞兔同籠,雞和兔共有 50 隻,腳共有 190 條,請問雞有


多少隻?(A)2 隻(B)3 隻(C)4 隻(D)5 隻。

答案: 

1.A  2. A  3. C  4.D 

二、 填充題 

1.  解下列各方程式: 

(1) 7x –{  4x –[  2x – ( 3x – 1 ) + 5  ]}= 26 ­ 3x,x=________。 

1  1  1 
(2) 1 –  {  [  ( 6x – 1 ) – 7]+ 8  }= 5,x=________。 
4  2  3 

2.  解下列各方程式: 


(1) 5 ( x – 2) + 12 = –6 (  x – 2 ),x =______。 

(2) (3a – 4 ) –[– 11 ( a – 2 ) + 9a]= 4 ( a – 7 ) + 13,a =______。 

3.  解下列各方程式: 


(1) 4.5 +5 ( x – 3.2) = – 4 (  x – 0.5 ),x =______。 

x + 5  15 ­ 2x  4x ­ 3 
(2)  ­  = + 2 ,x =______。 
2  3  6 

4.  小斌要出去旅行,他出發的時候,發現牆上時鐘上的時針與分
針在八點與九點間重合,請問他是______點______分出發的。 

5.  某項工程,若由甲獨自完成需要 9 天,由乙獨自完成需要 12 

天,由丙獨自完成需要 18 天,今乙丙兩人合作 3 天後,丙由甲

代替,則剩餘的工程,需要______天才可以完成。 

6.  若小哲哲騎腳踏車從家中到學校為等速行駛,通常需要 45 分

鐘,若速度每小時再增加 5 公里,即可節省 15 分鐘,則小哲哲

家離學校______公里。 

7.  一船在靜水中每小時航行 12 公里,今在水流速度每小時 4 公里

的河中,由甲地往乙地往返一趟需 12 小時,則甲、乙兩地距離 

______公里。 

8.  第一天耕了整塊地的一半少  5  公頃,第二天耕了餘下的一半多 

2  公頃,第三天耕了  20  公頃後剩下  5  公頃。若此塊地為  x  公

頃,請列出等式:__________________,並求出此塊地為______ 

公頃。 

9.  解以下各方程式: 

(1)  x + 10 = -3 ,,x =______。 

(2) 5 - x  = 7 ,,x =______。 

(3)  x + 2 = 0 ,,x =______。 

10.  解以下各方程式:
(1)  x - y  = 3 , x = 1 則 y = ______。 

(2)  x - y - 1 = 3 , x = 1 則 y = ______。 

(3)  x - 3 = -3 , x = ______。

答案: 

40  9  7 
1. (1) 2 (2) ­16  2. (1)  (2) 11  3. (1)  (2) 8  4. 8 點 43  分 
29  5  11 
5. 3  6. 7.5  7. 64  8. 98  9. (1)無解(2) x = 12 or - 2 (3) x = -2

10. (1) y = -2 , 4  (2) y = ±3 (3)  x - 3  ³ 0 \ 無解

You might also like